You are on page 1of 333

DAMAGE BOOK

00
a:

OU

160464

>m

A CONCISE

GEOMETRY
BY

CLKMENT
SJ.NIOI'

V.

IHJRELL, M.A.

MAIIIFMAIHAI MAS IRK,

\VTN<

111

SI

PR COT Kf>P
I

LONDON
G.

BELL AND SONS LTD.


192

First

Publhhcd

Reprinted

January IQZI
August

IQH

PREFACE
TITE primary object of this text-book is to supply a large number
of easy examples, numerical and theoretical, and as varied in
character as possible, in 'the belief that the educational value of
the subject lies far more in the power to apply the fundamentoKfrte
of geometry, and reason from thorn, than in the ability to reprocfoice

This collection has grown out of a get* of


privately printed geometrical exercises which has been in U&Q, for
many years at Winchester College the author is indebted to many
proofs of these facts.

friends

additions to

for

it,

and

to the following authorities for

permission to use questions taken from examination papgrs


The Controller of His Majesty's Stationery Office ; The Syncujj of
the Cambridge University Press and the Oxford and Cambric^
:

Joint Board.
Riders are arranged in exercises corresponding to groups of
There is also a
theorems, and Constructions are treated similarly.

Answers are given to all numerical


intermediate work is necessary.
Harder questions and papers are marked with an asterisk.
The book is arranged as follows

set of fifty Revision Papers.

questions,

except

where

no

I.

II.

III.

Riders, Numerical and Theoretical.


Practical

Geometry ; Construction Exercises.

Proofs of Theorems.

IV. Proofs of Constructions.

The Proofs

of Theorems and Constructions are collected together


instead of being dispersed through the book in order to assist
revision by arranging the subject-matter in a compact form.
When

learning or revising proofs of theorems and constructions, it is most


For this
important the student should draw his own rough figure.
reason, no attempt has been made to arrange the whole of the

PREFACE

VI

proof of a theorem on the same page as the figure corresponding


to

it.

The order and method of proof is arranged to suit those who


are revising for examination purposes, and is not intended to bo
that used in a first course.
It is now generally agreed that proofs
by superposition

of congruence tests

and proofs

of the fundamental

angle property of parallel lines should be omitted in the preliminary


course, but that these facts should be assumed without formal
proof and utilised for simple applications, the former being treated
by some such method as that noted on page 14, and the latter by
rotation or the set-square method of drawing parallel lines.
This
broadens "the basis of the geometrical work and enables the early
exercises to be of a more interesting nature.

The arrangement of riders in one group and


is made for convenience of reference.

practical

work

in

another

Naturally both
will
in
be
use
but
the
course
should open
groups
simultaneously
with the exercise on the use of instruments in the practical
;

geometry section.
No attempt has been made to include in the text the usual
preliminary oral instruction which deals with the fundamental
concepts of angles,
illustration

lines,

planes,

with simple models.

surfaces,

solids,

The examples

and requires
methods

start with

measurement and general use of instruments, which is the


which a book is really any use for class work.
The object throughout has been to arrange the book to suit the
student rather than the teacher, and " talk " is therefore cut down
to the minimum.
It is the nature of the examples which has been
the chief consideration, and if this part of the book receives
approval, the author will consider his object has been attained.
Valuable assistance has been given by Mr. A. E. BROOMFIBLD,
without whose advice, interest, and encouragement the work
could scarcely have been carried out.
of

earliest stage at

C. V. D.
August, 1920.

CONTENT'S
R1DEHS ON BOOK

......
......9
I

PAUH

ANGLES AT A POINT
ANGLES AND PARALLEL LINES

ANGLES OF A TRIANGLE, ETC.


ISOSCELES TRIANGLE?, CONGRUENT Tin ANGLES ( FIRST SECTION)
CONGRUENT TRIANGLES (SECOND SECTION), ETC.
.

...
.......
RIDERS ON BOOK

AREAS

PYTHAGORAS' THEOREM

SEGMENTS OF A STRAIGHT LINE


.

INTERCEPT THEOREM

....

RIDERS ON BOOK

...
...
......

.56
.60

67
.

CONVERSE PROPERTIES
Loci

.71
.76

.........
.......
......
........
.....
........
.......

iN-ClRCLE, Kx-ClRCLES

ORTHOCENTRE
CENTROID

82

.86

ClRCUMOlRCLE

46

111

PROPERTIES OF EQUAL ARCS


LENGTHS OF TANGENTS, CONTACT OF CIRCLES

MENSURATION

37

43

51

ANGLE PROPERTIES OF A CIRCLE


ANGLE PROPERTIES OF TANGENTS

25

48

SYMMETRICAL PROPERTIES OF A CIRCLE

15

II

.....

EXTENSION OF PYTHAGORAS' THEOREM

iNEtjUALI'IKK

.21

93
99
100
101

103

RIDERS ON BOOK IV

PROPORTION

105

SIMILAR TRIANGLES

Ill

vii

CONTENTS

Vlll

PACH8

RECTANGLE PROPERTIES OF A CIRCLE

AREAS AND VOLUMES (SIMILAR

FIOTJRKB, SOLIDS)

THE BISECTOR OF AN ANGLE OF A TRIANGLE

.120
,126

CONSTRUCTION EXERCISES BOOK

.131
I

.......
.......
...
.........
....

USE OF INSTRUMENTS

.13.5

DRAWING TO SUALK
MISCELLANEOUS

TRIANGLES, PARALLELOGRAMS, ETC

MISCELLANEOUS

II

CONSTRUCTION EXERCISES BOOK

AREAS

MISCELLANEOUS

III

150

.153
155

.158

CONSTRUCTION EXERCISES BOOK


.

148

II

SUBDIVISION OF A LINE

CIRCLES

145

111

.161

.172

.....

CONSTRUCTION EXERCISES BOOK IV


PROPORTION, SIMILAR FIGURES
MEAN PROPORTIONAL

.......
......
........
........
........
........
........
........
.

MISCELLANEOUS

IV

REVISION PAPERS, I-L

175

.178
180
181

PROOFS OF THEOREMS

I
BOOK
BOOK II
BOOK III
BOOK IV

.205
218
234

254

PROOFS OF CONSTRUCTION

BOOK I
BOOK II
BOOK III
BOOK IV

267

276
280
289

NOTES
GLOSSARY AND INDEX

313

ANSWBKS

315

305

A CONCISE GEOMETRY
RIDERS ON BOOK

ANGLES AT A POINT
THEOREM
If a straight line

(i)

(ii)

If lines

that L

CE

cuts a straight line

ACB

at C, then

CA, CB are drawn on opposite sides of a line CE such


ACE + BCE= 180, then ACB is a straight line.
L.

C
Fir,.

1.

THEOREM
If

two straight
equal.

lines intersect, the vertically opposite angles are

CONCISE GEOMETRY

ANGLES AT A POINT
EXERCISE

20, 150, 27* 45', 92 10' 1


75, 30 30', 10 48"?
what is the angle between two

2.

What
What

3.

4.

adjacent spokes ?
Guess the sizes of the following angles

1.

are the supplements of

are the complements of

wheel has six spokes,

FIG.

5.

What

is

17

the least

7.

8.

3.

of times

in order to turn through

reflex angle, (iii)


6.

number

an obtuse angle,

(ii)

more than oue complete revolution 1

10.

(i)

you must turn through

What is the angle between N.K. and S.E. 1


What is the angle between S.S.W. and E.N.E. ?
What is the angle between (i) 12 N. of W. and 5 E.
22 S. of W. and
of N.
S.W. and E.S.E.
(iii)
(ii)
9 N. of E.

9.

Through what angle does the minute hand of a clock turn in


15 minutes, 5 minutes, 20 minutes, 50 minutes, 2 hours
45 minutes t
Through what angle does the hour hand of a clock turn in
40 minutes, 1 hour ^

12.

Through what angle has the hour hand of a clock turned,


when the minute hand has turned through 30 1
What is the angle between the hands of a clock at (i) 4 o'clock,

13.

A wheel

14.

What

11.

ten minutes past four 1


makes 20 revolutions a minute, through what angle
does a spoke turn each second ?

(ii)

equation connects x and y

mentary,

(ii)

supplementary ?

if

x*

and y are

(i)

comple-

ANGLES AT A POINT
15.

The

OA

line

calculate

cuts the line

at

O;

if

LAOB = 2<

AOC,

<_AOB.

17.

What angle is equal


A man watching a

18.

The weight

19.

What

16.

BOC

to four times its

complement?

revolving searchlight notes that he is


in the dark fou** times as long as in the light, what
&ngle of country does the searchlight cover at any

moment ?
in a pendulum clock falls 4 feet in 8 days;
what
angle does the hour hand turn when the
through

weight
is

falls 1

inch

*?

the reflex angle between the directions S.W. and

N.N.W.

makes one complete revolution every 24 hours,


what
angle does it turn in 20 minutes ?
through
The longitude of Boston is 71 W., and of Bombay is 73 E.,
what is their difference of longitude ?
The latitude of Sydney is 33 S., and of New York is 41 K,
what is their difference of latitude 1
Cape Town has latitude 33 40' S. and longitude 18 30' E.,
Cologne has latitude 50 55' N. and longitude 7 E., what
is their difference of latitude and longitude ?

20. If the earth

21.

22.

23.

24.

25

26.

27.

z.POQ = 2^, L.QOR = 3#,

^POR = 4#;

find x.

OP, OQ, OR, OS are 4 lines in order such that L POQ = 68


L QOR * 53, L ROS = 129 ; find L SOP. Find also the
angle between the lines bisecting L POS, L QOR.
Q

OA, OB, OO are 3 lines in order such that L AOB = 54,


L BOC = 24 ; OP bisects L AOC ; find L POB.
CD is perpendicular to ACB ; CE is drawn so that L. DOE = 23 ;
find the difference between L ACE and L BCE.
What is
their

sum 1

CONCISE GEOMETRY
28.

Given

ZAOD =

calculate

ZBOC -

145,

Z.AOC

FIG.

29.

and

77,

ZAOB- ZCOD;

(Fig. 5).

5.

OF are 6 linen in order such that


BOG
Z
-67, Z COD -70, Z DOE -59,
ZAOB -43,
AOD and COF are straight lines.
that
Z EOF =^51; prove

OA, OB, OC, OD, OE,

Calculate the angle between the lines bisecting

ZAOF

and

ZBOC.
30.

ZAOB = 38; AO

produced to C;

is

OP

bisects

calculate reflex angle AOP.


are 4 lines in order such that
31. OA, OB, OC,

OD

= Z BOD
32.

33.

if

Z BOC = #*,

calculate

ZBOC;

Z AOC = 90

Z AOD.

AOB, COD intersect at O; OP bisects ZAOC; if


ZBOC = #, calculate ZDOP.
OA, OC make acute angles with OB on opposite sides;
OP bisects Z BOC prove Z AOB + Z AOC = 2 Z AOP.

Two

lines

34.

BOC at O; OP bisects ZAOB;


=
AOC
Z
prove Z POQ 90.
OQ
in
order such that ZAOB
4
lines
are
OD
OA, OB, OC,
= ZCOD and Z BOC = Z AOD prove that AOC is a
The

line

OA

cuts the line

bisects

35.

straight line.

36.

Given

ZAOB= ZDOC,

6), if

PO

and that

OP

bisects

is produced to Q, prove that

QO

ZAOD
bisects

(see Fig.

Z BOC.

ANGLES AND PARALLEL LINES

ANGLES AND PARALLKL LINES


THEOREM 5
In Fig.
(i)

If

ii)

If

ii)

If

7,

Z.PBC- Z.BCS, then PQ is parallel to RS.


Z. ABQ = /. BCS, then PQ is parallel to RS.
Z.QBC+ Z. BCS = 180, then PQ is parallel

to RS.

-Q

'D
FIG.

7.

THEOREM
In Fig.
If

7,

PQ

is parallel to

RS,

Then (i)/.PBC- Z.BCS (alternate angles).


(ii)/.ABQ~ /.BCS (corresponding angles),
= 180.
(iii) Z. QBC + /. BCS

CONCISE GEOMETRY

ANGLES AND PARALLEL LINES


EXERCISE
1.

II

In the following figures, a line cuts two parallel lines.


equations connect the marked angles 1 Give reasons.

What

FIG. 8.

2.

The following

figures

contain pairs of parallel

equations connect the marked angles 1

lines.

J
3.

What

Give reasons.

iL.

(i)

If

one angle of a parallelogram

is

60,

find its

other

(ii)

If

one angle of a parallelogram

is

90,

find

other

its

angles.
4.

If

AB

is

parallel to

ED, sec Fig.

10, prove that

^.BCD =

FIG. 10.

5.

The side AB of the triangle ABC is produced to D; BX is


drawn parallel to AC; z.BAC = 32, z.BCA = 47; find
the remaining angles in the figure and the value of L BAC -f

_ABC.

ANGLES AND PARALLEL LINES


6.

If

AB

is

'parallel to

/.ODE =180+

DE, see Fig.

11, prove that

Z.ABC +

Z-BCD.
E

Dr

FIG. 11.

[Draw CF
7.

parallel to

DE.]
In Fig. 12, prove that AB

is parallel

to

ED.

E
FIG. 12.

8.

In Fig. 13,

ED

is

if

/.ABC = 74, /.EDO = 38, L BCD = 36, prove

parallel to

AB.
A/

FIG. 13.

9.

10.

ABCD is a
L DAB -

quadrilateral

_
if

ABjis parallel to DC, prove that

DCB = L ABC if AB is parallel to


L

In Fig. 14,
two right angles.

L.

ADC.

DE, prove that # + y -

^"*r-^
FIG. 14.

equals

CONCISE GEOMETRY

8
11.

AC cuts two parallel lines AB, CD; B and D are on


same side of AC ; the lines bisecting the angles CAB,
ACD meet at O ; prove that /. AOC == 90.
If two straight lines are each parallel to the same straight
line

the

12.

line,

prove that they are parallel to each other.

ANGLES OF A TRIANGLE

ANGLES OF A TRIANGLE VNU OTHER


RECTILINEAL FIGURES
THEOKEM
(i)

(ii)

side BC of the triangle ABC is produced


= L BAG + L ABC.
ACD
L
In the AABC, Z.ABC+ Z.BCA+ L CAB = 180.
If

the

to

D,

C
FIG. 15.

THEOKEM 8
(i)

The sum

any convex polygon which


4 right angles.
If the sides of a convex polygon are produced in order, the
sum of the exterior angles is 4 right angles.
has

(ii)

of the interior angles of

sides is

2n

CONCISE GEOMETRY

10

ANGLES OF A TRIANGLE AND OTHER


RECTILINEAL FIGURES
EXERCISE

III

1. In a right-angled triangle, one angle

angle
3.

Two

angles of a

angle
.

A.

5.

jB.

37, what

is

the third

53, what

is

the third

is

triangle are each

L BAC = 43 and L ABC = 109, what is L ACB 1


The side BC of the triangle ABC is produced to D ; L. ABD =
19, L ACD 37, what is L BAC ?
In the quad. ABCD, /.ABC =112, /.BCD = 75, /.DAB =
51, what is ^.CDA?
ABCD is a straight line and P a point outside it ; L PBA =
110, /.PCD =163, find L BPC.
If

7.

Three of the angles of a quad, are equal ; the fourth angle

8.

120; find the others.


Can a triangle be drawn having its

15.

What

angles equal to

the remaining angle of a triangle,

is

if

two

(i)

is

43, 64,

of its angles

50, x

'; (iii) 2x, 3^; (iv) x +10,


(i) 120, 40; (ii)
20 - x degrees 1
). The angles of a triangle are #, 2#, 2# ; find x.
L BCA + L ABC, find
^H. If in the triangle ABC, L BAC
/.BAC.

are

C are the angles


=
B~C 30, find A.

/I2. If A,
.

13.

/1 4.

B,

The angles of a five-sided


x + 40 degrees, find x.
The angles of a pentagon

of a triangle

figure

and

are #, 2#,

are in the ratio

if

a?

A - B = 15,

+ 30,

x -10,

1:2:3:4:5;

find them.
5.

In

AABC, /.ABC = 38, /.ACB = 54; AD is perpendicular


BC AE bisects /. BAC, find L EAD.
ABC = 28; BC is produced to
AABC, /.BAC = 74,

to

43. In
r

X;

the

lines

Find /.BKC.

bisecting

/.ABC and

LACX

meet at K.

ANGLES OF A TEIANGLE
vt7.

11

AABC, i ABC = 32, L BAC = 40; find the angle at


which the bisector of the greatest angle of the triangle cuts

In

the opposite side.


18.

In

AABC,

ABC =110, <LACB = 50; AD is


A to CD produced prove that

pendicular from

19.

DAB =

BC of AABC is produced to D if L ABC= L ACB


ACD = o?, calculate L BAC.
In the quad. ABCD, L ABC -140, L ADC = 20; the lines
bisecting the angles BAD, BCD meet at O; calculate

The base
and

20.

the per-

21. In

if

AABC,

a?

and

L.

the bisector of

ACB = y,

BAC

calculate

BC

cuts

at

if

ABC =

L ADC.

>22wlf the angles of a quad, taken in order are in the ratio


1:3:5:7, prove that two of its sides are parallel.

-3. Each angle


-24.

of a polygon is

Find the sum of the

140; how many

sides has it?

of a 12-sided

interior angles

convex

polygon.
25.

Find the

26. Prove

interior angle of

that

the

convex polygon
27.

Each angle

sum
is

The sum
is

figure.

the interior angles of an 8-sided


sum of those of a pentagon.

of

twice the

of a regular polygon of

of a regular polygon of
find any values of #, y
28.

a regular 20-sided

sides

which

sides is f of each angle

express

will

in terms of #,

and

fit.

of the interior angles of an rc-sided convex polygon


sum of the exterior angles. Find n.

double the

29. In Fig. 16, prove that x

= a 4- b - y.

FIG. 16.

12

CONCISE GEOMETRY

30. In Fig. 17, prove that

x-y^a-b.

FIG. 17.

31. In Fig. 18, express

in terms of a,

6, c.

FIG. 18.

32.

In Fig. 19, express x in terms of

a,

ft,

c.

FIG. 19.

33. In Fig, 20, express

in terms of a,

FIG. 20.

6, c.

ANGLES OF A TRIANGLE
34.

If,

in Fig. 21,

x -fy = 3z, prove that the

triangle

13
is

right-

angled.

35.

Prove that the


relation

36.

IBS.

22 are connected by the

ABC such that


ADC = L BAC.
The diagonals of the parallelogram ABCD meet at O, prove
that L AOB = L ADB + L ACB.
If, in the quadrilateral ABCD, AC bisects the angle DAB and
ADC = L ABC.
the angle DCB, prove that
ABC is a triangle, right-angled at A AD is drawn perpendicular to BC, prove that L DAC = L ABC.
ABCD is a parallelogram, prove that the lines bisecting the
angles DAB, DCB are parallel.
In the AABC, BE and CF are perpendiculars from B, C to
AC, AB BE cuts CF at H prove that L CHE - L BAC.
the quadrilateral ABCD,
^.ABC= _ ADC and
in
If,
L BCD = L BAD, prove that ABCD is a parallelogram.
If in the AABC the bisectors of the angles ABC, ACB meet

is

$7.

reflex angles in Fig.

a + 6 = x + y.

a point on the base

DAG =

BC

of the triangle

L ABC, prove that L

L.

$9.
40.

"il.

^2.

43.

at

I,

prove that L BIC

= 90 + \ L BAC.

CONCISE GEOMETRY

14

The side BC of the triangle ABC is produced to D CP is


drawn bisecting L ACD if L CAB = L CBA, prove that CP
;

is parallel

45.

to

AB.

The side BC of
ABC is produced to D ; the lines bisecting
L ABC, L ACD meet at Q prove that L BQC = \ L BAG.
The base BC of A ABC is produced to D; the bisector of
L BAG cuts BC at K prove L ABD -f L ACD = 2 L AKD.
The sides AB, AC of the triangle ABC are produced to H, K;
the lines bisecting L HBC, L KGB meet at P; prove that
;

^48.

is

any

inside

point

the

triangle

ABC,

prove

that

LBPO/.BAC.
y49. In the quadrilateral ABCD, the lines bisecting L ABC,
= 2 L. BPC.
meet at P, prove that L. BAD + L

BCD

CDA

CONGRUENT TRIANGLES
Given a triangle ABC, what set of measurements must be made in
order to copy it
Measure AB, AC, /.BAG.
'?

1.

This

is

enough to

Therefore

fix.

will be congruent to

This result

is

the size and shape of the triangle.


drawn to these measurements

all triangles

ABC

given as Theorem

2.

Measure BC, Z.ABC, Z.ACB.

3.

Measure BC, CA, AB.

This also fixes the triangle.


This also fixes the triangle.

and

to each other.

3.

[Theorem

9.]

[Theorem

11.]

ISOSCELES TRIANGLES

ISOSCELES TRIANGLES

TRIANGLES

AND CONGRUENT

(FIRST SECTION)

THEOHEM
In the triangles ABC, PQR,
If AB-PQ, AC = PR,

Then

/.BAG-

-.

QPR,

AABC==APQR.

FIG. 23.

THEOREM

In the triangles ABC, PQR,


(i)

(ii)

L PQR, A.ACB = L PRQ,


=
Then AABC APQR.
If BC = QR, L ABC = L PQR, L BAG = u QPR,
Then AABC = APQR.

If

BC = QR, /-ABC=

THEOREM 10

ABC

is

a triangle.

(i)

If

(ii)

If

AB = AC, then L ACB = L ABC


L ACB - L ABC, then AB AC.

B
FIG. 24.

15

CONCISE GEOMETRY

16

ISOSCELES TRIANGLES

TRIANGLES

AND CONGRUENT

(FIBST SECTION)

EXERCISE IV
\.

The

vortical angle of

the base angles


2.

One base

110; what

isosceles triangle is

are

angle of an isosceles triangle

vertical angle
3.

an

is

62

what

is

the

*!

Find the angles of an isosceles triangle if (i) the vertical angle


is double a base angle, (ii) a base angle is double the vertical
angle.

4.

5.

In the triangle ABC, L BAG = 2 L ABC and


36 ; prove that the triangle is isosceles.

L.

ACB -

ABC

C are three points on a circle, centre O L AOB = 100,


BOC = 140, calculate the angles of the triangle ABC.
= AC, find x in terms of y.
Fig. 25, if AB

A, B,

L
6.

In

7.

is

a point on the base

that

x
8.

9.

BD = BA;

if

BC

of the isosceles triangle

Z-BAD = #

and

12.

/i3.

<LDAC=y,

such

express

in terms of y.

ABCDE

is a regular pentagon, prove that the line


bisecting
the angle BAC is perpendicular to AE.
In the triangle ABC, AB = AC ; D is a point in AC such that

AD = BD = BC. Calculate Z.BAC.


PQ bisects AB at right angles, prove that PA = PB.
Two unequal lines AC, BD bisect each other, prove that
AB = CD.
In the quadrilateral ABCD, AB is equal and parallel to DC;
prove that AD is equal and parallel to BC.
A line AP is drawn bisecting the angle BAC; PX, PY are
the perpendiculars from P to AB, AC prove that PX = PY.

10. If the line

'11.

ABC

ISOSCELES TRIANGLES
14.

BC of the triangle ABC, prove


are equidistant from the line AD.
is the
straight line cuts two parallel lines at A, B ;
cutting the
mid-point of AB ; any line is drawn through
that PC = CQ.
; prove
parallel lines at P,
is

the mid-point of the base

that

Yb.

17

B and C

16. If the diagonal


angles
17.

AC

DAB, DCB,

of the quadrilateral

prove that

AC

ABCD

bisects the

BD

at right angles.
arc the mid-points of AB, CD;

ABCD is a quadrilateral; E, F
if L AEF = 90 = L EFD,
prove

bisects

that

AD = BC.

The diagonals

of a quadrilateral bisect each other at right


angles, prove that all its sides are equal.
19. Two lines POQ,
bisect each other, prove that the triangles
18.

ROS

PRS,
20.

21.

PQS

are equal in area.

Two

lines

POQ, ROS

to

meet

at

if

intersect at

OP = OR

and

O SP and QR are produced


OS = OQ, prove TS - TQ.
BC is produced to D so that
;

ABC is an equilateral triangle;


BC = CD prove that /. BAD = 90.
In the AABC, AB = AC; AB is produced to D so that
BD = BC prove that L ACD = 3 ADC.
P is a point on the line bisecting /. BAG through P, a line
is drawn parallel to AC and cutting AB at Q;
prove
AQ = QP.
In AABC, AB = AC D is a point on AC produced such that
BD = BA; if ZCBD = 36, prove BC = CD.
If in Fig. 26, AB = AC and CP - CQ, prove /. SRP - 3 Z. RPC.
;

-82.

33.

#4.

5.

FIG. 26.

o.

The base

BC

of the isosceles triangle

the lines bisecting

Z.ACD=Z.BIC.
2

ABC

is

produced to

Z.ABC and ^lACB meet at

I;

prove

CONCISE GEOMETRY

18
'27.

In the quadrilateral
prove

28.

AB-AD

ABCD,

ABC is an acute-angled triangle AB< AC


;

radius

AB

Z.ADC,

the circle, centre A,

BC

at D, prove that /.ABC + /.ADC


are three points on a circle, centre
;
prove Z.

cuts

^9. A, B,

Z ABC-

and

CB = CD.
= 180.
ABC =

ZLOAB+/.OCB.
<#0.

AC

AB,

two chorda of a

are

BOC

ACB

and
32.

AD

circle,

centre

if

an altitude of the equilateral triangle

is

AB

/.

BAC - 90,

is

another equilateral triangle, prove that


to

a straight line.
AB = AC; the bisectors of the angles
meet at I, prove that IB ~ 1C.

prove that
31. In the
ABC,

DX

ABC
is

ABC

ADX

is

perpendicular

AC.

or

the base of an isosceles triangle ABC ; P, Q are point*


on AB, AC such that AP = PQ = QB = BC ; calculate L. BAC.

'33r~^C
34.

is

BC of the triangle ABC if


BAC = 90.
In the quadrilateral ABCD, AB = CD and ^ABC= L DCB,
prove BAD -= L CDA.
In the A ABC, AB>AC; D is a point on AB such that
AD = AC prove L ABC + L ACB 2 L ADC.
The triangle ABC is right-angled at A; AD is the perI>endicular from A to BC; P is a point on CB such that
CP = CA prove AP bisects L BAD.
D

the mid-point of the base

is

AD = DB,

35.

prove L

L.

'36.

"37.

38.

The

vertical angles of

ary
39.

isosceles triangles are

supplementprove that their base angles are complementary.


two triangles ABC, XYZ which are such that AB XY,

Draw

AC
'40.

two

XZ, L

In the

XYZ

A ABC, AB = AC;
PY

PX,

ABC =

prove

L.

but are not congruent.


is

any point on
P to AB,

are the perpendiculars from

XPB =

L.

42.

43.

In the

ABC

is

any triangle; ABX,

external to

produced

are equilateral triangles

produced;

YPB.

ACY
ABC prove CX = BY.
OA = OB = OC and L BAC is acute

41.

BC
AC

Prove

ABC, AB = AC
- L ADC 2 L BCD.

AB

is

prove L

produced to

BOC == 2 L BAC.
D

prove

L.

ACD

ISOSCELES TRIANGLES
44.

AB

a point on the side

is

CB AC

is

produced

A ABC,

to

AABC

of

1!)

such that

AD = DC =

prove L ECB = 3 L ACD.

BAG is obtuse the perpendicular bisectors


- 180.
of AB, AC cut BC at X, Y
prove L XAY 2 - BAG
In the A ABC, AB = AC and/- BAC>60; tho perpendicular
= 2 ABP.
bisector of AC meets BC at P
prove APB
D is the mid-point of the -side AB of A ABC the bisector of
L ABC cuts the line through D parallel to BC at K prove

45. In the

46.

L.

47.

L.

48. In the

A ABC,

L_

BAG = 90

and

on AB, AC such that AP = AQ


from A to BQ bisects CP.

AB=AC;

P,

are points

prove that the perpendicular

ABC ;
AB, AC of the
A
a
line
to
on
BC
PY
PX,
parallel
any point
through
are produced to meet BC at Q, R ; prove QR = BC.
50. ABC is a triangle; the perpendicular bisectors of AB, AC
meet at O ; prove OB = OC.
49. X,

51.

are the mid-points of the sides

is

ABC

is

prove that the perpendiculars from

are equal.
52. The sides AB,

AC

ABC

of the triangle

the lines bisecting the angles HBC,


that the perpendiculars from to AH,
I

53.

Two
one

54.

ABC

ABC, ACB

a triangle; the lines bisecting the angles

meet at

circles

have the same centre

AB,

AC

are produced to H, K ;
KCB meet at ; prove
A K are equal.

a straight line

PQRS

cuts

the other at Q, R ; prove PQ = RS.


a line AP is drawn on the same side of AC as

S and

circle at P,
is

to

CAP = _ ABC a line AQ


AB as C, meeting BC at Q, such
that L BAQ = L ACB prove AP = AQ.
The line joining the mid-points E, F of AB, AC is produced
to Q so that EF - FG
prove that BE is equal and parallel
B, meeting

is

BC

at P, such that /.

drawn on the same

side of
j

55.

toCG.
56. In the 5-sided figure

each 120
57.

ABC

is

ABODE,

prove that

a triangle

lines are

bisectors of the angles

D, E
prove
ABC.
;

that

DE

the angles at A, B, C,

are

AB + BC = DE.
drawn through C parallel to the
CBA to meet AB produced in

GAB,

equals the perimeter of the triangle

CONCISE GEOMETRY

20

CD are chords of a circle, centre O if Z AOB = 108


L BOG = 60, L COD = 36, prove AB = BC + CD. [From
BA cut off BQ equal to BO join OQ.]
In the triangles ABC, XYZ, if BC = YZ, ZABC=/.XYZ,
AB + AC = XY, prove L BAG = 2 /. YXZ.
In the A ABC, AB = AC and Z.ABC = 2Z.BAC; BC is produced to D so that /. CAD = \ /. BAG CF is the perpendicular from C to AB prove AD = 2CF.
9

58.

AB, BC,

59.

60.

21

CONGRUENT TRIANGLES

CONGRUENT TRIANGLES

(SECOND SECTION),

PARALLELOGRAMS, SQUARES,
THEOREM

ETC.

11

In the triangles ABC, XYZ,


If

AB = XY, BC = YZ, CA = ZX,

Then

FIG. 27.

THEOREM 12
In the triangles ABC, XYZ,

ABC = 90 = Z.XYZ, AC = XZ, AB = XY,


AABC = AXYZ.

If /.

Then

THEOREM 13
If

A BCD

is

Then

a parallelogram,
(i)

(ii)
(iii)

AB = CD and AD = BC.
L DAB = L DCB and L ABC == L ADC.
BD bisects ABCD.
B

FIG. 28.

CONCISE GEOMETRY

22

THEOREM
If the diagonals of the parallelogram

Then

AO = OC

and

14

ABCD

intersect at O,

BO = OD.

THEOREM 16
If

AB

is

equal and parallel to

Then

AC

CD,

equal and parallel to BD.

is

D
FIG. 30.

DEFINITIONS.

parallelogram

is

a four-sided figure whose

opposite sides are parallel.

rectangle

is

a parallelogram, one angle of which

is

a right

angle.

A
A

square

trapezium

a rectangle, having two adjacent sides equal.


rhombus is a parallelogram, having two adjacent sides equal,
but none of its angles right angles.
sides parallel.

is

is

a four-sided figure with one pair of opposite

'CONGRUENT TRIANGLES

CONGRUENT TRIANGLES

(SECOND

23

8wr;*>jr),

PARALLELOGRAMS, SQUARES, Era


EXERCISE V
the sides of a rhombus are equal.

1.

Prove that

2.

Prove that the diagonals of a rectangle are equal.


Prove that the diagonals of a rhombus intersect at right angles.
Prove that the diagonals of a square are equal and cut at right

3.

4.

all

angles.
5.

6.

7.

8.

The diagonals of the rectangle ABCD meet at O


calculate Z.OAD.

Z.

BOC = 44

Prove that a quadrilateral, whose opposite sides are equal,


a parallelogram.

ABCD
ABCD

/.ABC = 56;

is

a rhombus

is

a parallelogram

calculate Z.

prove that

B and D

is

ACD.

are equidistant

from AC.
9.

is

the mid-point of a chord

AB

of a circle, centre

prove

/. OX A = 90.

10.

The diagonals
through

11.

12.

14

CD

AD

is

parallel to

is a point inside the


square ABCD; a square
described on the same side of AE as D ; prove BE

ABC

is

line

and

prove

BC.

any triangle; BY,

cutting a line through


15.

Two straight lines POQ, ROS cut at


PR = QS, prove /. RPO = /. QSO.
In the quadrilateral ABCD, AB = CD and AC = BD;
that

13.

cut at O any
XO = OY.
O if PQ = RS

ABCD

of the parallelogram
at X, Y ; prove
AB,

O cuts

AEFG

is

DG.

CZ are lines parallel to AC, AB


A parallel to BC in Y, Z; prove

AY = AZ.
ABCD is a parallelogram P is the mid-point of BC DP
AB are produced to meet at Q prove AQ = 2AB.
ABCD, ABXY are two parallelograms; BC and BX
different lines
prove that DCXY is a parallelogram.
;

and

16.

are

17.

Two

unequal

that
18.

AB

bisects

The diagonals

AK

is

circles,

XY

centres A, B, intersect at X,
at right angles.

of a square
cut off equal to AC

ABCD
;

prove

cut at

Y; prove

O; from AB a

L AOK = 3 /. BOK.

part

CONCISE GEOMETRY

24

a straight line such that AB = BC = CD; BCPQ is


a rhombus ; prove that AQ is perpendicular to DP.
20. ABCD is a parallelogram ; the bisector of Z. ABC cuts AD at
19.

ABCD

X
21.

is

L BAD

the bisector of

cuts

BC

at

prove

XY = CD.

ABCD is a parallelogram such that the bisectors of Z. s DAB,


ABC meet on CD prove AB = 2BC.
In AABC, ZBAC^90; BADH, ACKE are squares outside
the triangle prove that HAK is a straight line.
The diagonals of the rectangle ABCD cut at O; AO>AB;
the circle, centre A, radius AC cuts AB produced at E if
L AOB = 4 Z.BOE, calculate ,/BAC.
;

22.

23.

24.

ABC is an equilateral triangle a line parallel to AC cuts BA,


BC at P, Q; AC is produced to R so that BQ = CR; prove

25.

that

PR

bisects

CQ.

one point of intersection of two

is

BR

are radii parallel to

circles, centres

is

a straight

In

is

perpendicular to AX.
is a triangle; the bisectors of Z.s

ABC

prove IA bisects /.BAG.

AQ,

AP

line.

27.

A,

same sense as BP,

in the

QPR
AABC, L BAG = 90; ABPQ, ACRS, BCXY are
outside ABC
prove that (i) BQ is parallel to CS;
prove that

26.

and

[From

squares
(ii)

ABC, ACB meet

BR

at

drop perpendiculars to

AB, BC, CA.]


28.

AABC, Z.BAC = 90; BCPQ, ACHK are squares outside


ABC AC cuts PH at D prove AB = 2CD and PD = DH.
In AABC, AB = AC; P is any point on BC; PX, PY are
the perpendiculars from P to AB, AC; CD is the perpendicular from C to AB prove PX + PY = CD.
In AABC, AB = AC; P is a variable point on BC; PQ, PR
are lines parallel to AB, AC cutting AC, AB at Q, R prove
that PQ -f PR is constant.
H, K are the mid-points of the sides AB, AC of AABC; HK
is joined and
produced to X so that HK = KX; prove that
CX
is
(i)
equal and parallel to BH (ii) HK= JBC and HK
In

29.

30.

31.

is parallel

to^BC.

RIDERS ON BOOK

II

AREAS
THEOREM 16
(i)

If

ABCD

and

(ii)

If

BH

is

are parallelograms on the same base and


parallels, their areas are equal,

ABPQ

between the same

the height of the parallelogram ABCD,


area of ABCD = AB BH.
.

FIG. 32.

THEOREM 17
If

AD

is

an altitude

of the triangle

areaof

ABC,

ABC = iAD.BC.

D
FIG. 38.
2,.

CONCISE GEOMETRY

26

THEOREM 18
(i)

(ii)

If

ABC

and

ABD

arc triangles on the

same

parallel**, i.e.

CD

is

parallel to

THEOREM
If

same base and between

the same parallels, their areas are equal,


If the triangle ABC, ABD are of equal area and lie on the
same side of the common base AB, they are between the

the triangle
same base

AB.

19(1)

ABC
AB

and the parallelogram ABXY are on the


and between the same parallels,

area of

ABC =

area of

ABXY.

AREAS
THEOREM
(i)

(ii)

Triangles (or

27

19(2)

parallelograms) on equal bases and between the

same parallels are equal in area,


Triangles (or parallelogram i) of equal area, which are on
equal bases in the same straight line and on the same side
of

it,

are between the

same
C

parallels.

FIG. 36.

MENSURATION THEOREMS
(i)

If the lengths of the parallel sides of

and

b inches,

area of

and

if

a trapezium are a inches

their distance apart is

trapezium = \h

(a -f-

inches,

b) sq. inches.

b
FIG. 37.

(ii)

If the lengths of the sides of a triangle are a,


if $==
(a + b + c),

area of triangle

/s

(a

-)(*- b)

(*

b, c

inches and

c) sq. inches.

CONCISE GEOMETRY

28

AREAS
TRIANGLES, PAKALLELOGKAMS, ETC.

EXERCISE VI
In Fig. 38, AD, BE,

CF

are altitudes of the triangle

ABC.

A ABC,

L ABC = 90, AB - 3", BC = 5* find area of ABC.


AD - 7", BC = 5" find area of ABC.
BE = 5", CF = 6*, AB = 4"; find AC.
AD = 6#*, BE = 4#", CF = 3#*, and the perimeter

1.

In

2.

3.

In Fig. 38,
In Fig. 38,

4.

In Fig. 38,

5.

In

6.

In quad.

7.

Find the area of a triangle whose

of

ABC

is

quad.

Find BC.

18*.

ABCD, AB = 12*,

BC = 1", CD = 9", DA = 8*,

L ABC = L ADC = 90 find the area of ABCD.


ABCD, AC = 8*, 60 = 11", and AC is perpendicular
to BD
find the area of ABCD.
;

In Fig. 39,

ABCD

pendiculars to

is

sides are 3", 4", 5".

a parallelogram

AP,

AQ

are the per-

BC, CD.
B

FIG. 39.
8.

In Fig. 39,

9.

In Fig. 39,
In Fig. 39,

10.

AB = 7*, AQ = 3* find the area


AB = 5* AD = 4", AP = 6* find
AP = 3*, AQ = 2 and perimeter

find its area.

of

/;

ABCD.

AQ.
of

ABCD

is

AREAS
11.

In quad.
if

29

ABCD, BC = 8", AD = 3", and BC

the area of

AABC

18

is

is parallel to AD;
find the area of AABD.

sq. in.,

13.

ABCD, AB = 5", BC = 3", CD = 2", L ABC = L BCD


= 90 find the area of ABCD.
In Fig. 38, AB = 8", AC = 6", BE -5"; find CF.

14.

The area

1 2.

In quad.

is

of

AABC is

36

sq. cms.,

the mid-point of

BC;

pendiculars from D to AB, AC.


the parallelogram ABCD, AB

B
16.
17.

18.

to

to

CD

is

3"

cms.,

AC = 9

find tho lengths of

15. In

pendicular from

AB = 8

= 8", BC = 5";

cms.,

the perthe per-

find the perpendicular

from

AD.

Find the area of a rhombus whose diagonals are 5", 6*.


In AABC, L ABC = 90, AB = 6", BC = 8", CA = 10" ; D is
the mid-point of AC.
Calculate the lengths of the perfrom
B
to
and from A to BD.
AC
pendiculars
On an Ordnance Map, scale 6 inches to the mile, a football
field is

approximately a square measuring \ inch each way.

Find the area of the


19. Fig.

field in acres, correct to

40 represents on a

acre.

scale of 1" to the foot a trough

and

the depth of water in it.


The water is running at 4 miles
an hour ; find the number of gallons which pass any point
in a minute, to nearest gallon, taking 1 cub. ft. = 6
J gallons.

FIG. 40.

20. Fig. 41 represents on a scale of 1 cm. to 100 yds. the plan


of a field ; find its area in acres correct to nearest acre.

FIG. 41.

CONCISE GEOMETRY

30

21. Fig. 42 represents the plan


scale of 1 cm. to 1 ft.
(i)
(ii)

and elevation

a box on a

of

Find the volume of the box.


Find the total area of its surface.

FIG. 42.

22.

The diagram

ol
(Fig. 43), not drawn to scale, represents the plan
The measurements given are in inches.
acres.

an estate of^6|

On what
line

PQ

scale (inches to the mile) is it


divides the estate in half ; find

drawn

The dotted

AQ.

FIG. 43.

23.

Find the area of

ABCD

(Fig. 44) in tormstof #, y, p,

</,

r.

31

AREAS
24.

ABC is inscribed in
ABC in terms of p,

a rectangle
r/,

(Fig. 45)

find the area of

r, s.

45.

Fir..

BCD = 90 Find the length


C to AD in terms of p, q, r.

/.ABC-

46

25. In Fig.

perpendicular from

/.

of the

FIG. 46.

26. In

Fig.

47

OB

a square,

is

is

4*;

OA=12",

AOAB, AOBC, AAOC,

Calculate areas of

ABC

side

a straight

00 = 6*.

and prove that

line.

C
B

FIG. 47.

AAOB, OA=-a, OB = b, Z.AOB = 90; P is a point on


AB PH, PK are the perpendiculars from P to OA, OB

27. In

prove

AD of the rectangle ABCD


Q
AB = ^, AD = y, PB=e, QD=/. Calculate area of PCQ in
are points on the sides

28.

P,

29.

The area

terms of

e,

AB,

/, #, y.

rhombus is 25 sq. cms., and one diagonal


the
half
other; calculate the length of each diagonal
of a

is

CONCISE GEOMETRY

32
30.

Find the area of the triangles whose vertices are


(i)

(ii)

(2,1);

31.

(4, 7).

1);(5,2);<6,5).

(0,0); (a,o);

(b, c).

(v)(0,0);(a,6);(c,d).
Find the area of the quadrilaterals whose
(i)
(ii)

32.

5);

(3, 2); (5, 4); (4, 8).

(iii)(l,
(iv)

(2,

vertices are

(0,0); (3,2); (1,5); (0,7).


(1,3);

(3,

2); (5, 5); (2, 7).

Find in acres the areas of the fields of which the following


field-book measurements have been taken
:

YARDS

YARDS
to
(1) to

to

to

250
200
150
100

80

B 50

40

to

(2) to
to

300
220
200
100
50

60
B 100

50

to

80

to

From A
33.

From A

Find from the formula [page 27] the area of the triangles
whose sides are (i) 5 cms., 6 cms., 7 cms.
(ii)

8*, 15*, 19*.

Find also in each case the greatest altitude.


Calculate
34. The sides of a triangle are 7", 8*, 10*.

its

shortest

altitude.

35.

AX,

BY

prove
36.

ABC

is

prove
37.

Two

40.

ABC;

if

AC = 2BC,

a line parallel to

BC

cuts AB,

AC

at P,

AOB,

COD

intersect at

O;

if

AC

is parallel

to

BD,

AAOD = ABOC.

of ABCD are at right angles, prove


ABCD = \ AC BD.
The diagonals of the quad. ABCD cut at O; if AAOB =
AAOD, prove ADOC = ABOC.
In the triangles ABC, XYZ, AB = XY, BC = YZ, Z.ABC +
Z.XYZ = 180, prove AABC = AXYZ.

The diagonals AC, BD


that area of

39.

the triangle

AAPC = AAQB.

lines

prove
38.

are altitudes of

AX = 2BY.

AREAS
41.

is

any point on tne median

AD

33
prove

AAPB-

drawn through A,

C parallel

of

AABC;

AAPC.
42.

ABCD id a quadrilateral
BD, and through B,

lines are

D parallel to AC

prove that the area of


the parallelogram so obtained equals twice the aroa of ABCD.
43. ABCD is a parallelogram ; P is any point on AD ; prove that
to

APAB APCD = APBC.


-f-

44.

ABC

a straight line;

is

a point outside

is

it;

prove

AOAB __ AB
AOBC~BC"
45.

ABCD

is

a parallelogram

perpendicular from
46.

is

to

any point on

AP;

ABCD-DQ.AP.
ABCD is a parallelogram; P

is

prove

BC DQ
;

is

the

that the area of

any point on BD; prove

APAB = APBC.
47.

ABCD is a parallelogram; a line parallel to BD cuts BC,


DC at P, Q prove AABP= AADQ.
AOB is an angle X is the mid-point of OB Y is the mid= ABXY.
point of AX prove AAOY
If in Fig. 48, AC is perpendicular to BD, prove area of
ABCD - J AC BD.
;

48.

49.

FIG. 48.

AC

50.

ABCD

51.

meets BC produced at P prove that AABP = quad. ABCD.


ABCD is a quadrilateral; E, F are the mid-points of A.B, CD;
prove that AADE + ACBF ABCE -f AADF.

52.

The diagonals

is

a line through

a quadrilateral;

parallel to

of

of equal area

53.

ABCD
cuts

and

BQ

at

PQ

a quadrilateral divide it into four triangles


it is a parallelogram.

prove that

AB = BCCD = PQ; PC
ADQP = 8 AOBC.

are parallel lines;

prove quad.

CONCISE GEOMETRY

34
54. X,

are the mid-points of the sides

that
55.

AXBY = AXCY and

AB,

deduce that

AC of AABC
XY is parallel

Two parallelograms ABCD, AXYZ of


common angle at A X lies on AB
;

prove
to

BC.

equal area have a

prove DX,

YC

are

parallel.

sides AB, BC of the parallelogram ABCD are produced


PCD = AQAD.
any points P, Q ; prove
57. ABC is a
D, E are the mid-points of BC, CA ; Q is any
;

56.

The

to

point in
at
58.

59.

AE;

prove

the line through

PQ

bisects

parallel to

QD

cuts

BD

AABC.

CF of AABC intersect at G; prove that


ABGC = ABGA - AAGC.
In Fig. 49, the sides of AABC are equal and parallel to the
sides of AXYZ
prove that BAXY + ACZX = BCZY.
The medians BE,

FIG. 49.

60.

AQB are equivalent triangles on opposite sides of AB


AB bisects PQ.
ABCD is a parallelogram; any line through A cuts DC at Y
and BC produced at Z; prove ABCY = ADYZ.
In Fig. 50, PR is equal and parallel to AB; PQAT and
CQRS are parallelograms; prove they are equivalent.
ABP,

prove

61.

62.

35

AREAS
63.

BE,

CF

are medians of the triangle

ABGC = quad.
64.

ABC,

ABD

same
to

cuts

AC,

and cut at

In Fig. 51,

prove

BD

at X,

65.

same base and between the


a line through O parallel
;
prove XO = OY.

are triangles on the


BC cuts AD at
;

parallels

AB

ABC

AEGF.

APQR

is

a square; prove

-.
AC
i

AB

BK

66.

AB is parallel to CD P is the mid= 2 AAPD.


ABCD
prove
ABCD is a parallelogram DC is produced to P AP cuts BD
- AAQB = ABCP.
at Q
prove A DQP
In Fig. 52, ABCD is divided into four parallelograms prove
POSD = ROQB.

ABCD

is

a quadrilateral

point of

67.

BC

68.

^~

= AABD.
APR +
69. In Fig. 52, prove
a
three
lines
is
ABC
70.
AX, BY, CZ meet
; any
parallel
where
AB
at
X, Y v Z ; prove
CA,
necessary
produced

AASQ

BC,

AAYZ

71. In ex. 70, prove


72.

ABCD

AXYZ-2AABC.

a parallelogram;
within
the angle
point
is

APBD.

AB

is

CBE;

produced to E; P
prove

is

any

CONCISE GEOMETRY

36
73*.

ABC is a A ACPQ, BCRS are


;

parallelograms outside

ABC

QP, SR are produced to meet at O ABXY is a parallelogram


such that BX is equal and parallel to OC prove that ACPQ
4- BCRS = ABXY.
;

74*. In Fig. 53, ABCD is divided into four parallelograms, prove


that SOQD - BPOR = 2 AAOC,
75*.

a variable point inside a fixed equilateral triangle ABC


PX, PY, PZ are the perpendiculars from P to BC, CA, AB

is

prove that
76*. In
ABC,

is

constant.

L ABC = 90; DBC Is an

equilateral triangle out-

ABC prove AADC - ADBC = \ AABC.


In AABC, Z. BAC - 90
X, Y, Z are points on AB, BC, CA
such that AX YZ is a rectangle and AX |AB prove AX YZ
= fAABC.
Two fixed lines BA, DC meet when produced at O E, F are
points on OB, CD such that OE = AB, OF = CD; P is a
variable point in the angle BOD such that APAB-f APCD
side

77*.

PX -f PY -f PZ

78*.

is

79*. G,

constant ; prove that the locus of P is a line parallel to EF.


H are the mid-points of the diagonals AC, BD of the quadri-

lateral

ABCD; AB and DC

prove quad.

are produced to meet at

P;

PYTHAGORAS' THEOREM

PYTHAGORAS' THEOREM
THEOREM 20
If,

in the triangle

ABC,

Z.

BAG - 90",

Then BA 2 + AC 2 = BC-.

A
Fio. 54.

THEOKEM
If,

in the triangle

21

ABC, BA + AC 2 = BC 2
Then ^.BAC-90
2

37

CONCISE GEOMETRY

38

PYTHAGOKAS' THEOREM
EXERCISE VII

AB = 5", AC =12", calculate BC.


AC = 6", BC= 10", calculate AB.
AB = 7", BC = 9", calculate AC.
AB
A ABC, = AC = 9", 60 = 8", calculate area of
AABC, AB = AC-13", BC = 10", calculate the

1.

In Fig. 54,

2.

3.

In Fig. 54,
In Fig. 54,

4.

In

5.

In

A ABC.
length of

the altitude BE.


6.

7.

Find the side of a rhombus whose diagonals are 6 10 cms.


A kite at P, flown by a boy at Q, is vertically above a point
3

R on

the same level as

8.

height of the kite.


In AABC, AC = 3",

9.

AD

the median

10.

is

is

find the

find the length of

AABC AD = 2", BD = Y DC = 4";


y

prove

AC = 13", BD-S

ABCD.
is 9000 yards, is 5000 yards
what length of the railway is com-

gun, whose effective range

from a straight railway


12.

PQ = 505', QR = 456',

AB = 8", L ACB = 90

a parallelogram;

calculate area of
11.

if

AD.

an altitude of

ABCD

manded by the gun ?


The lower end of a 20-foot ladder

is

10 feet from a wall; how

high up the wall does the ladder reach \


must it be put to reach one foot higher ?
13.

An

14.

15.

AB,

How much

closer

aeroplane heads due North at 120 miles an hour in an east


wind blowing at 40 miles an hour ; find the distance travelled
in ten minutes.

ship is steaming at 15 knots and heading N.W. ; there is a


6-knot current setting N.E. ; how far will she travel in one

hour?

AC are two roads meeting at right angles AB = 110 yards,


AC = 200 yards P starts from B and walks towards A at
3 miles an hour at the same moment Q starts from C and
walks towards A at 4 miles an hour. How far has P walked
before he is within 130 yards of Q
;

16.

Find the distance between the points

(1, 2), (5, 4).

PYTHAGORAS' THEOREM
17.

Prove that the points

whose centre
18.

The

7)

parallel sides of

an

area
1 9.

In

20.

In

21. In

(5,

is (2,

is

32

39

11), (6, 10), (7, 7) lie


find its radius.

on a

circle

and

isosceles

trapezium are

and

5*, II",

its

sq. inches ; find the lengths of the other sides.


8* ; find AB.
60,
90,

AC =
L ACB =
AABC, L ABC =
=
=
AABC, L ABC 90, L ACB 60, AB = 5"; find
= 2", BC = 4", CD = V iind AD.
Fig. 55, AB

BC.

90

ocT

ID
Flo. 55.

22. In quadrilateral

= L BCD = 90

90=
23.

In

Z.AQD

Fig.

56,

ABCD, AB = 5", BC =

are points on
P,
;
calculate BP, BQ.

12", 00 = 7"; Z.ABC


BC such that L APD =

AC = CB = 12", CD = 8*, ZACD = 90;

find

radius of circular arc.

ACB
FIG. 56.

24. Prove that the triangle


is

25.

whose

sides are x^

+ y 1 #2
,

2
,

%xy

right-angled.

AD is an altitude of the triangle ABC BD = 2 DC = y2


AD = xy prove that L BAC = 90.
AD is an altitude of AABC, L BAC = 90 AD = 4", CD = 3^
;

a;

26.

AB.
BC
are
two vertical poles, D and C being the ends on
AD,
the ground, which is level; AC = 12', AB = 10', BC = 3';
calculate

27.

calculate

28.

AD.

AD, BC are the parallel sides of the trapezium ABCD AB =


= 9, CD = 5, AD = 14 find the area of ABCD.
6, BC
In AABC, AB = AC = 10", BC = 8"; find the radius of the
;

29.

circle

which passes through A,

B, C.

CONCISE GEOMETKY

40
30. In

A ABC, AB*=4", 80 =
A ABC, AB*=8", BC =

31.

In

32.

A regular
its

5",

3",

polygon of n sides
is

perimeter

L ABC = 45;
Z. ABC ~60;
is

prove that

calculate
calculate

AC.
AC.

inscribed in a circle, radius r


its

area

is

? jir - jg )
2

Hence, assuming that the circumference of a

circle of radius

2
prove that the area of the circle is Tir
33. The slant side of a right circular cone is 10", and the diameter

is 27ir,

of its base

is

8"

find its height.

Find the diagonal of a cube whose edge is 5".


35. A room is 20 feet long, 16 feet wide, 8 feet high; find the
length of a diagonal.
34.

36.

A piece

37.

of wire is bent into three parts AB, BC, CD each of


the outer parts being at right angles to the plane containing
the other two; AB-12", BC-6", 00 = 12"; find the

distance of

from D.

hollow sphere, radius 8", is filled with water until the


surface of the water is within 3" of the top.
Find the
radius of the circle formed

A circular

38.

is

r feet

is

39.

cone
;

of height

is

by the water-surface.
h feet, and the radius

prove that the radius of

its

of its base

circumscribing sphere

feet -

pyramid of height 8" stands on a square base each edge of


which is 1'. Find the area of the sides and the length of
an edge.

40*.

= 6", BC = 8"; it is folded about


is
a^ rectangle; AB
so that the planes of the two parts are at right angles.
Find the new distance of A from C.

ABCD
BD

41.

AD

is

an altitude of the equilateral triangle

ABC

prove that

AABC, Z.ACB90; CD is an altitude; prove AC 2 +


BD2 = BC 2 + AD 2
ABN, PQN are two perpendicular lines prove that PA2 + QB 8

42. In

43.

'44.

The diagonals AC,


angles

BD of the quadrilateral ABCD


AB 2 + CD 2 = AD8 + BC 9

prove that

are at right

THEOREM

PYTHAGfOKAS'

45. It

the quadrilateral

AB 2 - AD 2

that
46.

a point inside a rectangle

is

PB + PD
2

In

48.

AC
ABCD
;

51.

52.

Is this true

ABCD
P

if

is

ADC = 90

prove that

outside

K are
2
2 =
BK + CH f BC 2

prove that
is a rhombus

/.

prove

H,

PA 2 + PC 2 =

ABCD ?

the mid-points of

AB,

BD 2 = 2AB 2 + 2BC 2
=
In the quadrilateral ABCD, /. ACB
L ADB = 90 AH, BK
are drawn perpendicular to CD; prove DH 2 + DK 2 = CH 2 +
;

prove that

AC?

-l-

CK 2
1)0.

A ABC, L BAG = 90;

&T

49.

ABCD, Z ABC =

CD 2 - CB 2

41

PW

are the perpendiculars from a point P to


the sides of the rectangle ABCD ; prove that PA2 -h PB 2 +

PX, PY, PZ,

PC 2 + PD 2 = 2(PX 2 + PY 2 + PZ 2 + PW 2 ).
In AABC, Z BAG = 90 and AC-2AB; AC is produced to
D so that CD AB; BCPQ is the square on BC; prove
BP-BD.
AD is an altitude of AABC P, Q are points on AD produced such that PD = AB and QD = AC prove BQ = CP.
In AABC, Z.BAC-90
AD is an altitude; prove
;

53.

54.

In

BC
AABC, Z.BAC = 90; AX

is

an altitude; use Fig. 24,

page 15, and the proof of Pythagoras' theorem to show that

BA2 = BX BC
.

;
'

and deduce that

B?T^ABC

is

that

an equilateral triangle; D, E are the mid-points of

2
2
BC, CD prove AE = 13EC
In the AABC, AB = AC = 2BC; BE is an altitude; prove
that AE = 7EC.
O is any point inside AABC; OP, OQ, OR are the perpendiculars to BC, CA, AB; prove BP 2 -f CQ 2 + AR 2 = PC 2 +
QA 2 +RB 2
AD is an altitude of AABC E is the mid-point of BC
2
2
prove AB - AC = 2BC DE.
.

59.

that

58.

AABC, Z.BAC = 90; AD is an altitude; prove


AD 2 = BD.DC.
ABC is an equilateral triangle D is a point on BC such
BC = 3BD prove AD 2 = \ AB 2

55. In

56.

=
f^
AC CX

60.

42

CONCISE GEOMETRY

61. Fig. 57 shows a square of side a 4-6 divided up;


formulae to prove Pythagoras' theorem a2 4- 6 2 = c 2 .

use area

FIG. 57.

62*.

ABC

is

a straight

same side of
63*. The diagonal
point

P;

AC
AC

line

ABXY, BCPQ are squares on


PX 2 4- CY 2 = 3(AB 2 4- BC 2 ).

prove
of the rhombus

prove that

AC

ABCD

PA PC = PB 2 - AB 2
.

ABCD

is

produced to any

64*.

The diagonal

65*.

PC = BC prove PB 2 = PA AC.
In AABC, Z.BAC = 90; BCXY, ACPQ, ABRS
outside ABC prove PX 2 4- RY 2 = 5BC 2
that

of the square

is

the

produced to P so

are squares

EXTENSIONS OF PYTHAGOKAS' THEOREM

43

EXTENSIONS OF PYTHAGORAS' THEOREM


THEOREM 22
In

A ABC,
to

Z.BAC

if

BA

is

obtuse and

produced,
then BC 2 =

if

CN

is

the perpendicular

BA 2 + AC 2 + 2BA AN.
.

FIG. 58.

THEOREM 23
In

AABC, if L BAG is acute, and if CN is


AB or AB produced,
then BC 2 = BA 2 + AC 2 - 2BA

the perpendicular to

FIG. 59(1).

AN.

FIG. 59(2).

THEOREM 24
In

AABC,

if

AD

is

then

a median,

AB2 + AC2 = 2 AD 2 + 2DB 2


A

D
FIG. 60.

CONCISE GEOMETRY

44

EXTENSIONS OF PYTHAGORAS' THEOREM


EXERCISE VIII
1.

Find by calculation which of the following triangles are


being as follows

obtuse-angled, their sides


(ii)

2.

7, 8,

11;

(iii)

8,9, 12;

(iv) 15,

4.

5.

5,

Find the area

7.

ABCDis
of

In

AC on AB is 6"
AABC, AC8

10.

The

11.

In

whose sides are

AB = 5", AD=

altitude;

altitude;

BC = 6

cms.,

9", 10*, 11".

3"; the projection

AC.

calculate

cms.,

/.ACB=120;

AB.

AABC, AB *= 8

In

12. In

of* the triangle

a parallelogram

calculate

altitude;

AN, BN, CN.

6.

9.

4,

In A ABC, BC = 6, CA-3, AB = 4; CN is an
calculate AN and CN.
In AABC, BC = 8, CA=9, AB = 10; CN is an
calculate AN and CN.
In AABC, BC = 7, CA=13, AB = 10; CN is an
calculate

8.

(i)

Each of the sides of an acute-angled triangle is an exact


number of inches; two of them are 12', 15"; what is the
greatest length of the third side

3.

16,22.

cms.,

AC = 7, BC = 3

sides of a triangle are 23, 27,

36;

prove

is it

L ABC = 60.

obtuse-angled?

AABC, AB = 9*, AC=ir, </ BAC>90; prove BOH".


AABC,AB = 14",BC = 10",CA = 6"; prove L ACB - 1 20.

calculate the length of the

13.

The

14.

Find the lengths of the medians of a triangle whose sides

15.

The

sides of

are 4, 7, 9;

shortest median.

are 6, 8, 9 cms.
sides of a parallelogram are
is

16.

;
diagonal
is a median of the

In

cms., 7 cms.,

AABC, AB = 6, AC = 8, AD = 5;

AABC, AB = 4, BC = 5, CA = 8; BC
CD = 5 calculate AD.

ABC is an
BC = CD

and one

BC.

that

18.

'5

find the length of the other.

AD

calculate
17.

8 cms.

is

produced to

so

equilateral triangle;
;

prove

AD

3AB

BC

is

produced to

so that

EXTENSIONS OF PYTIIAGOKAS' THEOREM


19.

A ABC, AB = AC;

In

CD

is

45

BC2 =

an altitude; prove that

2AB BD.
.

20.

AB

DC

and

prove that

are the parallel sides of the trapezium

AC2 + BD 2 = AD 2 -f BC 2 + 2AB

23.

24.

CF are altitudes of the triangle ABC prove that


AF.AB-AE.AC.
ABCD is a parallelogram; prove that AC -f BD 2 = 2AB 2 + 2BC 2
ABCD is a rectangle P is any point in the same or any
other plane prove that PA 2 -h PC 2 = PB 2 f PD 2
In A ABC, AB = AC; AB is produced to D so that AB^BD;
2
2
2
prove CD = AB + 2BC

21. BE,

22.

ABCD

DC.

AABC, D, E are the mid-points of CB, CA; prove that


2
2 CB 2
4(AD* BE ) = 3(CA
26. In AABC, Z.ACB = 90; AB is trisected at P, Q; prove
that PC 2 + CQ 2 + QP 2 = AB 2
27. The base BC of AABC is trisected at X, Y; prove that
AX 2 + AY2 4- 4XY 2 = AB 2 -h AC 2
28. The base BC of AABC is trisected at X, Y; prove that
AB 2 - AC 2 = 3( AX 2 - AY2).
29. AD, BE, CF are the medians of AABC;
prove that
2
2
2
2
2
2
4(AD + BE -h CF ) = 3(AB + BC + CA
30. ABCD is a quadrilateral
X, Y are the mid-points of AC,
BD prove that AB 2 -f BC 2 + CD 2 -f DA 2 = AC 2 + BD 2 +
4XY 2
31*. ABC is a triangle ABPQ, ACXY are squares outside ABC
2
2
2
2
prove that BC -h QY = AP + AX
32*. ABC is a triangle D is a point on BC such that p BD =
2
2
2
2
0.DC; provethatjp.AB -f^.AC --=(/>4-g)AD H-jp. BD -f
2
q DC
33*. AB is a diameter of a circle; PQ is any chord parallel
to BA
O is any point on AB prove that OP 2 + OQ 2 =
25.

In

).

).

34*

ABCD

is

prove that

a tetrahedron

BCD

is

L BAC = L CAD = L DAB

an acute-angled

triangle.

* 90

CONCISE GEOMETRY

46

RELATIONS BETWEEN SEGMENTS OF A


STRAIGHT LINE
EXERCISE IX

2.

AB is bisected at O P is any point on AO


prove PO~J(PB-PA).
A straight line AB is bisected at O and produced to P prove

3.

1.

straight line

5.

AB is bisected at O and
PA2 + PB 2 = 2PO 2 + 2AO 2

straight line

that
4.

ABCD

produced to P

prove

a straight line ; X, Y are the mid-points of AB,


that
AD + BC = 2XY.
prove
is

CD

AB is bisected at O and produced to P prove that AO AP


OB.BP + 2AO 2
AD is trisected at B, C; prove that AD 2 = AB 2 + 2BD 2
APB is a straight line; prove that AB 2 + AP2 = 2AB AP + PB 2
AB is bisected at C and produced at P; prove that AP 2 =
4AC.CP + BP 2
ABCD is a straight line if AB = CD, prove that AD 2 + BC 2 =
2AB 2 + 2BD 2
X is a point on AB such that AB.BX = AX 2 prove that
= 3AX 2
C is a point on AB such that AB.BC = AC 2 prove that
AC.BC = AC 2 -BC 2
X is a point on AB such that AB BX- AX 2 O is the mid2
2
point of AX prove that OB = 5 OA
.

6.
7.

8.

9.

10.

11.

12.

13.

AB

is

bisected at

prove that
14.

and produced to P so that

PA 2 = 5PB 2

OB OP = BP 2
.

AB is bisected at C and produced to D so that AD 2 ==3CD 2


BC is bisected at P prove that PD 2 = 3PB 2
AB is produced to P so that PA2 = 4PB 2 + AB 2 prove that

15.

PA_5
PB~2*

RELATIONS BETWEEN SEGMENTS


16.

ACBD

is

a straight line such that

point of

AB

.=

CB

BD

prove that
(i)
(ii)
(iii)

DA.DB = DC.DO.
AB.CD-2AD.CB.
OB 2 = OC.OD.
1

(iv)'
v

^ AD-^-AB

AC

-.

47
is

the mid-

CONCISE GEOMETRY

48

INEQUALITIES
THEOREM 26
In the triangle ABC,
(i)
(ii)

If

AOAB,

then /_

ABO

IfZ ABOZ.ACB, then

AOAB.

61.

THEOREM 27
If

ON

is

the perpendicular from any point

is

to a line

any point on AB,

ON < OP.

then

FIG. 62.

THEOREM 28
If

ABC

is

a triangle,

BA + AC >BC.

AB, and

if

49

INEQUALITIES

INEQUALITIES
EXERCISE X
1.

The
if

bisectors of the angles

ABC, ACB

of

AABC

meet at

AB >AC,

prove that IB >IC.


if BC<?AD, prove that
a median of

5.

AD is
Z.BAC
AABC;
<90.
ABC is an equilateral triangle P is any point on BC prove
AP>BP.
In AABC, the bisector of Z.BAC cuts BC at D; prove
BA>BD.
AD is a median of AABC; if AB>AC, prove that /.BAD

6.

In

2.

3.

4.

7.

<Z.CAD.
AABC, AB

AC; BC is produced to any point D; P is


DP cuts AC at Q prove AQ > AP.
In the quadrilateral ABCD, AD > AB >CD > BC prove /. ABC
any point on

AB

> Z. ADC.

Which

the greater, Z. BAD or /. BCD ?


the external bisector of L B AC cuts
is

8.

ABC

9.

produced at D; prove (i) AB>AC;


ABC is a triangle ; the bisector of /_

10.

11.

is

a triangle

14.

15.

16.

CD>AC.
BAC cuts BC

BC

(ii)

at

if

AB>AC, prove BD>DC.


ABC is an acute-angled triangle, such that /. ABC = 2 21 ACB
prove AC<2AB.
ABCD is a quadrilateral prove that AB + BC + CD >AD.

12. Prove that


13.

any

How many

side of a triangle

is less

than half

its

perimeter.

drawn such that two of the sides


are of lengths 4 feet, 7 feet, and such that the third side
contains a whole number of feet ?
ABC is a ; D is any point on BC prove that AD< (AB +
triangles can be

BC + CA).
ABCD is a quadrilateral
AD>CD.
ABC is a A P is any
;

AB<BC; /.BAD</.BCD;

point on

BC;

prove that

AP

prove
is

less

than one of the lines AB, AC.


17.

is

any point inside the triangle

> L BAC
18. A,

B
4

are

(ii)

ABC;

prove that (i)/_BOC

BO + OC< BA + AC.

any two points on the same

side of

CD,

A'

is

the

CONCISE GEOMETRY

50

image of A in CD ; A'B cuts CD at O ; P


on CD ; prove that AP + PB > AO + OB.

is

any other point

19.

AD

20.

21.

CA + AB).
In AABC, BC>BA; the perpendicular bisector OP of AC
cuts BC at P Q is any other point on OP
prove AQ + QB

is

is

a median of

any point

AABC;

inside

prove

AABC

AD<|(AB-f AC).
OA + OB -f OC > |(BC

prove

-f-

>AP + PB.
22.

Prove that the sum of the diagonals of a quadrilateral is


greater than the semiperimcter and less than the perimeter
of the quadrilateral.

THE INTERCEPT THEOREM

51

THE INTERCEPT THEOREM


THEOREM 29
If H,

are the mid-points of the sides AB,

then

(i)
(ii)

HK is parallel
HK = |BC.

to

AC of

the triangle

ABC,

BC.

B
FIG. 63.

THEOREM 30
If

two

lines

DS

ABCDE, PQRST

so that

BC = CD,

are cut

then

by the

7\
z
:/

parallel lines

QR = RS.

\a

R_

\T
Fio. 64.

BQ, CR,

CONCISE GEOMETRY

52

THE INTERCEPT THEOREM


EXERCISE XI
1.

P is any
H, K are the mid-points of AB, AC
BC prove HK bisects AP.
In AABC, L BAG = 90
D is the mid-point of BC; prove
that AD = BC.
[From D, drop a perpendicular to AC.]
In Fig. 65, if AC = CB and if AP, BQ, CR are parallel, prove

ABC

is

point on

2.

3.

4.

if AC = CB, and
=
CR
(BQ - AP).

In Fig. 66,
that

"

if

AP, BQ,

CR

are parallel, prove

c
V

FIG. 66.

5.

Qi R, S are the mid-points of the sides AB, BC, CD, DA of


the quadrilateral ABCD ; prove that PQ is equal and parallel

P.

toSR.
6.

ABC = 90; BOX is an equilateral triangle;


X parallel to AB bisects AC.
ABC is a A H, K are the mid-points of AB, AC BK, CH
are produced to X, Y so that BK
KX and CH HY prove
thatXY = 2BC.

In

AABC,

/.

prove that the line from

7.

THE INTERCEPT THEOREM


8.

a fixed point

is

a variable point on a fixed line

is

is

is

Prove that the


of

a fixed point ; P is a variable point on a fixed circle, centre


that the loo as of the mid-point of OP is a circle
; prove

whose centre
10.

AB

OP.

find the locus of the mid-point of


9.

53

at the mid-point of

mid points of opposite

lines joining the

any quadrilateral

OA.
sides

bisect each other.

11. If the diagonals of a quadrilateral are equal

and cut at right

angles, prove that the mid-points of the four sides are the
corners of a square.
12.

ABCD

is a quadrilateral; if AB is parallel to CD, 'prove


that the mid-points of AD, BC, AC, BD lie on a straight

line.

13.

ABC

is

AX,

AY

bisectors of the angles


to BC.
14.

ABCD

are the perpendiculars from A to the


is parallel
ABC,
prove that

15.

AD,
P,

16.

BE

ABCD
P,

are

Q, R

altitudes

is

BD

bisects

/.ABC and

the perpendicular from

of

A ABC

that

and

PQ = RS;

(i)

at

intersect

BC

HA, AB,

having
quadrilateral,
are the mid-points of

R,

prove

AH

are the mid-points of

is

Q,

to

AH = 2CD.

prove

XY

a quadrilateral such that

is

/.ADB = 90= /.BCD

BC

ACB

AB

H;

prove that

to

parallel

CD;

AD, BD, AC, BC;


PS = (AB + CD) ; (Hi)

(ii)

QR = |(AB~CD).
17

ABC

is

is

the mid-point of

BC

perpendicular from B to the bisector of

18.

ABC

is

is

the mid-point of

the foot of the

is

Z. BAG

BC; Q

is

prove that

the foot of the

perpendicular from B to the external bisector of


= (AB + AC).
prove that DQ
19.

ABCD

is

a quadrilateral having

mid-points of AD, AC, BD,


dicular to

QR.

AB=^CD;

BC

P,

prove that

Q, R,

PS

/.

S
is

BAC

are the

perpen-

CONCISE GEOMETRY

54
20.

In Fig. 67,
and AP =

if

BD = DC

and

AP = AQ,

BP = CQ

prove that

PIG. 67.

aqiKre box

ABCD,

each edge 13*, rests in the rack of a

and against the wall the point of contact


^ailwa^tearriage
E ,is iB^froin the wall CE = ED. Prove that C is 5" from
the* ^apfj),nd find the distances of A, D from the wall.
:

FIG. 68.

22.

ABC
CF

is

at

E,

G; AG

is

AC AB BE
AG GX

are the mid-points of


produced to X so that

cuts

and

BC at D; prove that (i) GBXC is a parallelogram;


= |GA = iDA.
DG
(ii)
23. ABCD is a parallelogram
XY is any line outside it AP, BQ,
CR, DS are perpendiculars from A, B, C, D to XY prove
that AP + CR = BQ + DS.
24* The diagonals AC, BD of the square ABCD intersect at O
cuts

THE INTERCEPT THEOREM


the bisector of

/.

BAG

BO

cuts

at X,

BC

55

at

prove that"

CY=20X.
25*.

Two

equal

circles,

circle passes

prove that

through

AB

is

O ; a third equal
the
and cuts
former circles at C, D ;

centres A, B, intersect at

equal and parallel to CD.

26*. A, B are fixed points;


are squares outside

is

a variable

APAB

27*.

jx>int

PAST,

PBXY

provo that the mid-point of

SX is fixed. [Drop perpendiculars from S, X to AB.]


ABCD is a quadrilateral having AD = BC E, F are the midpoints of AB, CD
prove that EF is equally inclined to A^
and BC. [Complete the parallelogram DABH
;

at

join

BK, KF.]

RIDERS ON BOOK

III

SYMMETRICAL PROPERTIES OF A CIRCLE


THEOREM

AB

is

a chord of a
(1) If

(2) If

ON

is

circle,

31

centre O.

the mid-point of AB, then L ON A -90.


is the perpendicular from O to AB, then AN

= NB.

THEOREM 32
AB,

CD

are chords of a circle, centre O.

(1) If
(2) If

AB = CD, then AB and CD


AB and CD are equidistant

A corresponding property holds


56

are equidistant from O.

from O, then

for equal circles.

AB = CD.

SYMMETRICAL PROPERTIES OF A CIRCLE

57

SYMMETRICAL PROPERTIES OP A CIRCLE


EXEEOI8E XII
1.

AB

a chord of a circle of radius 10 cms.;

is

find the distance of the centre of the circle


2.

A chord of

3.

cms.;

the
length 10 cms. is at a distance of 12 cms. from
centre of the circle ; find the radius.

chord of a circle of radius 7 cms.

from the centre


4.

AB = 8

from AB.

ABC

is

at a distance of

is

4 cms.

find ita length.

inscribed in a circle;

AB = AC = 13", BC==10";

calculate the radius of the circle.


5.

In a
of

5 cms., there are two parallel chords


6 cms. ; find the distance between

circle of radius

4 cms.,

lengths

them.
6.

Two

parallel chords

CD = 10"

7.

An

8.

The perpendicular
the

ABC

AB = 4",

bisector of a chord

is

AB

6 cms.,

is

inscribed

cuts

AB

at

and

calculate the radius of

is

find

such that

line,

circle

on

AC

AB = T, BC = 4"; PBQ

is

as diameter, perpendicular to

PQ.

a point on the diameter AB of a circle ; AP = 2", PB = 8* ;


find the length of the shortest chord which passes through P.

11.

The

is

centres of

apart

Two

P,

S and

variable line
a",

two

circles of radii 3", 4" are at a distance 5*

common

find the length of their

chord.

concentric circles are of radii 3*, 5"; a line

one at

which

AB = 6", CD = 1*

a straight
the chord of the

10.

13.

of a circle are 3" apart

circle.

AC;

12.

CD

equilateral triangle, each side of


in a circle ; find its radius.

the circle at

9.

AB,

calculate the radius of the circle.

at P,

equation
constant.

the other at Q,

PQRS

cuts

S and

between

x,

Q,
y,

two

R;
a,

R;

if

QR = 2",

PQRS

find

cuts

PQ.

fixed concentric circles of radii


if

&,

PQ = #", QR = /, find
and prove that PQ.QS

an
is

CONCISE GEOMETRY

58
14.

A crescent
Fig. 71)

at C,

15.

formed of two circular arcs of equal radius (see


the perpendicular bisector of AB cuts the crescent
if CD = 3 cms., AB = 10 cms., find the radii.

is
;

In Fig. 72,

ABCD

is

BC
arc

AB = CD = #;

the section of a lens;

BP = PC = y; PQ = z; AB, QP, DC
calculate in terms of r, y,

perpendicular to
the radius of the circular
are

AQD.
C

A
Fro. 72.

16.

AB

is

a chord of a

from
17.

Two

A and B

circles,

bisects
18.

Two

XY

circle,

prove

centres A,

centre

OT

O T

is

bisects /.

any point equidistant

ATB.

B, intersect at X,

prove that

AB

at right angles.

circles,

line parallel

centres
to

AB

A, B, intersect at C, D;
cutting the circles at P,

PCQ is a
Q prove
;

PQ = 2AB.
1 9.

Two

circles, centres

one
20.

21.

ABC

A, B, intersect at X,

circle, parallel to

PQRS cuts
RS.
prove PQ
line

is

XY

prove

two concentric

a triangle inscribed in a

that the mid-point of

BC

is

AB

PQ

bisects

circles at

circle

if

/.

is

a chord of

PQ.
P,

S and

Q, R

BAC=* 90, prove

the centre of the circle.

SYMMETRICAL PROPERTIES OF A CIRCLE


In Fig. 73,

22.

PQ

if

is

parallel to RS, prove

59

PQ = RS.

Fre. 73.

23.

CPD

APB,

OP
24.

are intersecting chords of a circle, centre

if

Z.APC, prove AB=-CD.

bisects

The diagonals of the


are drawn through

quadrilateral

ABCD meet at O circles


D, O, A
O, C C, O, D
;

;
;
B,
A, O,
;
;
prove that their four centres are the corners of a parallelogram.
25. AOB,
are two intersecting chords of a circle ; if AB = CD,

COD
prove AO = CO.

26. In Fig. 74, A, C,


if

AC = CB,

prove

are the centres of three unequal circles

Fig.

27.

PQ = RS.

74.

CD are two chords of a circle, centre O; if AB>CD,


O is nearer to AB than to CD.
Two circles, centres A, B, intersect at C, D PCQ is a line
AB,

prove

28.

cutting the circles at P,

prove

PQ

is

greatest

when

it is

AB.
29*. P is any point on a diameter AB of a circle QPR is a chord
such that L APQ = 45 ; prove that AB 2 = 2PQ 2 + 2PR
parallel to

'.

30*.

ABC

is

inscribed in a circle, centre

images of O in BC, CA,


bisect each other.
31*. AB,

CD

are

prove that

AB

AC + BD - 4OA2
2

are the

X, Y,
;
prove that AX, BY,

two perpendicular chords of a


2

circle,

centre

CZ

CONCISE GEOMETRY

60

ANGLE PEOPERTIES OF A CIRCLE

(1)

THEOREM 33
If

AB

is

an arc of a

circle,

centre O,

and

if

is

any point on the


which

remaining part of the circumference, then the angle


arc AB subtends at
equals 2 /. APB,

FIG. 75(1).

Fio. 75(2).

THEOREM 34
(1) If
(2) If

APQB is a circle, L APB= Z.AQB.


AB is a diameter of a circle APB, /. APB = 90.

FIG. 76.

FIG. 77.

ANGLE PROPERTIES OF A CIRCLE

61

(1)

THEOREM 35
(1) If

ABCD

is

(2) If the side


to P,

a cyclic quadrilateral,

AD

Z.ABC+ /.ADC =

of the cyclic quadrilateral

= Z.ABC.

FIG. 78.

ABCD

is

180.

produced

CONCISE GEOMETRY

62

ANGLE PROPERTIES OF A CIRCLE

(1)

EXERCISE XIII
1.

2.

ABC is a A inscribed in a circle, centre O; Z.AOC = 130


Z. BOG = 1 50, find L ACB.
Z. BAC = 35,
AB, CD are perpendicular chords of a circle
;

find
3.

4.

5.

Z.ABD.

ABCD is a quadrilateral such that AB = AC = AD if L BAD


= 140, find /.BCD.
ABCD is a quadrilateral inscribed in a circle AB is a
diameter; Z ADC -127; find ZBAC.
Two chords AB, CD when produced meet at O Z. OAD = 31
;

ZAOC-42

6.

ZOBC.
Two circles APRB, AQSB intersect at A, B PAQ, RBS are
= 70, find Z.PQS,
straight lines; if ZQPR-=80, ZPRS

7.

P,

find

Z.QSR.
Q,

are points of a circle, centre


;
P, R are on opposite sides of

= 36;
8.

9.

L POQ = 54, Z OQR


OQ find ZQPR and
;

Z.PQR.
The diagonals

of the cyclic quadrilateral ABCD meet at O


L BAC -42, ZBOC = 114, ZADB = 33; find /.BCD.
ABCD is a cyclic quadrilateral, EABF is a straight line;
Z.EAD = 82, ZFBC-74
ZBDC = 50; find angle be;

tween AC, BD.


10.

Two
at

DC of a circle, centre O, are produced


L AOB - 100, L EBC - 72, L ECB - 84

chords AB,

Z.COD.
11.

(i)

In Fig. 79,

(ii) If

if

y + z = 90,

z - 40, find
=
x
45.
prove

y = 32,

x.

to
;

meet
find

ANGLE PROPERTIES OF A CIRCLE


12.

a point on the baso BC of


the circles ADB, ADC;

is

of

find
13.

15.

if

AC cuts BD at O, if
L BAG = 2 L BCA.

XY arc parallel chords of


= OY.
prove OX
Two circles BAPR, BASQ cut at
AB,

lines; prove
16.

AB

18.

19.

20.

21.

/_

are the centres

ABCD

a circle;

A,

AY

BX

cuts

PAQ, RAS

/.BOC-

at

O;

are straight

P is any point on the


centre O
- 360.
2
APB
AOB
+
Z.
prove /_
if
AC
bisects
the angles at
cyclic quadrilateral

AB

is

y = 20, 2-40,

PBR-- Z.QBS.

a chord of a

is

minor arc
1 7.

H,

Z. AMD -70, Z.AKD = 80,

if

Z.BAC.

In Fig. 79,

100, prove

14.

A ABC

63

(1)

circle,

A and C, prove /.ABC = 90.


Two lines CAB, OCD cut a circle at A, B and C, D; prove
Z.OAD= ZOCB.
AB is a diameter of a circle APQRB prove Z.APQ+ Z.QRB
-270.
ABCDEF is a hexagon inscribed in a circle prove that /. FAB
+ Z.BCD+ Z.DEF-360".
Two circles ABPR, ABQS cut at A, B PBQ, RAS are straight
lines
prove PR is parallel to QS.
are three points on a circle, centre O
C
A, B,
prove that
=
+
OCA.
BAC
OBA
L
L
L
;

22.

23. A, B, C,

whose

are four points on a circle

sides are parallel to PA,

24.

AABC.
AP, AQ are diameters of
PBQ is a straight line.

25.

OA

is

PB,

the circles

prove that a triangle

PC

APB,

is

equiangular to

AQB

AP is any
circle, centre O
OA as diameter bisects AP.
AOB, COD of a circle intersect at O

a radius of a

prove that

chord

prove

that the circle on


26.

Two

chords

prove
27.

APC

is

if

AO = AC,

DO = BD.
an

AQOC
2LPCQ.

is

arc, less

than a semicircle, of a

another circular arc; prove

Z.

circle,

APC=

centre

O;
/.PAQ +

64
28.

29.

CONCISE GEOMETRY
a

point of

BC

ABC

is

OA, OB,

prove

30.

Two

31.

OB = BD,
ABCD is a

lines

OAB,
prove

ABCD

is

37.

is

the mid-

Z.AOB = 90,
A,

AB,

CD

the centre of the circle;

DC
at X

AD,

are produced to

B and

C,

EX F

BC

prove

prove

D;

if

are

meet at F

Z.AOC +

produced to
;

the circles

a straight line.
; prove
are perpendicular chords of a circle, centre O
cut

is

prove

Z.DAB= Z.OAC.
In AABC, AB = AC; ABD is an equilateral triangle; prove
that L BCD = 30 or 150.
ABC is a A D is a point on BC H, K are the centres of
the circles ADB, ADC; if H, D, K, A are concyclic, prove
L BAC = 90.
ABC is a A the bisectors of Z.s ABC, ACB intersect at
and meet AC, AB at P, Q if A, Q,
P are*concyclic, prove
=
BAC
60.
L
Two lines EBA, ECD cut a circle ABCD at B, A and C, D
O is the centre prove L AOD - /. BOC = 2 /. BEC.
ACB, ADB are two arcs on the same side of AB a straight
;

I,

38.

if

circle at

a cyclic quadrilateral;

EDC, FBC

36.

is

rectangle; any circle through A cuts AB, AC, AD


at X, Y, Z ; prove that ABD, XYZ are equiangular triangles.

meet at E ; AB,

35.

OCD cut
OC = CA.

O D

Z. BAC.

are three equal lines;


45 or 135.

32. In Fig. 80,

34.

/. BOD =

OC

Z ACB =

33.

inscribed in a circle, centre

39.

line

ADB

ACD
lies

cuts

them

at C,

D;

on the arc ACB, prove

if

the centre of the circle

CB = CD.

ANGLE PROPERTIES OP A CIRCLE

65

(1)

a quadrilateral inscribed in a circle BA, CD when


produced meet at E; O is the centre of the circle EAC;
prove that BD is perpendicular to OE.
inscribed in a circle; AOX, BOY, COZ are
41. ABC is a

ABCD

40.

is

three chords intersecting at a |>oint

A ABC;

inside

prove

Z.YXZ= /BOC- Z.BAC.


/

42.

is any point on the side


taken on AC, BC so that

drawn through D,

is

AEFG
ABC

43.

P,

is

is

equilateral.
a line
;

Q, R;

of Z.8

if

BPR,

E,

F are

Z.

circle

prove

PQR

B, C, Q,

BAC

of A ABC
points E,
EDA = 60 = /. FOB a
and cuts AB again at G

AB

cuts

BC

produced, CA,

AB

at

are concyclic, prove the bisectors

are at right angles.

AB bisects ^PAQ and ^XAY; prove


PQ is parallel to XY.
the
ABC is a A the bisectors of /.s ABC, ACB meet at
circle BIC cuts AB, AC again at P, Q; prove AB = AQ
and AC^AP.
AB is a diameter of a circle AQBR; AQ, BR meet when

APXBYQ

44.

45.

is

a circle;

46.

produced

at

O;

use an

47.

BQ.AO-AR.BO.
ABC is a A; the bisectors
and cut AC, AB at Y, Z
;

at
48.

area

of /.s

formula

ABC,

to

ACB

the circles BIZ,

prove

that

intersect at

I,

CIY meet again

X; prove Z.YXZ+ Z.BIC =180.

a triangle inscribed in a circle; AB = AC; BC is


=
produced to D; AD cuts the circle at E; prove Z.ACE

ABC

is

Z.ADB.
49*.

are perpendicular chords of a circle ACBD ; prove


to AD bisects, when prothat the perpendicular from

AOB,

COD

duced, BC.
50*.

51*.

ABCD is a quadrilateral inscribed in a circle, centre O if


AC is perpendicular to BD, prove that the perpendicular
from O to AD equals JBC.
OC is a radius perpendicular to a diameter AOB of a circle;
from A, B to any
P, Q are the feet of the perpendiculars
=
that AP 2 + BQ* =
and
line through C; prove that PC
QB
;

2OC 2
Two given
.

52*.

circles

ABP,

ABQ

intersect at A,

a variable

CONCISE GEOMETRY

66
line

PAQ

meets them at P,

prove

21

PBQ

of constant

is

size.

53*.

ABC

a variable point on a given circle


which passes through B, C; if P, A are on the same side
of

is

a given

BC, prove

/.

is

PBA - L PCA

is

constant.

54*. In Fig. 81, the circles are given; prove

Z.PRQ

is

of constant

size.

FIG. 81.

55*.

56*.

AB is

a fixed chord, and AP a variable chord of a given circle ;


are the mid-points of AB, AP ;
prove Z. AQC has one

C,

of

two constant

values.

variable circle passes through a fixed point A and cuts


two given parallel lines at P,
such that /.PAQ = 90;

prove that the


57*.

Two

circles

circle

PSQ

PRQ

through a second fixed point.

circle passes

PRQ, PSQ
lies

cuts circle

on

intersect at P,

circle

PRQ

at

PSQ;
;

prove

Q;

the centre

the diameter

QR

is

PS

parallel to

of

of circle

OP,

ANGLE PROPERTIES OF A CIRCLE

(2)

ANGLE PROPERTIES OF A CIRCLE

(2)

67

THEOREM 40
If

any point on a circle, centre O,


= 90.
at P, then Z.

is

and

if

PX

is

the tangent

OPX

P,Q
FIG. 82.

THEOREM 41
If

PA

is

any chord

K and X

of a circle

PKA, and

if

PX

being on opposite sides of PA,


then Z.APX= L AKP.

FIG. 83.

is

the tangent at P,

CONCISE GEOMETRY

68

ANGLE PROPERTIES OP A CIRCLE

(2)

EXERCISE XIV

2.

TBC cuts a circle ABC at B, C; TA is a


L TAG = 118, Z.ATC = 26, find Z.ABC.
ABC is a minor arc of a circle the tangents at

3.

T; if Z.ATC = 54, find /.ABC.


AOC, BOD are chords of a circle ABCD

1.

line

tangent;

A,

if

meet

at

the tangent at A
DB produced at T if Z.ATD = 24, /.COD = 82,
Find also the angle between
Z. TBC = 146, find ZL BAG.
BD and the tangent at C.
The aides BC, CA, AB of a A touch a circle at X, Y, Z
= 64, Z. ACB = 52; find Z.XYZ, ,/XZY.
,/ ABC

meets

4.

5.

Three of the angles of a quadrilateral circumscribing a circle


are 70, 84, 96 in order; find the angles of the quadrilateral

6.

TBP,

whose

TCQ

Z.QCA =
7.

In

A ABC,

AC

vertices are the points of contact.


are tangents to the circle ABC; Z.PBA
128; find /.BAG and ZLBTC.

L ABC = 50,

produced,

AB

Z.

ACB = 70;

produced at X, Y,

a
;

T TC

8.

A chord AB of

9.

Z.TBC = /.ACT.
Two circles APB, AQB intersect at A, B
= Z.ABQ.
tangents at A, prove Z.ABP

a circle

is

produced to

circle

find Z.

is

= 146,

touches BC,

YXZ.

a tangent from

to the circle; prove

10.

DA

11.

AC, prove Z. ADB => Z. ABC.


In A ABC, AB = AC; D is the

is

the tangent at

to the circle

ABC

AQ

AP,

if

mid-point of

DB

is

BC;

are the

parallel to

prove that

ADC

is perpendicular to AB.
the tangent at D to the circle
of
the
chords
circle
the tangent
are parallel
12. BC,
;

at

13.

AD
A cuts CB

ABCD

produced at

Z.PAQ=Z.BPQ.
Two circles ACB, ADB
to circles

ADB,

ACB

PD

cuts the circle at

intersect at A,

at A,

prove

CA,

AD

is

DB

prove

are tangents

parallel to

BC.

are equal chords of a circle ; the tangent ADE at


meets BC produced at D ; prove Z. BDE = 3 Z. CAD.

14.

CA,

CB

15.

The

bisector of

Z.BAC meets BC

at

circle is

drawn

ANGLE PROPERTIES OF A CIRCLE


touching

BC

16.

Two

17.

ABCD
A, C

18.

circles

passing through

^PDB=

meet at

A inscribed in a

produced at

B; the tangents

A,

PBQ
if Z. PBQ is

is

if Z.

the tangents at

Z. ABC ~ L ADC.

What happens
is

AC

Z.QDC.

T prove Z. ATC =

circles intersect at

ABC

AB,

AQB intersect at A, B; AQ, AP are the


A if PBQ is a straight line, prove Z. PAQ = 90.

;
tangents at
is a quadrilateral inscribed in a circle

Two

cuts

if it

APB,

circles at P,

19.

D and

at

at P, Q, prove

69

(2)

acute, prove Z.

at

B meet

the

PAQ = 2 Z. PBQ.

obtuse ?

circle

the bisector of

the tangent at C meets AB


cuts AB at D ; prove

L ACB

TC = TD.
AOB is a diameter

of a circle, centre O; the tangent at B


meets any chord AP at T ; prove Z.ATB = Z.OPB.
21. ABCDE is a pentagon inscribed in a circle ; AT is the tangent
at A, T and D being on opposite sides of AB ; prove L BCD
20.

+ ZlAED=:lS0 -f ,/BAT.
A ABC, AB = AC; a circle

is drawn to touch BC at B and


A
it
to pass through
if
cuts AC at D, prove BC == BD.
=
D is any point on BC DP, DQ are
In AABC, Z. BAC 90
= 90.
tangents at D to the circles ABD, ACD prove /_ PDQ
AB is a diameter of a circle ABC TC is the tangent from a
point T on AB produced TD is drawn perpendicular to TA
and meets AC produced at D prove TC = TD.
EAF, CBD are tangents at the extremities of a chord AB of a
if AB bisects
circle, E and C being on the same side of AB
Z. CAD, prove Z. EAC = L ADC.
Two circles touch internally at A the tangent at any point P
on the inner cuts the outer at Q, R AQ, AR cut the inner
at H, K; prove AS PQH, APK are equiangular.
PQ is a common tangent to two circles CDP, CDQ; prove
that Z. PCQ + Z. PDQ = 180.
Two chords AOB, COD of a circle cut at O the tangents at
the tangents at B, D meet at Y
A, C meet at X
prove
Z.AXC+ ^BYD==2Z.AOD.

22. In

23.

24.

25.

26.

27.

28.

29.

is

AABC;

the centre of a circle touching the sides of


a
drawn ; prove that it cuts off equal

larger concentric circle is

portions from AB, BC,

CA.

CONCISE GEOMETRY

70
30.

PQ, PR are equal chords of a

circle
PQ and the tangent at R
T prove L PRQ = 60 \ L PTR.
The diameter AB of a circle, centre O, is produced to T so that
OB = BT TP is a tangent to the circle prove TP = PA.
The bisector of L BAG cuts BC at D a circle is drawn through
D and to touch AC at A prove that its centre lies on the

intersect at

31.

32.

perpendicular from

AB.

to

common point of interchords are equal ; prove that

33. Three circles, centres A, B, C, have a


section

34.

also their

common

O is the centre of the circle inscribed in AABC.


AB is a chord of a circle the tangents at A, B meet at T
AP is drawn perpendicular to AB, and TP is drawn perpendicular to TA prove that PT equals the radius.
Two circles ABD, ACE intersect at A BAC, DAE are straight
lines
prove that the angle between DB and CE equals the
;

35.

angle between the tangents at A.


36. Assuming the result of ex. 21 (page 63), what special cases can
be obtained by taking (i) Q very close to S, (ii) Q very close
to B,

(iii)

very close to

B?

are given points on a given circle ; P is a variable point


on the circle ; the circles whose diameters are AB and AP

37. A,

intersect at Q.

Find the position of

when P

is

very close

to B.

38.

OA is a chord of a circle, centre C T is a point on the tangent


TA is proat O such that OA = OT and L AOT is acute
Find
duced to cut OC at B prove that /. OBA = J / OCA,
;

the position of

B when A

is

very close to O.

PROPERTIES OF EQUAL ARCS AND CIRCLES

PROPERTIES OF EQUAL ARCS AND


EQUAL CIRCLES
THEOREM
H,

37.

are the centres of two equal circles APB, CQD.


= arc CD.
(i) IfZ.AHB= zlCKD, then arc AB
(ii)

If

/_APB=* Z.CQD, then

arc

THEOREM

H K
(

AB = arc CD.

38.

are the centres of two equal circles


AHB
If arc AB = arc CD, then (i)

APB, CQD.

= L CKD,
(ii)/.APB = Z.CQD.

and

THEOREM
APB,

CQD
(i)
(ii)

are

two equal

39.

circles.

AB = chord CD, then arc AB = arc CD.


AB = arc CD, then chord AB = chord CD.

If chord
If arc

These properties also hold for equal arcs in the same

circle.

71

CONCISE GEOMETRY

72

PROPERTIES OF EQUAL ARCS AND


EQUAL CIRCLES
EXERCISE XV
1.

ABCD

Ls

in the
2.

3.

circle

is a side of a regular hexagon and AC of a regular octagon


inscribed in the same circle ; calculate the angles of AABC.

AB

AC cuts BD at
DC when proif Z. AEB = 55, Z. BFC = 35, Z. DOC = 85,

ABCD is a quadrilateral inscribed in a


O DA, CB when produced meet at
:

duced meet at F

4.

AEF is an equilateral triangle inscribed


calculate the angles of AECD.

a square and

same

prove arc BC = twice arc AB.


ABC is a triangle inscribed in a circle

BC

produced at T; /.BAT
ratio of the arcs AB and AC.
5.

CD;

calculate/.

AB,

A meets

the tangent at

ABCD

arc
6.

^135,ZATB-

are two points on the circle


arc AB is half the major arc AB

A,

circle

30

such that the minor

DAB = 74

Z.

find the

arc

BC =

A BD andZ_BDC.

ABCD is a quadrilateral inscribed in a circle; Z.ADB = 25,


Z. DBC = 65
prove arc AB + arc CD = arc BC + arc AD.
;

8.

CD are parallel chords of a


ABCD is a cyclic quadrilateral

9.

7.

AB,

</

circle
;

if

prove arc

AB = CD,

AD

prove

arc
Z.

BC.

ABC =

BCD.

AOBP passes through the centre O of a circle ABQ


OP bisects Z.APB.
are
two equal circles; PBQ is a straight line;
ABP, ABQ
=
AP
AQ.
prove
AB, BC, CD are equal chords of a circle, centre O prove that
AC cuts BD at an angle equal to Z. AOB.
ABCD is a square and APQ an equilateral triangle inscribed
circle

prove that

10.

11.

12.

in the

13.

same

circle,

BP = arcPC.
On a clock-face,

P being between B and C; prove

prove that the line joining


perpendicular to the line joining 5 and 12.

4 and 7

arc

is

PROPERTIES OF EQUAL ARCS AND CIRCLES


14.

X, Y are the mid-points of the arcs AB,


cuts AB, AC at H, K ; prove AH = AK.

15.

APB,

AQB

16.

ABCD

18.

ABC

are

is

DC

prove

a tangent to the

DP

is

a chord

PB = AD.

inscribed in a circle; if two pairs of opposite


sides are parallel, prove that the third pair is also parallel.
is

hexagon

A inscribed

is

through B, C
are produced to meet

in a circle;

circle

any

cuts AB, AC again at P, Q ; BQ, CP


tho circle ABC at R, S ; prove AR AS.
19. ABCDEF is a hexagon inscribed in a circle;

"20.

XY

of a circle;

a rectangle inscribed in a circle;

is

equal to
17.

AP

two equal circles;


AQB, prove AB = BP.

circle

AC

73

Z.DEF, prove AF is parallel to CD.


is a quadrant of the circle ACDB
=
cuts BC at P
prove AC CP.

CD

AB

/.ABC =

if

a diameter

is

AD

P is any point
is a
inscribed in a circle, centre O
on the side BC ; prove that the circles OBP, OCP are equal.
22. In AABC, AB = AC; BC is produced to D; prove that the
21.

ABC

23.

ABCD

ABD,

circles

ACD

are equal.

a quadrilateral inscribed in a

is

Z.ABC

to F; the bisector of

that
21.

DE

ABCD

is

bisects /.ADF.
a cyclic quadrilateral

meet at E

a circle

is

drawn

circle

CD

is

cuts the circle at

BC

and

produced

E; prove

AD

are produced to
through A, C, E and cuts AB,
;

CD again at P, Q prove PE = EQ.


AB, AC are equal chords of a circle BC
that CD = CA; DA cuts the circle at
;

25.

/.ABC.
an equilateral

is

produced to

D so
BE

prove that

E;

bisects

26.

ABC

is

the mid-points

of

triangle inscribed in a circle ; H,


arcs AB, AC; prove that

the

by AB, AC.
are two chords of a

are

HK

is

trisected

27.

BC
AB is

AB,

prove
28.

ABCD

circle

(AB>BC)
AB and

folded over about the chord

BD
is

the minor arc


cuts

AC

at

BC.

a quadrilateral inscribed in a

circle

the mid-points of the arcs AB, BC, CD,


29.

XZ is perpendicular to YW.
In AABC, AB>AC; the bisectors

of

X, Y, Z,

DA;

W are

prove that

^ ABC, ACB

meet

74

CONCISE GEOMETRY
at

the circle

BIG

cuts

AC

AB,

at P,

QI = IB.
30.

ABC

prove PI

=r

1C and

a triangle inscribed in a circle, centre O ; PQ is the


diameter perpendicular to BC, P and A being on the same
is

side of

BC

is

ABC ~ L ACB = /. POA.


circles are equal and AD = BC;

prove

31*. In Fig. 85, the

/.

prove

XBYD

a parallelogram.

FIG. 85.

AABC, AB = AC

any point on BC; X, Y are the


centres of the circles ABD, ACD; XP, YQ are the per= YQ.
pendiculars to AB, AC; prove XP
33*. AB, CD are two perpendicular chords of a circle, centre O ;
2
2
2
prove that AC + BD = 4OA
[Use Theorem 25(2).]
A 2rt is a regular polygon of 2n sides ; if 2i&>
34*. A 1 A 2 A3
32*. In

is

p>q>r>8, prove that Ap A r is perpendicular to A g A 5 if


p + r = q + s + n.
ABC is an equilateral triangle inscribed in a circle D, E are
points on the arcs AB, BC such that AD = BE, prove AD +
DB = AE.
C is the mid-point of a chord AB of a circle D, E are points
on the circle on opposite sides of AB such that L DAC =
Z.AEC; prove that Z.ADC= Z.EAC.
P, Q, R are points on the sides BC, CA, AB of AABC such
that ^PQR=ZABC and Z.PRQ=Z.ACB; prove that
the circles AQR, BRP, CPQ meet at a common point, K say,
and are equal; prove also that (i) /.BKC = 2z.BAC; (ii)
AK=*BK = CK; (iii) PK is perpendicular to QR.
f

35*.

36*.

37*.

38*.

Two

fixed circles cut at A,

is

a variable point on one

PROPERTIES OF EQUAL ARCS AND CIRCLES


PA,

PB when

produced cut the other at

QR

prove

75

QR

is

of constant length.

39*.

a fixed point on a fixed circle ; B is a fixed point on a


fixed line BC ; a variable circle through A, B cuts BC at P
is

and the

fixed

circle at

a fixed point.

circle at

Q;

prove that

PQ

cuis the fixed

CONCISE GEOMETEY

76

L-ENGTHS OF TANGENTS AND CONTACT


OF CHICLES
THEOREM 42
If

TP,

TQ

T to a circle,
TP - TQ.
Z.TOP = TOQ.

are the tangents from

then

(i)
(ii)

(in)

OT

bisects Z.

centre O,

PTQ.

FIG. 86.

THEOREM 43
If

two

circles, centres A, B,

then

APB

is

a straight

touch, internally or externally, at P,


line.

Y
FIG. 87(1)

FIG. 87(2).

If the circles touch externally (Fig. 87(1)), the distance


= sum of radii.
the centres

between

If the circles touch internally (Fig. 87(2)), the distance


the centres AB = difference of radii.

between

AB

77

LENGTHS OF TANGENTS

LENGTHS OP TANGENTS AND CONTACT


Otf

CIRCLES

EXEBCISE XVI
1.

radius 5 cms., touches two concentric circles and


encloses the smaller: the radius of the larger circle is
circle,

7 cms.

what

is

the radius of the smaller

centres A, B, C, touch each other externally;

2.

Three

3.

AB = 4", BC - 6", CA - V find their radii.}


In AABC, AB = 4", BC = 7", CA = 5* two circles

circles,

with B,

with

C
as

as centres touch each other externally ; a circle


centre touches the others internally ; find their radii.
4. Fig. 88 is formed of three circular arcs of radii 6*7 cms.,

2-2 cms., 3*1 cms.; X, Y, Z are the centres of the circles;


find the lengths of the sides of

FIG. 88.
5.

In Fig. 89,
quadrant

AB is a
BC at B

quadrant touching AD at A and the


L ADC - 90, AD - 12", DC = 9" ; find

the radii of the circles.

FIG. 89.
6.

The distance between the centres of two circles of radii 4 cms.,


7 cms. is 15 cms.; what is the radius of the least circle
that can be drawn to touch them and enclose the smaller
circle

CONCISE GEOMETRY

78
7.

= 5", CB
is a point on AB such that
radius of the circle which touches AB at

AC

the circle on
8.

A,

AB

= 3*

calculate the

and

also touches

as diameter.

aje the centres of two circles of radii 5 cms., 3 cms.

AB

BC

a radius perpendicular to BA; find


the radius of a circle which touches the larger circle and

12 cms.;

is

touches the smaller circle at C.


9.

BC

[Two

answers.]

two equal quadrants touching at B ; their radii


are 12 cms.; find the radius of the circle which touches
arc AB, arc BC, AC.

AB,

are

FIG. 90.

10.

A ABC, AB =

= 6", CA = 7"; a circle touches BC,


4", 80
find
BX and AY.
Z
AB
at
CA,
X, Y,
In AABC, AB = 3", BC = 7*, CA = 9*; a circle touches CA
produced, CB produced, AB at Q, P, R; find AQ, BR.
In

11.

12.

Two circles of

13.

The

14.

The distance between the centres of two circles is 10 cms.,


and the lengths of their exterior and interior common tangents

15.

ABCD

16.

In one corner of a square frame, side 3', is placed a disc of


radius 1' touching both sides ; find the radius of the largest

radii 3 cms.,

12 cms. touch each other externally ;

common tangent.
distance between the centres of two circles of radii 11 cms.,
5 cms. is 20 cms. ; find the lengths of their exterior and

find the length of their

interior

common

tangents.

are 8 cms., 6 cms. ; find their radii.


is a square of side 7*; C is the centre of a circle of
radius 3* ; find the radius of the circle which touches this
circle

disc

and touches

which

will

fit

AB

at A.

into the opposite corner.

17. a, b are the lengths of the diameters of

two

circles

which

touch each other externally ; t is the length of their common


2
tangent ; prove that t = ab.
18. Two circles of radii 4 cms., 9 cms. touch each other externally ;

LENGTHS OF TANGENTS
find the radius of the circle

and

circles

also

thejr

79

which touches each of these

common

[Two answers

tangent.

use ex. 17,]


19.

OA = a*, OB = ^, Z_AOB = 90;

two variable circles are


drawn touching each other externally, one of them touches
OA at A, and the other touches OB at B if their radii are
af y", prove that (x -f a) (y f b) is constant. If a = 8, b = 6,
x - 4, calculate y.
;

20.

Four equal

spheres, each of radius

1*,

are fixed in contact

with each other on a horizontal table, with their centres


at the corners of a square; a fifth equal sphere rests on

them ;

21.

find the height of its centre

touches the sides of

circle

the mid-points of AB,


of
22.

Two

AC,

above the

table,

A ABC at

X, Y, Z; if Y, Z are
that
X is the mid-point
prove

BC.
circles

A ; any line through


prove that the tangents at P,

touch each other at

the circles at P,

Q;

A cuts
Q are

parallel.

23.

ABCD

is

a quadrilateral circumscribing a

circle,

prove that

AB-f CD = BC + AD.
24.

ABCD

25.

Two circles touch externally at A PQ is their common tangent


prove that the tangent at A bisects PQ and that Z. PAQ = 90.
In Fig. 91, prove AB - CD = BC - AD.

26.

is a parallelogram; if the circles on AB and CD as


diameters touch each other, prove that ABCD is a rhombus.
;

CONCISE GEOMETRY

80
27.

28.

ABCDEF is a hexagon circumscribing a circle; prove that


AB + CD + EF - BC + DE + FA.
O is the mid-point of BC circles
In A ABC, L BAC = 90
with
AB
and
AC
as diameters prove that two
are drawn
;

can be drawn with

circles

as centre to touch each of

these circles.
29.

Two circles touch externally at A


BP is a tangent to the other

AB

is

a diameter of one

prove that

/.

APB = 45

IZ.ABP.
30.

A BCD

is

31.

Two

a quadrilateral circumscribing a

Z.AOB+

prove

Two

circles

one

at

circle,

A
=

a chord

of one touches

PQ

Z. QAR.
provo L PAR
touch internally at A
any line

PQRS cuts
prove /_ PAQ =

S and

P,

centre

COD -180.

touch internally at

circles

the other at
32.

Z.

the

other at

Q,

Z.RAS.
33.

Two

equal

is parallel

34.

Two

circles

on each

Two

circles

/_

aro points,
prove that PQ

P,

XY.

P, Q are points, oue


touching internally at A
that
such
Z PAQ ^=90; prove that the
circle,
;

tangents at
35.

such that

circle

to

A
=
90
PAQ

centres X, Y, touch at

circles,

one on each

P and

touch at

are parallel.

any

line

PAQ

cuts one circle at P,

prove that the tangent at P is


perpendicular to the diameter through Q.
36. In
ABC, Z. ABC -90; a circle, centre X, is drawn to

and the other

at

AB produced,
Z.AXC-45

touch

AC

produced,

and

BC;

prove

37.

Two
at

touch externally at A ; a tangent to one of them


cuts the other circle at Q, R; prove Z.PAQ+ Z.PAR

circles

= 180.
38.

Two

circles, centres A, B, touch externally at P ; a third circle,


centre C, encloses both, touching the first at
and the
second at R ; prove /. BAC = 2 /. PRQ.

39.

A circle,
at X,

centre A, touches externally two circles, centres B,


cuts the circle, centre C, at Z ; prove BX
;

XY

CZ.
are two

C
is

parallel to

40. PR,

QR

circular arcs touching each other at R,

and

LENGTHS OF TANGENTS

OQ

touching the unequal lines OP,


Z.

PRQ = 180 - \ L POQ

81

at P,

prove that

Q;

(see Fig. 92).

FIG. 02.

41*.

PQ
42*.

is

RST;

if

centre A, passes through the centre B of a


RP, SQ are common tangents, prove that

touches the circle RST.


the centre of a fixed circle

touch the fixed

Q,

P,

43*.

PBQ,

circle

circle

two variable circles, centres


internally and each other

externally; prove that the perimeter of


given circles touch internally at

AOPQ
A

Two

through

cuts the circles at P,

44*. OA,

OB

circle

touches OA,

radius

= JOA.

is

is

constant.

a variable

line

prove that the

j)er-

passes through a fixed point.


are two radii of a circle, such that /. AOB = 60

pendicular bisector of

45*.

circle

PQ

OB

and the

arc

AB

prove that

its

the mid-point of AB ; semicircles are drawn with AC,


as diameters and on the same side of AB ; a circle

CB, AB
is

drawn

radius
46*.

to touch

the three semicircles; prove that

its

= JAC.

square

ABOD

PQRS
AB = 5PQ.
is

is

inscribed in a circle,

and another square

inscribed in the minor segment

AB

prove that

CONCISE GEOMETRY

82

CONVERSE PROPERTIES
THEOREM 36
(i)

(ii)

Z.APB= ZAQB and

if

P,

the four points A, B, P,

lie

on a

If

If

of
(iii)

If

are on the

same

side of

AB,

circle,

180 and if P, Q are on opposite sides


Z
the
four
AB,
points A, B, P, Q lie on a circle,
-=
P lies on the circle whose diameter is AB.
then
APB
.
90,

/.APB +

AQB-

CONVERSE OF THEOREM 41
If

and
/_

T are points on opposite sides of a line AB and such that


BAT = ACB, then AT is a tangent to the circle which

passes through A, C, B.

FIG. 95

ler at

83

CONVERSE PROPERTIES

CONVERSE PROPERTIES
EXERCISE XVII
1.

a parallelogram; if /.ABC ^60, prove that the


ABCD
of
centre
the circle ABD lies on the ciicle CBD.
is

2.

BE,

3.

The

CF

are altitudes of

altitudes

BE

AD,

A ABC; prove that


AABC intersect

of

^AEF-= ZABC.
H prove that

at

Z_DHC= ZDEC.
4.

ABCD is a parallelogram any circle through A, D cuts AB,


DC at P, Q prove that B, C, Q, P are concyclic.
ABC is a A inscribed in a circle BE, CF are altitudes of
:

5.

AABC
6.

The

prove that

BCGF

circle

lies

EF

parallel to the tangent at A.

is

ADHE; OABCD

inside the circle

and

OEFGH

7.

are two lines cutting them ; if A, B, F, E are concyclic, prove that C, D, H, G are concyclic.
cuts BD at
is a parallelogram ;
that the
; prove

AC

ABCD

circles
8.

AOB, COD

AD

line

is

touch each other.

trisected at B,

C BPC
;

is

an equilateral triangle

prove that AP touches the circle PBD.


9 AB is a diameter, AP and AQ are two chords of a circle

AQ
10.

cut the tangent at

in X,

are concyclic.
is a
inscribed in a circle

ABC
BC

at X,

and

the tangent at

any
at

AP,

prove that P, X, Y,

lino parallel to

AC

prove B, X, A,

cuts

are

concyclic.
11.

In Fig. 96,
prove that

BQP

QA

and
is

BAC

are equiangular isosceles triangles;

BC.

parallel to

12.

ABCD

is

a parallelogram

and cutting AB,


concyclic.

AC

circle is

at P,

Q;

drawn touching

AD

prove that P, Q, C,

at

Bare

CONCISE GEOMETRY

84
13.

ABCD is a rectangle; the line through C perpendicular to


AC cuts AB, AD produced at P, Q prove that P, D, B, Q
;

are concyclic.
14.

A ABC,

In

CA, BA

L BAG = 90

produced at P,

the perpendicular bisector of BC cuts


Q ; prove that BC touches the circle

CPQ.
15.

16.

ABCDE is a regular pentagon BD cuts CE at O prove that


BC touches the circle BOE.
OY is the bisector of ZLXOZ; P is any point; PX, PY, PZ
are the perpendiculars to OX, OY, OZ
prove that
XY-YZ.
AA BB 1 CC 1 are equal arcs of a circle AB cuts A 1 B 1 at P
AC cuts A C1 at Q prove that A, A 1 P, Q are concyclic.
CA, CB are two fixed radii of a circle P is a variable point
;

17.

18.

on the circumference

P
19.

to

ABC

is

cuts

20.

CA,
a

BC

CB

PQ, PR

prove that

inscribed

at P,

QR

are the perpendiculars from


of constant length.

is

in a circle;

and the tangent at

a line parallel to AC
A at T ; prove that

A ABC X is a variable
cut AB, AC at Z, Y
CXO
BXO,

a fixed point inside a given

is

point on

BC

the circles

prove that (1) O, Y, A, Z are concyclic, (2) the angles of


are of constant size.

AXYZ

21.

Four

circular coins of

unequal

sizes lie

on a table so that each

touches two, and only two, of the others; prove that the
four points of contact are concyclic.
22.

ABC, ABD

are two equal circles

touches the circle


23.

AB,

CD

are

parallel to

of

26.

prove that

AC

to

CQ

are

prove that

PQ

of a circle

CD, AB

AP,

formed by the external bisectors

is cyclic.

any quadrilateral
BD are two perpendicular chords of a circle ; prove that
the tangents at A, B, C, D form a cyclic quadrilateral.
AB, AC are two equal chords of a circle AP, AQ are two

AC,

chords cutting
27.

AB = BC,

BD.

24. Prove that the quadrilateral

25.

if

two intersecting chords

the perpendiculars from A,


is

ABD.

The diagonals

BC

at X,

of a cyclic

prove P, Q, X,

quadrilateral

ABCD

are concyclic.
intersect at

CONVEKSE PBOPERTIES

right angles at
from
to AB,

28.

O
COD
AOB,

29.

ABC

is

DP

is

drawn

prove that the feet of the perpendiculars

DA

BC, CD,

are concyclic.

are two perpendicular chords of a circle ; DE is


other
chord ; AF is the perpendicular from A to DE ;
any
prove that OF is parallel to BE.

CF

30. BE,

A inscribed in a

AD is an altitude of AABC ;
;
and meets the tangent at A at P ;

circle

AB

parallel to

Z.CPA = 90.

prove

AABC X

is

tho mid-point of

BC

AABC; X is
L BAG.

the mid-point of

BC;

are altitudes of

prove that XE-XF.


31. BE, CF are altitudes of
prove that
32.

85

Z.

FXE -

180 - 2

circles APRB, ASQB intersect at A, B ; PAQ and RAS


are straight lines ; RP and QS are produced to meet at
;
are concyclic.
prove that O, P, B,

Two

33.

COD

AOB,

are

chords CP,

two perpendicular diameters of a

CQ

AB

cut

at H,

circle

two

prove that H, K, Q,

are

concyclic.

34.

The

CD of
on CD

side

point

bisector of /.

35*.

the square ABCD is produced to


the line from P perpendicular to

ADE

at

prove

36*. X,

are the centres of the circles

straight line

PX and QY are

that X, Y, B, R are concyclic.


CF are altitudes of AABC;

prove that
38*.

PQ
is

is

Z.ZEF= /. ABC

a chord of a

circle

any point such that

at E,

prove that

any

cuts the

is

CA,

DB

D meet

at

ABQ PAQ
;

meet at R

are

T
is

prove

the mid-point of

AB

/. BAC.

the tangents at P, Q meet at T ; R


;
RP, RQ cut the circle again
is a diameter.

TR -= TP

EF

CD are parallel chords of a circle the tangent at D cuts


PQ at T B is the point of contact of the other tangent from
T prove that BC bisects PQ.
ABCD is a parallelogram O is a point inside ABCD
such that L AOB + Z. COD = 180; prove that Z.OBC =

39*. PQ,

40*.

ABP,

produced to

37*. BE,

PB

is

BP= PQ.

CD

are parallel chords of a circle, centre


produced to meet at P ; the tangents at A,
prove that A, D, P, O, T are concyclic.

AB,

E; P

Z.ODC.

CONCISE OKOMETRY

86

MENSURATION
1.

For a
(i)
(li)

circle of radius

r inches,

the length of the circumference^- 2?rr in.


~TTI* sq.
the area of the cucle

in.

(in) the length of an arc, which subtends 6 at tho centre of


t\

the chcle,

o,.::

^irr in.

A
(iv)

tho area of a sector of a circle of angle

-n-r

aq. in.
2.

For a sphere of radius r inches,


(i)

(ii)

the area of surface of sphere = 47rr 2 sq. in.


tlie volume of the sphere
\Trr* cub. in.

(in) the area of the surface intercepted

'A

(i)

4.

tho area of the curved surface

2?rrA

s<j.

in.

irr^h cub. in.


the volume of the cylinder
(ii)
For a circular ume, radius of base r inches, height h inches,

length of slant edge


(i)
(ii)

5.

between two parallel

%-irrd s<{ in
planes at distance d inches apart
For a circular cylinder, radius r inches, height h inches,

inches,

P~*a + /A
area of the curved surface

= ?rrZ

sq. in.

= ^7rr% cub. in.


(Hi) volume of cone
The
of
volume
any cylinder = area of base x height.
(i)
of
The
volume
any pyramid = \ area of base x height.
(ii)

w= 2 2
7

approx. or 3*1416 approx.

87

MENSURATION

MENSURATION
EXERCISE XVIII
of radius
(1) the circumference, (2) the area of a circle

1.

Find

2.

The circumference

(i)

correct to
3.

The area

An

what

if

its

radius

what

is its

radius correct to

arc of a circle of radius 3 inches subtends

at the centre
5.

of a circle is 5 inches

inch

of a circle is 4 sq. cms.

iV cm.
4.

100 yards.

4", (ii)

what

is its

The angle of a sector of a


what is its area ?
cms.

an angle of 40

T\> inch 1
its radius
and
108,

length correct to
circle is

is

2*5

6.

7.

what
AB is an arc

8.

square

ABCD

is

inscribed in a circle of radius 4 inches

is the area of the

5 cms.

of a circle, centre

find /.

of the circle

AOB,

by AB.
AO = 5 cms. and arc

off

AB =

correct to nearest minute.

piece of flexible wire is in the form of an arc of a circle of


radius 4 '8 cms. and subtends an angle of 240 at the centre

radius
9.

minor segment cut

horse

it is

bent into a complete

circle

what

is

the

is

tethered by a rope 5 yards long to a ring which can


low straight rail 8 yards long ; what is the

slide along a

10.

area over which the horse can graze 1


OA, OB are two radii of a circle prove that the area of sector
;

AOB
11.

What

x arc AB.

equals |OA
the area contained between two concentric circles of

is

radii 6 inches, 3 inches

C
FIG. 97.

12.

DA

are quadrants of equal circles of


Find the area of the
radii B cms., touching each other.

In Fig. 97, AB, BC, CD,


figure.

CONCISE GEOMETRY

88
13.

Find

the volume,

(i)

the total surface of a closed cylinder,

(ii)

height 8*, radius 5".


14.

1 Ib.

of tobacco

and height

is

8"

packed in a cylindrical tin of diameter 4*


what would be the height of a tin of

diameter 3" which would hold

packed
15.

Ib.

of tobacco, similarly

How many

cylindrical glasses 1" in diameter can be filled to

a depth of 3" from a cylindrical jug of diameter 5" and


height 12"!
16.

Find

the volume,

(i)

(ii)

the area of the curved surface of a

circular cone, radius of base 5*, height 12".


17.

of

sector

of

circle

radius

cms.

bent to form the surface of a cone;

and angle 60
find the

is

radius of

its base.

18.

The curved surface


4*

folded out

is

obtained
19.

Find

of a circular cone, height 3*, radius of base


What is the angle of the sector so
flat.

the volume,
pyramid, whose base
(i)

(ii)

the total area of the surface of a

is

a square of side 6* and whose height

(ii)

the area of the surface of a sphere of

184".
20.

Find

21.

Taking the radius of the earth as 4000


area between latitudes 30 N and 30 S.

the volume,
(i)
diameter 5 cms.

is

this

earth
22.

Two

area of

the

area of

the

total

find

miles,

What

the

fraction

surface

of

the

cylinders, diameters 8*

and

6", are filled

with water to

5" respectively: they are connected at the


depths
bottom by a tube with a tap when the tap is turned on,
what is the resulting depth in each cylinder 1
10*,

23. Three draughts,

and an

elastic

1"
band

in diameter, are placed flat


is

put round them.

Find

on a table

its

stretched

length.

24.

What

is

the length of a belt which'passes round two wheels of

89

MENSURATION

diameters 2", 4", so that the two straight portions cross at


right angles 1 (see Fig. 98).

FIG. 93.

25.

26.

Find the volume of the greatest circular cylinder that can be


cut from a rectangular block whose edges are 4", 5", 6".
Fig. 99 (not drawn to scale) is a street plan, in which EF

27.

circular metal disc, 9" in diameter, weighs 6 Ib. ; what is


the weight of a disc of the same metal, 6" in diameter and
of the same thickness 1

a quadrant and the angles at A, H, D, E, F are 90;


yards; HD = 300 yards; CH = 150
distances of A from D by the routes
two
the
Find
yards.

is

AE = AB = DF = 100
(i)

AEFD, (ii)-ABCD.

Find

also the area in acres of the plot

28.

ABCOFE.

CA

are three circular arcs, each of radius 6 cms. and


(see Fig. 100)
touching each other at A, B,
the area of the figure,
(i) Calculate

AB, BC,

(ii)

Find

its

perimeter.

FIG. iOO.

CONCISE GEOMETRY

90
29.

a circle of radius 5 cms. and place in it a chord AB of


length 4 cms. ; find the area of the major segment AB,

Draw

making any measurements you


30.

like.

rectangular lawn 15 yards by 10 yards

is

surrounded by

man

can, without stepping off the lawn,


water the ground within a distance of 5 feet from the
What is the total area of the beds he can so
edge.

flower-beds

water

What would be

the area within his reach,

in the shape of (i) a scalene triangle,


of perimeter 50 yards 1

31.

ABC

if

the lawn was

any convex polygon,

a right-angled triangle circles are drawn with AB,


as diameters ; prove that the area of the largest is

is

CA

BC,

equal to the
32. Fig.

(ii)

sum

of the areas of the other

bisects
(i)
(ii)

two

circles.

AC = DB

and
represents
at right angles.
Prove that
Curves AXB, AVB are of equal lengths ;
Area of figure area of circle on
as diameter.

101

four semicircles;

XOV

AB

XV

BSARC

33. In Fig. 102, BQA, ARC,


the sum of the areas of

the area of

A ABC.

are semicircles, prove that

the lunes

BSAQ, CRAP

equals

MENSURATION
34. In Fig. 103, AB =
touch the sides of

BC = CA = 2

AABC

91
and the

cms.,

circular arcs

find the area of the figure.

FIG. 103.

35*.

A hoop,

of radius

plane, with

high.

its

rests in

2',

a vertical position on a horizontal


A with a thin vertical peg,

rim in contact at

The hoop

is

rolled over the

sponding position on the other side


area thus swept out.

>f Calculate

peg into the correFig. 104 shows the

this area.

FIG. 104.

36*.

triangular piece of cardboard

ABC

AC = 6*, /.BAG = 90. It is placed


edge BC against the wall and a pin is

is

such that

on the

floor

BA = 8*,
with the

put through the mid-

The cardboard is now turned about C till CA


point of BC.
is against the wall, then about A till AB is against the wall,
then about B till BC is against the wall; the cardboard
Construct
remains in contact with the floor throughout.
the curve which the pin scratches on the floor and find the
area between this curve and the wall.
37*.

ABCD cut at
= 4". The
right angles atO; AC -6", OC-OD-2*, OB
triangle DOC is cut away and the triangles AOD, BOC are

The diagonals AC,

BD

of the quadrilateral

CONCISE GEOMETRY

92

folded through 90
a tetrahedron on

about OA,

AOAB

Find

(ii)

so as to form

two

faces of

as base.

the volume of the tetrahedron

(i)

(iii)

OB

the area of the remaining face ;


the length of the perpendicular from

to

the

opposite face.
38*.

ABCD

is

a rectangle;

DWA'are

AB

--=

10",

AD = 6" AXB, BYC, CZD,


;

isosceles triangles, all the equal sides of

;
they are folded so as to form a pyramid with
base and X, Y, Z,
at the vertex.

9"

which are

ABCD

as

Find

(i)
(ii)

the height of the pyramid ;


the volume of the pyramid ;

the total area of the surface of the pyramid.


of the
AB=y, AD = /, AX ^r", and if the
(iii)

If

2
2
pyramid = A*, prove that A = r

Jp

height^

\q

2
.

93

LOCI

LOCI
THEOREM 45
A,

are

two

fixed points;

PA = PB,

if

a variable point

P moves

then the locus of (or path trace^

out,

by)

so that

is

the

perpendicular bisector of AB.

THEOREM 46
AOB,

COD

if a variable point P
;
perpendicular distances PH, PK from
these lines are equal, then the locus of (or path traced out by)
P is the pair of lines which bisect the angles between AOB

are

two

fixed intersecting lines

moves so that

its

and COD.

FIG. 106.

DEFINITION.

PX

Given a point P and a

from P to

AB

pendicular
then P1 is called the image

line

AB,

1
produced to P so that
in
or reflection of P
AB,
is

if

the per-

PX = XP1

CONCISE GEOMETRY

94

LOCI
EXERCISE XIX
1.

variable point
is its locus

is

variable point
is its locus

is

what
2.

what
3.

from a given

at a given distance

line,

at a given distance from a given point,

variable circle touches a fixed line at a fixed point,


its centre 1

what

is

the locus of
4.

variable circle passes through


its centre ?

two

fixed points,

what

tue

locus of
5.

variable circle touches

centre

its
6.

two

fixed lines,

what

is

the locus of

variable circle of given radius passes through a fixed point,


is the locus of its centre ?

what
7.

variable circle of given radius touches a fixed circle,


its centre ?

what

is

two

fixed concentric circles,

what

is

variable circle of given radius touches a given line,


the locus of its centre ?

what

is

the locus of
8.

variable circle touches

the locus of
9.

10.

PQR

is

its

centre

a variable triangle; /.QPR*=90,

PQ

and PR pass

through fixed points ; what is the locus of P ?


11. A, B are fixed points ; APB is a triangle of given area
is

12.

the locus of

what

P?

Given the base and

vertical angle of a triangle, find the locus

of its vertex.
13.

variable chord of a fixed circle

the locus of
14.

its

mid-point
a fixed point on a fixed

is

circle

find the locus of the mid-point of


15.

is

is

of given length,

what

is

AP
A P.
;

is

a variable chord

a variable point on a given line ; O is a fixed point outfind the locus of the mid-point of OP.
;

side the line

are fixed points


PAQB is a variable parallelogram of
given area ; find the locus of P.

16. A,

17.

ABC

is

a given triangle

BAPQ, CBQR

are variable parallelo-

95

LOCI
grams

P moves on a

if

fixed

circle,

the

centre A, find

locus of R.
18.

variable chord

of a given circle passes through a fixed

PQ

point ; find the locus of the mid-point of PQ.


19. The extremities of a line of given length move along
iixod perpendicular lines

20. A,

21.

OR

OA OPQ
/. OPQ

are fixed points ; P is a variable point on


triangle such that OP -f PQ is constant and

constant

PQR

if

are variable arcs of given lengths of a fixed circle,


meets
at S ; find the locus of S.

O PQ

point.

ABPQ

AP
PQ, QR

22. O,

two

are fixed points ;


is a variable parallelogram
is of given
length, find the locus of Q.

centre

23.

mid

find the lorus of its

prove that the locus of

and

QR

is

a straight

is
is

line.

the mid-points of PQ and PR are


passes through a fixed point; find the locus

a variable triangle

is

fixed

of P.

24. A,

are

fixed

bisector of

points

AB; AP

the locus of Q.
25. A, B are fixed points

P moves along the perpendicular

is

produced to

is

so that

AP= PQ

find

AP 2 4-

a variable point such that

PB 2 is constant find the locus of P.


P is a variable point such that PA 2 A, B are fixed points
2
PB is constant prove that the locus of P is a straight line
;

26.

27.

perpendicular to AB.
AC arc two fixed lines;
Z. BAC such that the sum of

AB,

its

a variable point inside


distances from AB and AC

is

;
prove that the locus of P is a straight line.
are fixed points ; P is a variable point such that
the sum of the areas^f the triangles PAB, PCD is constant ;
prove that the locus of P is a straight line.

is

constant

28. A, B, C,

29.

If

P1

30.

variable line

the image of

is

OQ

in

find

the

AP = AP X
A
point O

AB, prove that

passes through a fixed

another fixed point;


31. A,

in the line

locus of the image of

is

OQ.

are

two points on the same

image of
(i)
(ii)

AO
if

side of a line

CD A 1
;

is

CD A*B cut CD at O prove that


and OB make equal angles with CD

in

is

any other point on CD,

AP + PB > AO -f OB.

the

CONCISE GEOMETRY

96
32.

BK are the perpendiculars from A, B to XY. AH =


BK = 7", HK = 16" what is the least value of AP + PB 1

AH,

"

FIG. 107.

33. A,

CD

are fixed points on opposite sides of a fixed line


;
for which PA~~PB has its greatest
P on

CD

find the point

value.

How many images

are formed when a candle is placed between


two plane mirrors inclined to each other at an angle of
(i)90; (ii)60?
35. If a billiard ball at A moves so as to hit a perfectly elastic
cushion XY at P, it will continue in the line A*PB where
A1 is the image of A in XY; or, in other words, the two
portions of its path AP and PB make equal angles with XY.
ABCD is a rectangular billiard table with perfectly elastic
cushions a ball is at any point P ; it is struck in a direction
34.

parallel to

AC

will again pass

prove that after hitting

through

P.

all

four cushions

it

THE TRIANGLEOONOURBENOY PROPERTIES

97

THR TRIANGLECONCURRENCY PROPERTIES


THEOREM 47
If

ABC

a triangle, the perpendicular bisectors of BC, CA,

is

meet at a point

AB

(say).

Fio. 108.

is

is

called the circumcentre.

the centre of the circumcircle of the triangle

ABC, and

THEOREM 48
If A.BO is

BCA,

a triangle, the internal bisectors of the angles ABC,


CAB meet at a point (say).
I

PC

FIG. 109.

is

(i.e.

the centre of the circle inscribed in the triangle ABC


the in-circle of AABC), and is called the in-centre.

The external

AB

produced,

the angles ABC, ACB meet at


the centre of the circle which touches

bisectors of

a point Ip which

is

AC

produced,

be escribed to BC, and


7

is

BC;

called

an

this

circle

ex-centre.

is

aaid

to

CONCISE GEOMETRY

98

THEOREM 49
If

ABC
H

is

triangle, the altitudes

AD, BE,

CF

meet at a point

(say).

H is called the orthocentre of the


DEF is called the pedal triangle

triangle
of

ABC.

The

triangle

AABC.

THEOREM 50
If

ABC
G

is

a triangle, the medians AD, BE,

(say),

and

CF meet

DG- |DA.
A

BOO
FIG. 111.

is

called the centro^l of the triangle

ABC.

at a point

CONCURRENCY PROPERTIES

THE TRIANGLE

99

THE TRIANGLE CONCURRENCY PROPERTIES


EXERCISE XX

THE CIRCUMCIRCLK
1.

If

of
2.

the circumcentre of

is

BC, prove

Z.

A ABC and

of the quadrilateral
are the circumcentres of

P, Q, R,

if

is

the mid-point

BAG.

The diagonals

ABCD intersect at O;
AS AOB, BOC, COD,

PQ = RS.
In AABC, /.BAC=90; P is the centre of the square
described on BC prove that AP bisects /_ BAG.
In AABC, L BAG = 90; prove that the perpendicular
bisectors of AB and AC meet on BC.

DOA

3.

BOD =

Z.

prove

4.

5.

ABC

is

a scalene

bisector of

triangle
6.

BC

triangle;

prove that the perpendicular

and the bisector of

/.

BAG

meet outside the

ABC.

ABCD is a parallelogram; E, F are the circumcentres


AS ABD, BCD prove that EBFD is a rhombus.

of

7.

The

two

fixed lines

circle is

ABCD

is

^-^

ABC

AD, BE are

prove this for the case where

altitudes of

AD, BE,

AABC;

DE

AABC, AB = AC; P

circumcentres of
to PF.

the radius of the circum-

AB = CD

/.

B AC = 90.

find a point

AOAB s AOCD.

bisectors of

In

A,

is

a quadrilateral such that

such that

11.

OB

the area of the triangle

8. If

10.

OA,

AOPQ is

centre of

9.

PQ of given length lie on


the locus of the circumthat
;
prove
a circle, centre O.

extremities of a variable line

prove that the perpendicular

are concurrent.

is

any point on BC;

AS ABP, ACP

prove that

E,

AE

F are the
is

parallel

CONCISE GEOMETRY

100

THE IN-CTRCLE AND EX-CIRCLES


12.

In Fig. 112,

if

BC = a, CA-&, AB = c,

and

prove that
(i)
(ii)

AY = s~a

AQ^.

(iv)

BP-XC.
YQ-ZR.

(v)

XP-6-c.

(iii)

IX

(vi)

(vii)

(viii)

(ix)

(x)

A = area of triangle ABC.

where
3

lP

B, I, C, ^ are concyclic.
AZ-f BX-f CY =
if L BIG -100, calculate
,s.

FIG. 112.

13.

AB

14.

circle; the tangents at A, B meet at T;


the
that
in-centre
of
lies on the circle.
prove
is the in-centre and
the circumcentre of
ABC; prove
is

a chord of a

ATAB

ZIAO = (ABC~Z.ACB).
is the in-centre of A ABC
prove that L AIC = 90 -f \ L ABC.
is the in-centre and AD is an altitude of A ABC;
prove that
L IAD \( L ABC - L ACB).
In Fig. 112, prove that AB - AC = BX - XC.
that

15.
16.

17.

18.

The

A ABC

in-circle of

19.

ABCD

20.

Two

BC at X, prove that the


touch each other.

touches

AS ABX, ACX

in-circles of

101

CONCURRENCY PROPERTIES

THE TRIANGLE

a quadrilateral circumscribing a circle; prove that


the in-circles of
ABC, CD A touch each other.
is

concentric circles are such that a triaiglo can be inscril>e<l


in one and circumscribed to the other; piove that the

triangle is equilateral.
21. In
ABC, Z.BAC 90;

in-circle of

22.

The

AABC

prove that the diameter of the

BC.
equals AB + AC
of a variable line lie on two fixed lines

extremities P,
CD; the bisectors of

AB,

Z_s

CQP

APQ,

meet at R;

find

the locus of R.
23.

is

the in-centre of

BC

escribed to

AABC;
Ij

C lie on a
AABC; if
AQ = AC.

AB

at Q, prove
the in-centre of

is

the centre of the circle

cuts the circumcircle of

prove that I, lt B,
is the in-centre of
\

24.

I,

circle,

AABC at

centre P.

the circumcircle of

ABIC

cuts

AABC

AP, AQ are the perpendiculars


;
that
PQ is parallel to BC.
;
prove
is the
26*. The in-circle of
touches BC, CA at X, Y;
=
at
90.
XY
meets
Al
in-centre;
P; prove Z.BPI
25.

is

from

to Bl, Cl

AABC

THE ORTHOCENTRE
27. If

AD, BE,

CF

are the altitudes of

AABC

and

if

is its

orthocentre (see Fig. 110), prove that


(i)
(ii)

(iii)

(iv)

(v)
(vi)
(vii)

/.BHF^BAC.

Z.BHC+Z.BAC = 180.
AS AEF, ABC are equiangular,
AS BDF, EDC are equiangular,
AD bisects L FDE.
Z.EDF = 180-2 iBAC.
2

is

in-centre of

ADEF.

28.

Where

29.

30.

= 90= Z.QDC prove that AQ is perpendicular to BD.


If D is the orthocentre of AABC, prove that A is the orthocentre of ABCD.

is

is

the orthocentre of a right-angled triangle 1


ABCD such that

a point inside the parallelogram


;

Z.QBC

CONCISE GEOMETRY

102
H

31. If

is

As AHB, AHC

circles of

32.

is

A ABC,

orthocentre of

the

the in-centre and

lf

prove that the circum-

are equal.
I

2,

34.

is

is

the orthocentre

an altitude; prove that BF = FH.


the circumcentre and H the orthocentre of

CHF
that

AABC,

are the ex-centres of

prove that l is the orthocentre of Al


33. In AABC, AB = AC, L BAG = 45; H

a:

is

AABC

prove

/. HBA = Z.OBC.

are the mid-points of BC, CA, AB ; prove that the


orthocentre of
is the circumcentre of AABC.

35. P, Q,

APQR

36.

is

circumcircle of
37.

AABC AH

the orthocentre of

AABC

the circumcentre,
is the in-centre, H
Al
that
bisects Z.OAH.
AABC; prove
is

38. BE,

CF

are

AABC; O

of

altitudes

BC at D
HD = DP.

meets

P; prove that

at

is

is

and the

the orthocentre of

its

circumcentre;

OA

is perpendicular to EF.
prove that
the circumcentre of
39. H is the orthocentre and

AABC AK

is

parallelogram,

(ii)

prove that (i) BHCK is a


equals twice the distance of O from

a diameter of the circumcircle

CH

AB.
40*.

is

the orthocentre and

AC = AH,
41.

42*.

is

prove

43.

the circumcentre of

AABC BH

the orthocentre of

K; prove AH = AK.
The altitudes BE, CF
mid-points of
to EF.

AABC

if

/.BAG = 60.

of

AABC

meets the circumcircle at

meet at H

AH, BC; prove that PX

Given the base and

is

P,

are the

perpendicular

vertical angle of a triangle, find the locus

of its orthocentre.

44. [Nine Point Circle.]


H is its orthocentre

BC, CA,
(i)
(ii)

PZ

AD,

B, CF

are altitudes of

AABC

X, Y, Z, P, Q, R are the mid-points of


AB, HA, HB, HC ; prove that

is

parallel to

L PZX = 90

(iii)

P, Z, X, D,

(iv)

The

circle

D,

E, F.

and

lie

BE and ZX

is

parallel to

AC.

L PYX - 90.
on a

circle.

through X, Y,

passes through P, Q, R,

THE TRIANGLE

CONCURRENCY PROPERTIES

103

THE CENTflOID
are the mid-points of BC, CA, AB ; prove that the
ABC, XYZ have the same centroid.

45. X, Y,

triangles

46.

ABCD

47. If the

at

prove that

medians AX,

BY

is

AQ

the mid -point of

AABC

of

AB; CP

BC.
meet at G, prove that

bisects

AS

CGY

are equal in area.


the centroid of

BGX,
48. If

a parallelogram;

is

BD

cuts

AABC

is

and

if

AG = BC,

prove that

L BGC = 90.
49. If

two medians of a triangle are equal, prove that the triangle


is isosceles.

50. X, Y,
of

51.

BC, CA, AB AD is an altitude


ZXY = L ZD Y = Z. BAC.
medians of AABC, prove that BY+CZ

are the mid-points of


;
prove that Z.

AABC

AX, BY,

CZ

are the

>AX.
52. If the centroid and circumcentre of a triangle coincide, prove
that the triangle is equilateral.
53.

ABCD
AD

is

a parallelogram; H,

prove that
54*. In a tetrahedron
;

corners add

CH

up

tetrahedron, that

ABCD,
to 180
its

CK

and

are the mid-points of

trisect

AB,

BD.

the plane angles at each of three


prove, by drawing the net of the

opposite edges are equal.

RIDERS ON U0<

IV

>

PROPORTION
THKOBEM
If

the heights AP,

XQ

51

of the triangles

ABC, XYZ

are equal,

AABC^BC
AXYZ YZ'
A

P C

FKI. 113.

THKOKKM 52
(1) If a straight line,

drawn

parallel to the base

AC (produced if
AH
AK
,AH
= AK
then - = - and
HB KC
AB AC
If H, K are joints on the sides AB, AC (or
AH - A
of the triangle ABC such that
~,
HB KC
ABC,

cuts the sides AB,

BC of the triangle
necessary) at H, K,

,,

(2)

to

the sides produced)

then

HK

is parallel

BC.

B
Fia.

K
FIG. 114(2).

FIG. 114(3).

CONCISE GEOMETRY

106

PROPORTION
EXERCISE XXI
1.

What

is

(i)

the value of the following ratios


2 f t. ; (ii) U. '2s.
20 min.
(iii)
:

ins.

3 sq. ft. 2 sq. yd.


25 cms. 1
:

2.

Find x
(i)

x 24
:

3.

-=

If

-,

^=4

10,

= c = -,

If

fill

/...v

/..\

+ d+f

Solve the equations

v
(

a,

(m)

up the blank spaces

+ b = c-\-d

in the following

_;

a+c

,...,

T \'

'6-3.

=-

"*--

(i)

&

*~

-2

5,r

(ii)

8 inches,

(ii)

*'

3J, 5, 8, 12

6.

Are the following in proportion

7.

6 degrees, 1 2 degrees, 9 inches ?


Find the fourth proportional to (i) 2, 3, 4 ;
Find the third proportional to (i)
\ ; (ii)

(i)

(ii)
J?,

at, be, cd.

.ry.

10.

Find a mean proportional between (i) 4, 25 ; (ii) a 2 6, be2


A line AB, 8" long, is divided internally at P in the
2:3; find AP.

11.

9.

AB, 8"

line

7:3;
12.

AB
to

13.

AB
to

is

A
is

,.. x

~b
5.

(iv)

x feet 5 yards = 2 3 (iii) 6 :x =


50 minutes = 3 shillings x shillings.

(u)ad-6e,

a,

,.^

3 m.

(vi)

prove that

(,)_=-;

4.

hr.

120

(ii)

(iv) 2 hoars

/. v

in the following

(v) 3 right angles

find

long,

is

divided externally at

ratio

in the ratio

BQ.

divided internally at
or B !

in the ratio 5

6.

Is

nearer

divided externally at
or B ?

in the ratio 9:7.

Is

nearer

107

PROPORTION
14.

AB

15.

16.

ABODE

divided externally at

is

to

a straight

is

line

line

at

is

long,

line of length

BE

(i)

is

J|

(ii)

A and

divided by

(iii)

D.

divided internally at C and externally


3 ; O is the mid-point of AB ; prove

2
.

divided internally at C and externally


4:1, O is the mid-point of CD prove

AB, 6" long,


in the ratio
1

ration

which

AB BC CD DE-=

such that

AC.

in the ratio 7

AO = 6BO

that

find

00,00 -OB

that

19.

ratios in

AB, 8*

line

Find the

BE = 4",

at

nearer

AB, 6" long, is divided internally at P in the ratio


2:1, and externally at Q in the ratio 5.2; find the ratios
in which PQ is divided by A and B.

If

18.

Is

5.

B1

or

Find the

in the ratio 3

line

1.3.2-5

17.

is

and

x"

is

find the length of

CD.

divided internally in the ratio <i.l>\

find the lengths of the parts.

20.

21.

line of length

divided externally in the ratio

AB

jc

ABCDEF

find

is

a\b\

in tho ratio ,y;:y;

OP
-.

divided internally at

is

ratio

23.

is

find the lengths of the parts.


and divided at
line AB is bisected at
find the ratio

22.

if

CD
(i)

(ii)

and externally

the ratio in which

a straight line such that


find

p:r/:r:s:t',

(i)

*?,

(ii)

^,

at

in the

divides

CD.

AB BC CD DE EF =:
:

(m) the

ratios in

which

If BD -a;", find AE.


divide OF.
AB BC CD =
two
AXYZ
are
straight lines such that
ABCD,
AX XY YZ. Fill up the blank spaces m the following
AB AC
XZ_
a BC
(I11)
(U)

A and E

24.

,..

,.

WAX"-"'

25.

ABC

26.

The
is

is

a straight

..v

AY~AC
AD^AZ'
AR
= A. AB, find
line; if AC

sides of a triangle are in the ratio

inches

find the sides.

x y
:

in

and

terms of

its

A.

perimeter

108
27.

CONCISE GEOMETRY

AB

is parallel

to

CD OB = 2", OD - 2 J", BC = 5*
;

AD.

find

D
FIG. 115.

28.

AB, CD, EF arc


lind BD.

parallel lines;

AC =-2', CE = 3", BF-4";

FIG. 116.

29*

-=
GB
OQ-6;
;

AP, BQ,

GN

are

perpendicular to

OX

OP -a,

JJL

30*.

find

The medians

ON.

of

AABC

OQ = b, OR = e
31.

ABC

is

JAB and
to

PQ

prove

AP, BQ, CR,

OX

if

GN are
OP = a,

AB,

AC

such that

a line through

AP =

parallel

AB.

32. Three parallel lines

prove that

to a line

ON = J(a 4- b + c).

A; P, Q are points on
CQ = JCA; prove that

bisects

intersect at

the perpendiculars from A, B, C,

AX, BY,

CZ

cut two lines

ABC,

XYZ

PKOPOFTION
33.

The diagonals
parallel to

34.

35.

36.

ABCD

of the quad.

DC, prove

BC
AH.AC-AK.AB.
line parallel ^o

cuts

iuterse

AB

if

is

AB,

AC

I-

-rove

a,t

tj AB
-a
any point inside the AABC; a ae XV
to
u 6 3
cut
cuts OA, OB at X, Y; YZ is drawi *ur,ii
OC at Z ; prove XZ is parallel to AC.
ABCO is a quadrilateral P is anj po; nt on AB ; lines PX, PY
are drawn parallel to AC, AU to cut BC. 3D at X, Y
prove
XY is parallel to CD.
D is the foot of the perpendicular from A to the bisector
of /.ABC; a line from D parallel to BC cuts AC at X;
prove AX = XC.
is

-.

37.

38. In Fig. 118, prove

O
FIG. 118.

39.

is

the in-centre of

= BC CA
:

40.

AABC;

prove that

AlBC AICA
:

AB.

In Fig. 118, prove

A AQD AC
= -.

Rf^I^

Rfi

AH = HB, AK = 2KC;

find the ratio of the


41*. In Fig. 119,
areas of the small triangles in the figure; hence find the
,.

ratio

CO
_.

FIG. 119.

CONCISE GEOMETRY

110
42*.

43*.

44.

is a
H, K are points on AB, AC such that HB =
;
=
and
KC
JAB
JAC; BK cuts CH at O; prove BO = OK
and CO = 2OH. [Use method of ex. 41.]

ABC

ABC is a A Y, Z are points on AC, AB such that CY = $CA


and AZ = JZB; BY cuts CZ at O; prove OY = |BY and
OZ = fCZ. [Use method of ex. 41. J
Two circles APQ, AXY touch at A APX, AQY are straight
AP = AQ
r
;

lines; prove

^.

C cuts AB
^
=
produced, AD produced at P, Q prove
BP DA
46*. ABCD is a parallelogram a line through C cuts AB, AD, BD
2
=
(produced if necessary) at P, Q, O prove OP OQ OC
47. ABC is a A; three parallel lines AP, BQ, CR meet BC,
CA, AB (produced if necessary) at P, Q, R prove that
45.

ABCD

a parallelogram;

is

any

line

through

B_P

PC
48*.

x99 x *?=l

is

AB

RB

QA

any point inside

parallel to

prove
49*.

ABC
P,

is

AD, BE,

CF

a triangle; a

...

[This

D, E,

to

F are points on BC, CA,


are drawn from O

lines

meet BC, CA,

AB

at P, Q,

OP -f OQ + OR = AD.

Q, R ;

prove

AABC

AD = BE = CF;

such that

(i)

is

CX

is

drawn

BP BR
=
pc R-x

known

....

(n)

line

cuts

BC

produced, CA,

parallel to PQ, meeting

BP CQ
x
pc QA

AR
x

RB

as Menelaus'
Theorem.]

==1

AB

AB
at

at

111

SIMILAR TRIANGLES

SIMILAR TRIANGLES
THEOREM 53

XYZ are equiangular (/.ABC= Z.XYZ


Z.ACB-ZXZY),
AB = BC = CA
then

If the triangles

ABC,

and

,,

c.

Fro. 120.

THEOREM 54
If the triangles

ABC, XYZ

~ -^-, then
^=
X
AR

are such that

triangles are equiangular, Z.

ABC =

Z.

BC

CA

fm

21 /V

XYZ,

L ACB =

Z.

the

XZY,

ZlBAC= Z.YXZ.
THEOREM 55
If,

in the triangles

ABC, XYZ, Z.BAC-Z.YXZ and

then the triangles are equiangular,

Z.ACB=ZXZY.

= ^?
XY XZ>

Z.ABC= /.XYZ and

CONCISE GEOMETRF

112

SIMILAR TRIANGLES
EXERCISE XXII
1.

2.

pole 10' high casts a shadow 3J' long; at the same time
a church spire casts a shadow 42' long.
What is its height ?

In a photograph of a chest of drawers, the height measures


6" and the breadth 3*2* if its height is 7J feet, what is its
;

breadth
3.

Show

that the triangle whose sides are 5'1", 6*8", 8*5"

is

right-angled.
4.

halfpenny (diameter 1") at the distance of 3 yards appears


nearly the same size as the sun or moon at its mean distance.
find its

2160
5.

How

Taking the distance of the sun as 93 million miles,


diameter.
Taking the diameter of the moon as

miles, find its

mean

distance.

far in front of a pinhole

camera must a man

stand in order that a full-length photograph


"
on a film 2
high, 2 J* from the pinhole ?
6.

The

slope of a railway

is

marked

as

in 60.

may

6' high
be taken

What

height

(in feet) does it climb in

7.

f mile 1
A light is 9' above the floor ; a ruler, 8* long, is held horizontally 4' above the floor ; find the length of its shadow.

8.

Two

9.

The bases

triangles are equiangular \ the sides of one are 5", 8", 9"
the shortest side of the other is 4 cms. ; find its other sides.

ef the
10.

of

two equiangular

first is

tho height

5* ; find the area of the second.

AABC, AB = 8", BC = 6*, CA = 5" a line XY parallel to


BC cuts AB, AC at X, Y AX = 2" find XY, CY.
In quadrilateral ABCD, AB is parallel to DC and AB = 8",
AD = 3", DC = 5" AD, BC are produced to meet at P find

In

11.

triangles are 4*, 6*

PD.
12.

line parallel to

5"

BC meets AB, AC at
XY are 2'" apart.

the lines BC,

AXY.

X,

Y; BC = 8*,

XY =

Find the area of

SIMILAR TRIANGLES
13.

113

In Fig. 121,
if

(i)

if

(ii)
(iii)

if

(iv)

if

AO = 3*, OB = 2", AB = V, DC
AO^S", BO = 4*, AC^T", find

1 J", find

CO, DO,

BD.

PA = 9", PB^S", AB = <r, PC = 3* find PD, CD.


PA = 9", PB = 8", AC = 6", PC = 4*, find BD, D.
P

Show

that the line joining (1, 1) to (4, 2) is parallel to and


half of the line joining (0, 0) to (6, 2).
are cut by two parallel lines PQR,
15. Three lines APB, AQC,
AR = 3*, RD = 2", BC = 4" ; find PQ.
;
14.

ARD

BCD

16.

In Fig. 122, AB is parallel to


;
DQ = 9 ; findOD, BP.

OD AB = 6', BO = 20', BE = 5',


;

B
FIG. 122.

17.

The diameter
15*

3" from
18.

of the base of a cone

is

9" and

its

height

find the diameter of a section parallel to the base

is

and

it.

a straight line ; AC, XY, BD are the perpendiculars


from A, X, B to a line CD; AC = 10, BD = 16, AX = 12,

AXB

is

XB = 6;

findXY.

OX

and at distances
are points on the same side of a line
and
R
AB
divide
it
5*
from
internally and externally
;
1",
and R from OX.
in the ratio 5:3; find the distances of

19. A,

CONCISE GEOMETRY

114
20.

rectangular table,
floor

wide, 8' long, 3' high, stands on a level


the shadow on the floor of the
;

long; find the length of shadow of the


longer side and the height of the lamp above the table.
sphere of 5" radius is placed inside a conical funnel whose
shorter side

21.

5'

under a hanging lamp


8'

is

is 12" and whose greatest diameter is 14"; find


the distance of the vertex from the centre of the sphere.

slant side

22.

The length

of each

arm

of a pair of nutcrackers is 6"

the distance between the ends of the arms

diameter

placed with

is

its

when a nut

find

1" in

nearer end 1" from the apex.

23. In Fig. 123,


= 7,
If
(i)
(ii)

If

PQBR is a rectangle.
AB
PQ - 1, PR = 2, find BC.
AB = 7, BC = 5, PR = #, PQ=y,

between

find

an equation

#, y.

AK

FIG. 123.

24. In

AABC,

bisector of

25.

ABC = 90, AB - 5", BC = 2"


AC cuts AB at Q find AQ.

Z.

The diameter

the perpendicular

of the base of a cone

parallel section,

V from

the base,

is

8*

is

6"

the diameter of a
find the height of

the cone.
26. In

Fig.

124, AB, PN, DC are parallel;


calculate PN.
;

CD = 3"

FIG. 124.

AB = 4*, BC = 5*,

115

SIMILAR TRIANGLES
27.

ABCD is a quadrilateral such


AC -5", BC = 3", 00 = 10";

that

L ABC = 90 = L AGO,

calculate the distances of

from BC, BA.


28.

PQ

a chord of a circle of length 5 cms.; the tangents at P, Q


T ; PR is a chord parallel to TQ ; if PT = 8 eras.,

is

meet at

PR.

find

29.

(i)

man, standing in a room opposite to arid 6' from


a window 27" wide, sees a wall parallel to the plane
of the window.
With one eye shut, he can see
-

18"

less

length

of

the wall from the

than with

wall

both

eyes

open;

find the distance

2" apart,

supposing his eyes are

window and the

of

total length of wall

visible.
(ii) If

the

window

is

covered

by a shutter containing a

vertical slit \" wide, show that there is a part of the wall
out of view which lies between two parts in view and find
its length.
(iii)

A man

in

bed at night sees a star pass slowly across a

shortly afterwards, this occurs


that
he sees the same star twice 1
again,
possible
Explain your answer by a figure.
vertical slit in the blind
la

30.

it

rectangular sheet of paper ABCD is folded so that D falls


on B; the crease cuts AB at Q ; AB = 11", AD = 7";
find

AQ.

HK

31. Fig. 125 represents an object


cave mirror, centre O, focus F.

and

CH-w, CP-v, CF = FO=/


1
1
1
= vx
prove that (i) - = -(u) y
.,

its

,.. x

,. v

image

PQ

in a con-

116

CONCISE GEOMETRY

32. In Fig. 126, with the

~
t

4- -,

and

find

same notation as in

ex.

31, prove that

in terms of x, u, f.

1.
""--

r~^

H;:

^i

FIG. 126.

33. Fig. 127 represents an object

concave

HK

and

its

image

a thin

PQ

iu

PQ

in a thin

lens, centre O, focus F.

prove that

--

(i)

= ?--U

(ii)

r-7

B\ -A* z- : : : ; Q- : :

--K
-i

*F

"H

FIG. 127.

34. Fig.

128 represents an object

convex

HK

and

its

image

lens, centre O, focus F.

= v. OF
=v
OH = u. OP
^"
**
"* = ^i
=f,
x. PO
j HK
\e
j

prove that -

= - + -, and

f u

'

>

find

ii j

in terms of #. u,

J<

.13

...P....r::ll\Li-----'
r

- "*""""'""

""

H"

]V:;-----'--~T ]o

cf"

f.

v
FIG.. 128.

35.

OAOB

is

a quadrilateral on level ground;

Z.OBC, ZLOAC=135,

OB = 9', OA = 12';

Z.AOB = 90 =

covered by
a plane roof resting on pillars ; the
at
pillars
A, B are 10'
high, the pillar at O is 8' high ; find the height of the pillar

atC.

it is

117

SIMILAR TRIANGLES
36.

DC

AB,

are the parallel

diagonals cut at

CF

37. BE,
38.

Two

ABC

is

BC

at

spaces in

and the

-- =

prove

AC

CF

up the

fill

cut a circle at A, B, C,

circle

~- -

(ii)

BD

circle at

OC

the bisector of

prove
l

(ii)

AP

AB

fill

L BAG

= ^ and

PC
BO
-= = ~
AB
In AABC, Z.BAC = 90; AD is an altitude;
DC
AC
CD = -.
__. ~
the equation
and complete
H
F
DA DB
AC BC
The medians BY, CZ of AABC intersect at G;
the equation
1

41.

(i)

A inscribed in a

OCD

OAB,

the

OC CD

AABC;
*=

(i)

straight lines

up the blank
40.

prove

are two intersecting chords of a circle

blank spaces in
39.

are altitudes of

COD

AOB,

ABCD

sides of a trapezium

cuts

complete

prove that

42.

CF

43. BE,

44.

Two

are altitudes of

lines

AOB, POQ

cut again at

AABC;

prove that

intersect at

prove that

,/vp

~ =^.

the circles

prove that

AOP,

BOQ

'^y

common tangents of two non-intersecting circles


divide (internally and externally) the line joining the centres
in the ratio of the radii.

45. Prove that the

46.

is

the mid-point of

angles on opposite
47.

AB

is

AB

a diameter of a

48.

APB,
that

AQB
BP

BQ

AB

are two circles

A
;

if

ABP

circle

from P to the tangent at

MYB are equilateral triXY cuts AB at Z prove

AXB,

sides of

prove

PAQ

PT is the
PT = AP

perpendicular

is

a straight

equals the ratio of their diameters.

line,

prove

CONCISE GEOMETBY

118
49.

ABCD

a parallelogram

is

produced,
50.

ABCD

AD

CD

BC
.=
GH AB
EF-

prove

at E,

prove

BX

AB

AB

cuts

two perpendicular lines are drawn


F ; the other cuts AD, BC at G, H

51.

In the quadrilateral

52.

prove AB = CD,
The diagonals AC,

ABCD, L ABC -/ ADC and

BD

of the quadrilateral

AOD

the radius of the circle

if

any Hue through

produced at X,

a rectangle

is

one cuts AB,

is

ABCD

^?^ CD

DA

BC

meet at

three times the radius of

BOC, prove AD^3BC.


a parallelogram ; P is any point on

the circle
53.

ABCD
AC

54.

AB,

is

O
V

at

DC

;
'

prove

at X,

cuts

-.

AB cuts CA, CB
AB - XY.

prove
a parallelogram
;

DP

DQ

are the parallel sides of the trapezium

line parallel to

ABCD

AB

AB

at H,

ABCD any
DK cut AB
;

DH,

AC

is any point on
lines POQ,
;
;
are drawn, cutting AB, CD, BC, AD at P, Q, R, S ;
prove PS is parallel to QR,
56. In
is bisected at E;
ABC, D is the mid-point of BC ;
BE cuts AC at F; prove
2FA. [Draw EK parallel to

55.

is

ROS

AD

CF=

BC
57.

BC,

to cut

YZ

AP,

AC

at K.]
are the bases of

XQ

are medians

two similar

triangles

ABC, XYZ

L BAP = L YXQ.

prove
58. P is a variable point on a given circle ; O is a fixed point
outside the circle \ Q is a point on OP such that OQ = $OP ;
59.

prove that the locus of Q is a circle.


is a
;
E, F are the mid-points of AB,

ABC

drawn so that FD

-=

2EF

prove

BF

bisects

AC EFD
;

AABC, Z BAC = 90; ABXY, ACZW are squares outside


BZ, CX cut AC, AB at K, H prove AH - AK.
In AABC, the bisectors of /.a ABC, ACB meet at D; DE,
DF are drawn parallel to AB^ AC to meet BC at E, F prove

60. In

AABC

61.

is

AD.

BE_BA
FC~AC*

119

SIMILAR TRIANGLES
62*. Tn

AABC,

BAC = 90 AD is an altitude H, K are the


AS ADB, ADC prove that AS DHK, ABC are

Z.

in-centres of

similar.

63*. D, E, F are the mid-points of the sides BC, CA, AB of a


triangle ; O is any other point ; prove that the lines through
D, E, F parallel to
64*. In AABC,

ABa

the

bisector of

PQ "
^
PA

OC are concurrent.
AC BQ is the perpendicular from B to
BAC BC cuts AQ at P prove? that

OA, OB,
.

CONCISE GEOMETRY

120

RECTANGLE PEOPERTIES OF A CIRCLE


THEOREM 56
(i)

If

two chords

AB

and

CD

of a circle intersect at a point

(inside or outside a circle),

then

OA.OB = OC.OD.
,0

FIG. 129(1).

(ii)

If

FIG. 129(2).

from any point O outside a circle, a line is drawn touching


the circle at T, and another line is drawn cutting the circle
at A, B,

then

OA.OB = OT2

FIG. 130.

RECTANGLE PROPERTIES OF A CIRCLE

121

THEOKEM 57
If

AD

an altitude of the triangle ABC, which

is

is

right-angled

at A,

then

AD 2 = BD DC
.

(i)

(ii)

BA2 = BD BC.
.

FIG. 131.

DEFINITION.

is

If a, #, b are such that

(iii)

properties are important


If two lines AOB,
are such that

COD

If

If

b.

then A,B, C,
(ii)

mean proportional between a and

called the

The converse
(i)

~ = ~ or x* = ab,

lie

on a

AC OB = CO
.

two lines OAB,

ODC are such that OA OB = OC

then A, B, C,

two

lines

then the

OBA,

on a

lie

OT

OD,

OD,

circle.

circle.

are such that

circle

OA OB = OT 2
OT at T.
.

touches

through A, B,
Alternative proof of Theorem 57
(i) Draw the circle on BC as diameter
:

A, since

L BAG = 90.

again at E.
Since the chord

AE

is

Produce

it

AD

perp. to diameter

passes through
to cut the circle

BC,

AD = DE.

But

(ii)

Draw

the

AD.DE=BD.DC;
AD 2 =BD.DC.
circle on AC as diameter

D, since

/.

Z.ADC = 90, and

it

touches

by Theorem 56 (ii),

passes through
at A, since

BA

BA2 = BD

BC.

CONCISE GEOMETRY

122

RECTANGLE PROPERTIES OF A CIRCLE


EXERCISE XXIII
1.

Find a mean proportional between

2.

From

3.

a point P on a circle,
diameter AB ; AN - 3", NB

In

A ABC,

3 and 48

5.

AB, AC at
The diagonals

(ii)

is

an

altitude;

I2x,
to a

AB = r>",

BD.

find

In

drawn perpendicular
find PN.

AB-8, AC -12; a circle


P,Q BP = 5 find CQ.

4.

through B,

cuts

of a cyclic

<

AC = 9, BD = 12, OA - 4

quadrilateral

find

ABCD

meet at O;

OB.

In Fig. 132,

AB - 9, BO = 3, find OT.
OB - 6, OT- 12, find AB.
If OA= 3, AB - 2, AT- 4, find BT.
If AB = 8, AT -6, BT-5, find OT.

(i)

If

(ii)

If

(Hi)
(iv)

7.

is

12"

A ABC, Z BAG -00; AD

AC = 12";

G.

PN
-=

(i)

ABC is a triangle inscribed in a circle; AB = AC = 10"


BC = 12" AD is drawn perpendicular to BC and is produced
to meet the circle in E
find DE and the radius of the circle.
In A ABC, Z ABC = 90, AB = 3", BC = 4"; find the radius
of the circle which passes through A and touches BC at C.
In A ABC, Z.BAC-9CT; AD is an altitude BC-a, CA = 6,
AB = c, AD = A, BD = x, DC = y prove that (i) A2 = xy
;

8.

9.

(ii)

i'=ry (* + y);

10. In Fig. 132,

if

(iii)

he

OA-2OT,

* b*

prove

(iv)

AB-3BO.

123

RECTANGLE PROPERTIES OF A CIRCLE


11.

COD are two perpendicular chords of a circle, centre K;


AO = 6, CO = 10, OD = 12 find OK, AK.
X is the mid-point of a line TY of length 2" TZ is drawn so
that /. ZTX = 45
a circle is drawn through X, Y touching
TZ at P prove /1TXP = 90, and find the radius of tlie

AOB,

12.

circle.
1

3.

ABC

is

A inscribed in a

circle

14. Express, in the

= r2
(ii) pq
15.

prove
16.

The diagonals

Two

form of equal

(iii)

17.

ratios, the
.

lines

OCD

OAB,
.

(i)

xy^ab\

- OP 2
(iv) ON OT
ABCD intersect at O
.

cut a circle

at A, B,

C,

prove

AD.

O if D is the
=
CD.
CA
CB
CO
AB, prove
the bisector of L ABC
In A ABC, AB - AC and L BAC = 36
2=
BC
AP 2
at
P
AC
CP
meets AC
prove
at H
intersect
The altitudes BE, CF of A ABC
prove that
=
CH.HF; (ii) AF AB AE AC (iii) CE.CA
(i) BH.HE
= CH.CF.
In AABC, AB = AC D is a point on AC such that BD = BC
2
prove BC = AC CD.
Two circles intersect at A, B P is any point on AB produced
Two

CD

chords AB,

of a circle intersect at

20.

equations

19.

AB

AD OC = BC OD.

mid-point of arc

'18.

of a cyclic quadrilateral

OA BC - OC
.

meets

AB^r, BD = #, CD=-y;

OA OB = OC OD
.

the tangent at

produced in D; BC=^jp, CA-</,


find x, y in terms of p, <?, r.

21.

prove that the tangents from

to the circles are equal.

AABC, Z.BAC = 90, AB = 2AC; AD is an altitude;


prove BD = 4DC.
PQ is a chord of a circle, centre O the tangents at P, Q meet
2
at T OT cuts PQ at N
prove ON OT = OP
AB is a diameter of a circle PQ is a chord the tangent at
B meets AP, AQ at X, Y prove AP AX = AQ AY.
AB, AC are two chords of a circle any line parallel to the
=
tangent at A cuts AB, AC at D, E prove AB AD AE AC.
P
is
a
on
BD such
is
a
ABCD
point
cyclic quadrilateral;
=
that L PAD
L BAG prove that (i) BC AD AC DP
=
AC.BP; (iii) BC AD + AB CD- AC BD.
(ii) AB.CD

22. In

23.

24.

25.

26.

CONCISE GEOMETRY

124
27.

AB

a diameter of a

is

chords
28.

30.

prove

are equal

AD is an altitude of

ABC
29.

O; AP, PQ

centre

circle,

AP PB = AQ OP.

A ABC

"-

equals

prove that the radius of the

[Draw diameter through

circle

A.]

B PQ is their common tangent


bisects
AB
PQ.
prove
In AABC, AC is equal to the diagonal of the square deD is the mid-point of AC prove Z. ABD =
scribed on AB

Two

circles intersect at A,

Z ACB.
PR 2 = PQ. RQ; TQR
= PQ.
prove PT

31.

A line PQ

32.

A such that TQ = TR = PR
a A inscribed in a circle;
PQR

is

divided at

so that
;

QR

produced at T; prove

AABC, ZBAC = 90; E is a


AE = AB prove BE BC = 2AE 2
of

altitude

BC

point on

such that

AD is an
AB = CD,

P meets

TO =r _PO 2

33. In

34.

the tangent at

is

is

AABC;

AB.BC--=AC 2 and

if

if

Z.BAC = 90.

prove
chords AB, AC of a circle are produced to P, Q so that
AB = BP and AC = CQ ; if PQ cuts the circle at R, prove

35.

Two

36

The tangent at a point C on a circle is parallel to a chord


DE and cuts two other chords PD, PE at A, B; prove

AR 2 = PR.RQ.

37.

AC "~
= AD
CB BE'
AB is a diameter

of a circle, centre

meet any other tangent at H,


38.

39.

Two

lines

OAB,
drawn

OCD

is

prove

XO 2 = XA.XD.
a

parallel to

A inscribed

BC

meet

to

in a circle

cuts the tangent at


;
'

AB

is

at

a chord of a circle
;

prove

PH, PK,

PH.

PX

the tangents at A,

AH BK = AO 2
.

line

AQ
APB

DA

through O,

produced at

through

parallel

1
i

the tangents at A,

are the perpendiculars to

PK-PX 2

a line through

A in P a
AP
AB 2
- =to AB cuts AP in Q
v prove
^
AC 2

40*.

prove

cut a circle at A, B, C,

a line

ABC is
to AC

TA, TB,

j
\

AB

RECTANGLE PKOPERTIES OF A CIRCLE


41*.

AC

AB,
line

are tangents to the circle

prove

is

a straight

BE CD = BD CE.
.

are points on the radius


.
circle, centre O, such that

42*. P,

BDCE ADE

125

OA

and

OA

produced of a

OP OQ = OA 2 R is any other
point on the circle prove that RA bisects Z. PRQ.
In AABC, AB-AC, Z.BAC = 36; prove AB 2 -BC 2 =
AB BC.
The internal bisector of /.BAG cuts BC ut D, prove that
AD 2 = BA AC - BD DC. [Use ex. 1 7.]
;

43*.

44*.

45*

The external

bisector of

Z.BAC

cuts

BC

produced at E;

2
- BA AC.
prove that AE = BE EC
46*. ABCD is a parallelogram ; H, K are fixed points on AB,
HP, KQ are two variable parallel lines cutting CB,
.

AD
CD at

P,

prove BP,

DQ

is

constant.

CONCISE GEOMETRY

126

AREAS AND VOLUMES


THKOEEM 58
If

ABC, XYZ

are two similar triangles,

and

if

EC,

YZ

are a pair

of corresponding sides,

AABC^BC*
AXYZ YZ2

then

C
FKI. 133.

More

generally, the ratio of the areas of


is equal to the ratio of the squares

any two similar polygons


on corresponding sides.

THEOREM 59
If

AB

and

CD

PAB,
,.
(i.e.
V

are corresponding sides of any two similar polygons


QCD, and if AB, CD, EF are three lines in proportion

AB
CD

CD,
),

EF''

The following facts are also


The ratio of the
(i)
(ii)

of importance (see ex. 34, 35)


areas of the surfaces of similar solids
:

equals the ratio of the squares of their linear dimensions,


of the volumes of similar solids equals the ratio

The ratio

of the cubes of their linear dimensions.

AREAS AND

127

VOLU]\<ttS

AREAS AND
EXEBCISIJ XXIV
1.

6'
high (not necessarily reeUM,
of material for covering
h-v

screen,
L; 7 sq.

ft.

mm

a screen of the same shape, 4'

liig}

iPr
';

'quires

uf ;ded for

2.

On

3.

the area (in acres) of the plot 1


The sides of a triangle are 6 cms., 9 cms., 12 cms. ; how many
triangles whose sides are 2 cms., 3 cms., 4 cms. can be cut

4.

out of it 1 How would you cut it up ?


Show how to divide any triangle into 25 triangles similar
The area of the top of a table, 3 feet high, is 20 sq. ft.

is

5.

map whose scale is 6" to the mile, a plot of ground


represented by a triangle of area 2 J ;;q. inches what is
a

area of
of the

shadow on the

its

lamp above the

floor is

45

sq.

ft.

to
;

it.

the

find the height

floor.

light is 12 feet above the ground; find the area of the


shadow of the top of a table 4 ft. high, 9 ft. long, 5 ft. broad.
are similar triangles; AD,
are altitudes;
7. ABC,

6.

XK
CA = 13, AD = 12, XY = 5 find XK and
the ratio of the areas of AS ABC, XYZ.
A triangle ABC is divided by a line HK parallel to BC into
two parts AHK, HKCB of areas 9 sq. cms., 16 sq. cms.
XYZ
AB = 15, BC -

8.

14,

BC-7
9.

is

cms.; find HK.

the mid-point of the side AB of a square ABCD


at O ; AB = 3" ; find the area of EBCO.

AC

cuts

ED
10.

ABC

is

A such

such that
cuts

AB

Z.

in

that

DBC =
E

Z.

AB = AC = 2BC; D is a point on AC
BAC a line through D parallel to BC
;

find the ratio of the areas

AABC ABCD
:

ABED AEDA.
:

11.

in a supply pipe of diameter 1 ft. comes out through a


tap $" in diameter in the pipe it is moving at 5* a second ;
with what velocity does it come out of the tap ?

Water

3 to gild a sphere of radius 3


cost to gild a sphere of radius 4 ft. 1

12.

If it costs

13.

Two

hot-water cans are the same shape

ft.,

what

the smaller

is

will it

9* high

CONCISE GEOMETRY

128

and holds a quart


it

14.

hold

the larger

is

15* high

how much

will

How many

times can a cylindrical tumbler


high and 3* in
filled from a cylindrical cask 40* high and 30*
in diameter 1

diameter be

15.

A metal

16.

17.

Two models of

sphere, radius 3*, weighs 8 Ib. ; find the weight of a


sphere of the same metal 1' in radius.
cylindrical tin 5* high holds | Ib. of tobacco ; how much will

a tin of the same shape 8* high hold ?


the same statue are made of the same material

one

is

what

3* high and weighs 8 oz.

the other weighs 4

Ib.

is its

height 1
lodger pays 8 pence for a scuttle of coal, the scuttle being
20" deep ; what would he pay if the scuttle was the same

18.

19.

20.

Two

shape and 2 feet deep.


tap can fill half of a spherical vessel, radius 1| feet, in 2
minutes; how long will two similar taps take to fill onequarter of a spherical vessel of radius 4 feet ?
leaden cylinders of equal lengths and diameters 3*, 4
and recast as a single cylinder of the same length

are melted

what

is its

diameter ?

AB, CD
CD = 4 cms.,
QDR are

21. In the given figure, not drawn to scale, the lines


bisect each other at right angles ; AB
6 cms.,
are
arcs
of
RBS
of
circles
1
radii
cm. ; PCS,
PAQ,

3J cms., touching the former arcs.


Construct a similar figure in which the length of the line
corresponding to AB is 9 cms.
arcs of circles of radii

The area
is

of the

first figure is

approximately 18

the area of the enlarged figure

sq. cms.,

what

ABEAS AND VOLUMES

129

is rotated about AB to form


an egg-shaped solid, its volume Is approximately 48 c.c. ;
what is the volume of the solid obtained similarly from the

If in the given figure, tho curve

enlarged figure
22.

The

sides of

AABC

are trisected as in the figure

PQRSXY

the area of

=--

A ABC.

prove that

X/\.A

Fro. 136.

AS ABC, XYZ, Z.BAC=:


AABC = AB AC
that ^
A XYZ XY.XZ

23. If in the

prove
F
24.

Two

4.1

Z.YXZ,

4.

lines

OCD

CAB,

meet a

circle at A, B,

C, D, prove that

A PAD

25.

AD
\yhat result is obtained by
J making
8 B coAOBC BC 2
incide with A ?
H, K are any points on the sides AB, AC of AABC, prove
AAHK = AH AK
that ^
AABC AB AC
In AABC, Z.BAC = 90 and AD is an altitude
AB* = BD
prove
AC 2 DC
ABCD is a parallelogram P, Q are the mid-points of CB, CD
.

4.1

4.

26.

--

27.

28.

prove

Any

AAPQ = f

circles

parallelogram

through

B,

ABCD.

cuts

AB,

AC

at

D,

E;

an altitude;

DE

prove

AADE^DE
AABC~BC
29.

In

'

AABC, Z.BAC = 90
,.

perpendicular from
30.

AP

is

a chord and

tangents at A,

AB

~
D
is

P meet

and
.

to

*
AB

AD
;

is

prove

a
at

prove

the

5.

diameter of a circle, centre


;

is

BE = BA 2

-=

jj-

the

CONCISE GEOMETRY

130
31.

ABC
P,

32.

is

an equilateral triangle;

Q;

if

ZPAQ- 120,

BC

~
PR

prove

is

produced each way to

AP 2
j^-

AABC, Z BAG = 90; BCX, CAY, ABZ

In

triangles with X, Y,

are

corresponding points; prove

similar

A CAY

-f A ABZ -= A BCX.

33.

A room

34.

If

is lighted by a single electric bulb in tflie ceiling; a


table with level top is moved about in the room ; prove that

the area of the shadow of the top on the floor does not alter.
,r ins. is the
length of some definite dimension in a figure of

given shape,

its

different sizes.

diagonal x\

area^ta? 2

Find k

(iii)

circle,

sq. ins.

x;

(vii)

35. If

Ic

is

constant for

(i) sqnare,iside #; (ii) square,


radius x\ (iv) circle, perimeter x

-,

(v) equilateral triangle, side

radius

where

for

x\

surface of cube, side

(vi) regular
;

(viii)

hexagon, side

surface of sphere,

jr.

ins. is

the length of some definite dimension in a figure


its volume = Avr 8 cu. ins. where k is constant

of given sha}>e,

for different sizes.

Find

/<

for (i) cube,

edge

jc

(ii)

cube,

diagonal x (iii) sphere, diameter x ; (iv) sphere, equator x ;


(v) the greatest circular cylinder that can be cut from a cube,
;

edge x
(vii)

;
(vi) circular cone, vertical
regular tetrahedron, edge x.

angle 90, height x

BISECTOR OF VERTICAL ANGLE OF A TRIANGLE

131

THE BISECTOR OF THE VERTICAL ANGLE


OF A TRIANGLE
THEOREM 60
(i)

a triangle ; if the line bisecting /. BAG (internally or


externally) cuts BC, or BC produced at D,

ABC

is

,,

then
(ii)

If

is

BA
si-

AC

BD
DC

BC

a point on the base BC, or

triangle

ABC

such that

or externally /.

AC

DC

then

produced of the

AD

bisects internally

BAG.
A

DC
FIG. 137(1).

C
FIG. 137(2).

CONCISE GEOMETRY

132

THE BISECTOR OF THE VERTICAL ANGLE


OF A TRIANGLE
EXERCISE XXV
1.

AABC, AB = 6

In

internal

BC

cms.,

ami external

produced at P,

Q;

BC = 5
find

CA = 4

cms.;

the

BC

and

BP and BQ and show

that

cms.,

bisectors

of

Z.BAC

cut

L+ JU1.
BC*

BP

2.

BQ
AABC, AB = 4", BC = 3* CA-5"; the bisector of ZACB
find CD.
cuts AB at D
In AABC, AB=12, BC = 15, CA = 8; P is a point on BC
such that BP = 9 prove AP bisects /. BAG if the external
bisector of /. BAC cuts BC produced at Q, and if D is the
2
mid-point of BC, prove that DP DQ = DC
The internal and external bisectors of /. BAC meet BC and
BC produced at P, Q BP = 5, PC = 3 find CQ.
ABCD is a rectangular sheet of paper AB = 4*, BC = 3"
the edge BC is folded along BD and the corner is then cut
In

3.

4.

5.

along the crease

off
6.

In

AABC, AB = 6*, AC = 4";


BE at O; the

what

The

is

the area of

internal

BC
8.

AX

9.

ABCD

A AOB

and external

produced at
F

find the area of the remainder.

the median

7.

the bisector of

area of

AABC

/.BAG meets
is

sq.

in.;

BC

and

bisectors of

P,'*': prove
F

Z.BAC

cut

oo RO
= ~-?.
PC CQ

is a median of AABC; the bisectors of /.a


AXB,
meet AB, AC at H, K ; prove HK is parallel to BC.

at

K;

AXC

a parallelogram ; the bisector of /. BAD meets BD


the bisector of Z.AB'C meets AC at L; prove LK is

is

parallel to

AB.

BISECTOR OF VERTICAL ANGLE OF A TRIANGLE


The tangent

10.

OB

The

11.

is

bisector of

centres are

HK

prove

A of a circle, centre O, meets a radius


the foot of the perpendicular from A to OB ;

at a point

T D

at

is

Z.BAC

BC

cuts

at

drawn through D and


parallel to BC.

A ABC;

13.

AHB cut BC, CA, AB


BX CY
AZ
-__ =
XC YA ZB
Two lines OAB, OCD meet a circle at A,

the

inside

any point

CA

8A,

the

BHC, CHA,

with B,

circles

[cut

12.

is

133

bisectors

at X,

at H,

of

as

/.s

Z; prove

Y,

___

\f

ZAOC

of

The

14.

N/

cuts

bisector of

AC,

BD

/.BAC

at H,

at

the bisector

=
^J
KB
HC ?^.

K; prove

BC

cuts

B, C,

D; the

circle

through

A, B, D cuts AC at P ; the circle through A, C, D cuts AB


at
;
prove BQ = CP.
15. Two circles, centres A, B, touch at O; any line parallel to
AB cuts the circles at P,
respectively; AP and BQ are

produced to meet at

A straight

16.

The tangent

1 7.

prove

line cuts four lines

^POR-90

if

and

OR

at a point

produced at

O;

OK

bisects

bisects /. AKB,

OP, OQ, OR,

on a

18.

circle cuts

ABCD

19.

PQ when
TP, TQ at

a quadrilateral;

at L,

altitude; the bisector

-.
K; prove ^t = KA
LD

the bisectors of Z.s DAB,

if

meet on DB, prove that the bisectors of /.s ABC,


meet on AC.

Two

20.

circles

touch internally at

touches the smaller at


.

If

is

is

^=

a chord

/.TOP meets

the bisector of

/.ABC meets AD, AC

of

at P, Q, R,

Z.QOS, prove

2
2
X,
prove TX = TY = PX QY.
In A ABC, Z. BAC = 90 and AD is an

OS

the in-centre of

AIAB + AC

prove

a chord

PQ

QP = PR
OQ RQ

A ABC, and if Al

DCB
ADC

of the larger

meets

BC

at D, prove

CONCISE GEOMETRY

134
22*.

The

internal

X,

Y; prove

and external bisectors

^XPY = 90.

if

varies so that

is

circle.

is

PB

If A,

of

Z.APB meet AB

constant, prove that the locus of

\Apolloniu3

circle.]

bisectors of /. BAG meet BC and


BC produced at D, E, prove DE 2 = EB EC - DB DC.
ABC is a triangle such that AB4-AC = 2BC the bisector of
L BAG meets BC at D prove AD 2 = 3BD DC.

23*. If the internal

and external

24*.

at

are fixed points and

EXAMPLES ON THE CONSTRUCTIONS


OF BOOK 1
USE OF INSTRUMENTS
EXERCISE XXVI

USE OF RULER, DIVIDERS, AND PROTRACTOR


1.

Measure in inches and cms. the

lines a, 6,

c,

d.

FIG. 138(1).
2.

Draw a

3.

Draw a

4.

Draw a

straight line across your sheet of paper arid mark off


by eye lengths of 4 cms., 7 cms., 2 inches; then measure
them and write down your errors.
line and cut off from it a length of 5" ; measure
cms. and find the number of cms. in 1 inch.

it

5.

AD.

in

and cut off from it a length of 10 cms. ; measure


and hence express 1 cm. in inches.
138(2), measure in cms. the lengths of AC, BD, BC,
What are the values of (i) AC + BD ; (ii) AD + BC.

line

in inches

In Fig.

it

B
FIG. 138(2).
135

136
6.

CONCISE GEOMETRY

Measure in inches and cms. the length of


1" = 2'54 cms. approx., find

how

far

this page.

Taking

your measurements

agree with each other.


a straight line across your paper
mark the middle
and
measure the two parts.
How far is the
point by eye

7.

Draw

8.

Draw a

you have marked from the

point

real mid-point of the line

straight line across your paper and divide


into three equal parts
measure the three parts.

it

by eye

Rei>eat ex. 8, dividing the line into four equal parts.


10. Draw a straight line across your paper and use your dividers
9.

(i)

to bisect

it

(ii)

to trisect

it.

line AB produced beyond


an obstacle which obstructs the view. C is one of the points
required, perform the construction and verify it.

11. It is required to obtain points

on a

FIG. 139.

12.

Measure the angles

a, b y c, d.

FIG. 140.

13.

Use your protractor to draw angles


(iv) 124, (v) 220, (vi) 300.

of

(i)

30,

(ii)

90,

(iii)

48,

137

USE OF INSTRUMENTS
14.

Measure the angles

a, 6,

c,

d and

write

down

their sum.

FIG. 141.

15.

Measure the angles

a, b

and write down

FIG.

16

Measure the angles


them ?

a,

6,

sum.

14'2.

y.

What do you

FKJ. 143.

17.

their

Measure the angles AOB, BOG, AOC.

notice about

CONCISE GEOMETRY

138
18.

Measure the three angles of the triangle


their sum.

ABC

and write down

FIG. 145.

19.

Measure the three angles of the triangle


their sum.

DEF and

write

down

FIG. 146.

20.

Without measurement, say which is the larger of the angles,


a in Fig. 147 or b in Fig. 148, and roughly by how much.

FIG. 147.

eye (with a ruler) angles of 15, 30, 60, 110, 160.


Measure them and write down your errors.
22. Without measurement state whether the angles a, 6, c, d, e in
Fig. 148 are acute or obtuse or reflex.
21.

Draw by

FIG. 148.

23.

Draw an

angle

ABC

Measure Z.ABD.

equal to

108; produce CB

to

D.

USE OF INSTRUMENTS
24.

Draw an

angle

AOB

equal to 82

139

produce AO, BO, to C, D.

Measure /.COD.
25.

Draw any

five-sided figure

ABODE

Measure each of the


and write down their sum.

DE, EA.

26.

27.

Draw any

triangle ABO;
ZCBA, /.CAB, Z.ACD.
,/ ACD?

Draw a
of

and produce AB, BO, CD,


formed

five exterior angles so

BO to D.
Z.CBA+Z.CAB

produce
Is

Measure
equal

to

149; find by measurement the values

figure like Fig.

Z.ABC+ Z.ADC+ /.BAD and /.BCD.

28. Enlarge

/.DAB = 90= Z.CBA.

points F, G, H, K, L, M,

them

to

AB = 8

150, making^

Fig.

make with

130, 150.

AD = BO = 2

cms.,
the mid-point of AB.
Mark
on CD such that the lines joining

cms.,

is

OB

angles of 30, 50, 70, 90, 110,


Measure in cms. FG, GH, HK.

FIG. 150.

USB OF COMPASSES
29.

Draw a

circle,

30.

O; draw any diameter AB; take any


R on the circumference. Measure /. s APB,

centre

three points P, Q,

AQB, ARB.
Draw two circles

of radii 3 cms., 4 cms. so that their centres


are 5 cms. apart.
Draw their common chord, i.e. the line

CONCISE GEOMETRY

140
the

joining
length.

the

is

joining the centres

which they cut, and measure its


angle at which it cuts the line

at

points

What

31. Take two points A, B 3 cms. apart; construct two points P, Q


such that PA = PB -= 5 cms. = QA = QB.
32. Take a point P;
describe a circle of radius 4 cms.
construct a chord PQ of length
passing through P;

6 cms.
33.

Draw

a circle

34.

Draw

take four points A, B, C,

Measure (i) /. ACB and Z.ADB;


What do you notice 1

ABC

a large triangle

Z.

(ii)

(not isosceles)

and AC as diameters.
35. Take two points A, B 5 cms.

AB

on it.
and /.ADC.

in order

ABC

describe circles on

they meet on BC 1
Construct a point
apart.

Do

C such
=
cms.
Draw
with
circles
CB
centres
7
CA=Gcms,
What do you
B, C and radii 2, 3, 4 cms. respectively.

that

A,
notice about

36.

them 1
Take two points A, B 3 cms.
that

CA = CB = 6

Z.CBA, Z.ACB.

L CAB

Is

L ACB

37.

38.

Construct a point

CB

equal to

such

and measure Z.CAB,

Z.CBA?

Is

/.CAB

equal to twice
Draw a circle of radius 3 cms.
']

and place in it 6 chords each


of length 3 cms., end to end; what figure is obtained?
Measure the angle between two adjacent chords.
Draw a straight line AB ; construct a point C such that CA =

CB = AB.

Measure the angles of

A ABC.

a straight line AB and take any point P outside it.


Construct a point Q such that QA= PA and QB= PB. Join

39.

Draw

40.

Draw two

PQ
of

and

let it

cut

AB

at R.

circles of radii

Measure

/.

PRA.

3 cms., 4 cms. so that the part

the line joining their centres which

circles is 1

41.

apart.

Join CA,

cms.

lies

inside both

cm.

AB 5 cms. long; construct a point C so that


= 4 cms. Join CA, CB. Bisect with
cms., CB
Measure /. ACB and
dividers or by measurement AB at D.

Draw

a line

CA = 3
CD.

42.

Is

COCAS'!

Draw a line AB

3 cms. long

to pass through

and

B.

construct a circle of radius 4 cms.

141

USE OF INSTRUMENTS
43.

Take two points A, B 6 cms. apart. Construct 10 positions


of a point P (on either side of AB) such that PA-f PB= 10
cms.

PA = 3, PB = 7

(e.g.

positions

PA- 4, PB = 6,

or

freehand a curve through these positions.


the curve to pass through A or B ]
44.

All these

etc.).

on a smooth curve called an

lie

ellipse:

draw

Would you expect

circle, centre O, and take any point T outsile it; on


as diameter describe a circle cutting the first at P, Q.
Join TP,
and produce both. What do you notice about

Draw a

TO

TQ

these lines
45.

Draw a

such that
RS,

etc.,

^POQ

What

end to end, each equal to PQ.

so obtained
46.

centre O, of radius 3*5 cms. ; draw a chord PQ


72. Construct four other chords QR,

circle,

is

the figure

Draw two unequal


diameters PX,

PY

circles intersecting at P,

of the circles.

Join XY.

Q; draw

the

XY

pass

Does

through Q 1
47. Draw a circle, centre O, and take any six points A, B, C, D, E, F
Measure /.s ABF, ACF,
in order on the circumference.
notice
Do
AOF.
ADF, AEF,
any connection between
you

them?
48.

as centre and any radius (not


angle AOB ; with
too short), describe a circle cutting OA, OB at P,
with
;
radius
too
as centres and any
P,
(not
short), deMeasure /1AOR,
scribe two equal circles cutting at R.

Draw any

/LBOR.
This construction enables you to bisect a given angle.
49. Draw any straight line AB ; with A, B as centres and any
radius (not too short), describe two equal circles cutting at
Measure AR, RB
Join PQ and let it cut AB at R,
P, Q.

and ^ARP.
This construction enables you to draw the perpendicular
bisector

50.

of a given straight

Draw any
on

straight

line

line.

AB

and

take

any

point

it.

With

with P,

as centre, describe

two equal

as centres

and

any

circle cutting

AB

at P,

aify radius (not too short), describe

circles cutting at R.

Join CR.

Measure

/.

ACR.

CONCISE GEOMETRY

142

This construction enables you

pendicular

to

to

a given straight

draw a

line

straight line per-

from a given point on

the line.

51.

With C

straight line AB and take any point


as centre, describe any circle cutting

with

Draw any
P,

C outside it.
AB at P, Q

and any radius (not too short), describe


Join CR and let it cut AB
cutting at R.

as centres

two equal circles


at S.
Measure /. ASC.

This construction enables you to draw a straight line per-

pendicular

to

a given straight

line from

a given point

outside

tfie line.

52.

53.

straight line AB and take any point C outside it.


Take any point P on AB. Join CP and bisect it at Q.
With Q as centre and QC as radius, describe 'a circle, cutting
AB at R. Join CR. Measure Z.ARC.
This construction gives an alternate method to Ex. 51.
With any point O as centre, describe a circle draw any chord
PQ construct the perpendicular bisector of PQ. Does it

Draw any

pass through
54.

Draw

a triangle

ABC

(not

pendicular bisectors of

with

as centre and

AB

OA

the circle pass through

isosceles)

AC

and

construct
let

the

them meet

as radius, describe a circle.

B and

55. In Fig. 151, without producing


perpendicular to AB.

per-

at

Does

AB, construct a

line

through

xC
A

B
FIG. 151.

56.

Draw a

AB
57.

58.

line

AB, construct a

line

through B perpendicular to

without producing AB.

Draw an

obtuse-angled triangle ABC; construct the


pendiculars from each vertex to the opposite side.
they concurrent ?

Draw a

circle and take four points A, B, C,


struct the perpendiculars XP, XQ,
to
What do you notice about P, Q, R ?

XR

per-

Are

X on it ; conBC, CA, AB.

143

USE OF INSTRUMENTS
59.

Draw

a circle of radius 3 cms. and take points A, B, C on


AB = 4 cms., AC = 5 cms. Measure Z.BAC:

such that

there more than one answer

it
is

and take any two points C, D outside


it ; construct a point P on AB such that PC= PD.
61. Draw any triangle (not isosceles) and construct the bisectors
of its three angles.
What do you notice about them ?

AB

60.

Draw

62.

Draw any triangle ABC; construct


ABC, ACB and let them meet at

a line

the bisectors
I.

of

/.s

Construct the peras centre and IX as

With
pendicular IX from I to BC.
What do you notice about this
radius, describe a circle.
I

circle

63.

Draw two
bisectors

lines

BP,

BQ

ABC, BD,
of

ZABD,

cutting

at

B;

64. Construct (without using a protractor) angles of


(iii)

105,

(iv)

the

construct

Z.CBD; measure L PBQ.


(i)

30,

(ii)

45,

255.
and take any

three points A, B, C on it
the
construct
perpendicular bisector of BC and
(AB:4=AC);
the bisector of /. BAG and produce them to meet.
What

65.

Draw

66.

Draw an

a circle

do you notice about

their point of intersection ?


obtuse angle and construct lines dividing

it

into

four equal angles.


67. Draw a triangle ABC (not isosceles) ; construct a point P on
BC such that the perpendiculars from P to AB and AC are
equal.
68.

69.

Draw
Draw

a right angle and construct the lines trisecting it.


a line PQ (see Fig. 152), cutting two other lines AB, CD
at P, Q; the bisectors of Z.s APQ, CQP meet at H; the
bisectors of

Z.s

when produced
and

CD

BPQ,

DQP

meet at K; verify that

passes through the point of intersection of


and bisects the angle between them.

HK
AB

144
70.

CONCISE GEOMETRY

Copy

the following figures 153-167 on any convenient scale.


154

153

155

166

165

156

167

FIGS. 153-167

USE OF SET SQUARES


71.

and take three points P, Q, R outside it use


draw lines through P, Q, R parallel to AB.
Draw a line AB and take three points P, Q, R outside it use
set squares to draw lineo through P, Q, R perpendicular
Draw, a

line

AB

set squares to

72.

to

AB.

DRAWING TO SCALE
73.

74.

75.

145

line AB and ta.e <* point C on it


use set squares to
draw a line through C perpendicular to AB.
Draw a line AB and take a point P outside it use set squares
to draw two lines PQ, PR making angles of 60 with AB.
Draw a triangle ABC and use set squares to draw its three
altitudes (i.e. perpendiculars from each corner to the opposite

Draw a

side).

76.

Draw

ABC
ABCD

parallelogram
77.

Use

and use
measure

a triangle

set squares to

complete the

its sides.

draw a four-sided figure having its opposite


measure the
and one angle a right angle

set squares to

sides parallel

diagonals.
78.

Draw

a triangle

ABC

draw a
measure AK, KC.

squares to
79.

Draw any

angle

(not isosceles)

line

BAC

HK

bisect

parallel to

and cut

AB

off

BC

AB
to

at

equal to

use set

AC

meet

AC

at

use set

squares to construct bisector of /. BAC.


80. Use set squares to draw a right angle, and use them to trisect it.
81. Draw a line AB and divide it into five equal parts as follows
five equal parts
and cut oft' from
draw any other line
:

AC
AC
AP PQ> Q R RS ST J "1 BT through P, Q, R, S draw
lines parallel to TB
these cut AB at the required points.
>

>

DRAWING TO SCALE
EXEBCISE XXVII

FIG. 168.

FIG. 169.

OA

In Fig. 168, if
is horizontal,
DEFINITIONS.
(i)
called the angle of elevation of B as viewed from O.

In Fig. 169, if QH is horizontal,


angle of depression of R as viewed from Q.
(ii)

1.

courtyard

is

80

feet long

and 50

feet

distance between two opposite corners

10

Z.HQR

is

Z.AOB

is

called the

wide ; what

is

the

CONCISE GEOMETRY

146
2.

gun whose range is 5000 yards is in position at a point


3500 yards from a straight railway line; what length of
the line can

3.

4.

The ends

command 1

it

5 feet long, is resting against a vertical wall ; the


ladder,
foot of the ladder is 6 feet from the wall ; how high up the
1

wall does

it

reach ?

10 feet long, are fastened to two nails


15 feet above the ground; the nails are

of a cord,

each of which

is

5 feet apart ; a weight is attached to the mid-point of the


cord how high is it above the ground 1
:

5.

straight passage runs from A tq B, then turns through an


angle of 70 and runs on to C ; if AB is 80 yards and BC
is

100 yards, what distance


from A to C 1

is

saved by having a passage

direct
6.

A man
takes

rows due north at 4 miles an hour, and the current


north-east at 5 miles an hour ; how far is he from

him

his starting-point after


7.

A man

starts

from

20 minutes 1

A and

walks 2 miles due south to B, then

3 miles south-west to C, then 1 mile west to


the direction and distance of D from A ?
8.

Southampton

9.

what

is

12 miles S.S.W. of Winchester; Romsey is


32 S. of Winchester. Find the distance and

Romsey from Southampton.

An

aeroplane points due north and flies at GO miles an hour ;


the wind carries it S.W. at 15 miles an hour.
What is its
position ten minutes after leaving the aerodrome

10.

is

10 miles W.
bearing of

Andover

is

12 miles from Winchester and

15 miles

from

Salisbury ; Salisbury is 20 miles W. of Winchester. [Andover


is north of the Salisbury- Winchester
Find the bearing
line.]

Andover from Salisbury.


is 42 miles from Dorchester and 64 miles from Bristol
Bristol is 55 miles due north of Dorchester; Barnstable is
33 miles N.E. of Exeter. What is the distance and bearing
of Barnstable from Dorchester ?
A weight is slung by two ropes of lengths 12 feet,
16 feet, from two pegs 18 feet apart in a horizontal line.
of

11. Exeter

12.

What

is

the pegs)

the

depth of

the weight

below

the line of

DRAWING TO SCALE
13.

147

From two

points 500 yards apart on a straight road running


due north, the bearings of a house are found to bo N. 40 E.
and E. 20 S. find the shortest distance of the house from
;

the road.
14.

There are two paths inclined at an angle of 40 which lead


from a gate across a circular field one ruiif across the centre
:

of the field

and

120 yards long; what

is

the length of

is

the other ?
15.

path runs round the edge of aequare ploughed

you follow the path from A to


than if you walk straight across.
if

side of the field


16.

field

yot go 50 yards

What

is

ABOD

farther

the length of a

One end

of a string, 5 feet long, is fastened to a nail, and a


weight is attached to the other end; the weight swings
backwards and forwards through 15 each side of the

What

vertical.

positions
17.

is

the distance between

its

two extreme

At a distance

of

40 yards from a tower, the angle of elevation


35 ; find the height of the tower

of the top of the tower is


in feet.
18.

kite is flown at the

end of a string 120 yards long which

makes an angle of 65
19.

height of the kite.


is the elevation of the sun

What

casts a
20.

with the ground: find in feet the

shadow 20

feet long

when a

pole 12 feet high

fenced level road running due north suddenly turns due


with the result that the shadow of the fence is increased

east,

in breadth

the sun
21.

The

from 3

feet to 5 feet:

what

is

the bearing of

elevation of the top of a chimney is 20; from a place


it is 30 ; find its height in feet.

60 yards nearer,
22.

From
of

the top of a

cliff

a boat out at sea

from the

is

150
20

feet high, the angle of depression


;

what

is

the distance of the boat

in yards t
of
a tower 250 feet high, the angles of depression
From
the
23.
top
in a line with and at the same level as the
houses
of two
cliff

foot of the tower

apart in yards.

are 61

and 48.

Find their distance

CONCISE GEOMETRY

148

MISCELLANEOUS CONSTRUCTIONS

EXERCISE XXVIII
1.

Draw an

angle

AC

on AB,
2.

BAG

and a

such that

RS

is

line

PQ;

equal and

construct points R,
parallel to PQ.

a circle and construct points P, Q, R on it such that


take any other point X on the circle.
Measure XP, XQ, XR and verify that the longest of these

Draw

PQ = QR=RP;
equals the
3.

Draw an

Draw a

of the other two.

BAG

angle

such that
4.

sum

of

50

construct on AB,

Z.QPA = 90 and PQ = 4

cms.

AC

points P,

Measure AP.

4 cms., and take a point A at a distance


from the centre: construct a chord PQ passing
and bisected at A.

circle of radius

of 2*5 cms.

5.

through A
Draw a large quadrilateral ABCD, so that AB is not parallel
to CD ; construct a point P such that PA = PB and PC = PD

6.

Draw a

AB

line

and take a point

construct a circle with


S" apart.
7.

S.

as centre, cutting

AB

//

at

from AB
two points
;

its radius.

angle

"

Draw two
them

from C.

Measure PQ.

AC and

take a point P somewhere between


construct a line to pass through P and cut oft' equal
lines

lengths from
10.

distant 2

BAG of 70; construct a point P whose


Measure AP.
distances from AB, AC are 3 cms., 4 cms.
Draw a line AB and take a point C distant 2" from AB
construct two points P, Q each of which is 1* from AB
Draw an

and
9.

Measure

AB,

AB and AC.

AB, CD and take any point E between them.


Construct a line to pass through E and the (inaccessible)
point of intersection of AB, CD.
[Use the system of parallel

Draw two

lines

lines

shown

in Fig. 170.]

FIG. 170.

MISCELLANEOUS CONSTRUCTIONS
11.

Draw

a triangle

ABC; construct
/. BAG and let

the bisector of

12.

14.

parallel to

BA

produced at E.
Measure AE, AC.
Draw a circle and take two points A, B outside it. Construct
a circle to pass through A, B and have Its centre on tho first

When

circle.

13.

a line through

meet

149

is

this impossible

Draw a circle and take a


HAB, HOC, cutting the

point

it

01 tside it

circle at A, B, D,

draw two

lines

AD, BC,
; join
and produce them to meet at K. Construct a circle to pass
through H, A, D and a second circle to ;*ass through K, D, C.
Do these circles cut again at a point on HK 1
Construct five points in the same relative position to each
other as are A, B, C, D,

in Fig. 171.

FIG. 171.

15.

Take a

line

AB

and a point

C
C

outside

it

such that the

foot of the perpendicular from


to AB would be off the page.
Construct that portion of the perpendicular which comes on

the page.
16.

Take a

AB and a point C and suppose there is an obstacle


C and AB which a set square cannot move over
Construct a line through C parallel to AB.
172).

line

between
(see Fig.

B
FIG. 172.

17.

bisects a given angle,


folding, obtain a crease* which (i)
at
line
a given
right angles.
(ii) bisects

By

150
18.

CONCISE GEOMETRY

By

folding, obtain the perpendicular to a given line

given point outside it.


19. By folding, obtain an angle of
20.

from a

45.

Take a triangular sheet of paper and


which is equidistant from the three

find

by folding the point

corners.

CONSTRUCTION OF TRIANGLES,
PARALLELOGRAMS, ETC.
EXERCISE XXIX
1.

when possible^ the

Construct,

triangle

ABC

measurements, choosing your own unit.


different solutions, construct both

from the following


If there are two

a
Fio. 173.
(i)
(ii)

(iii)

(iv)

(v)
(vi)
(vii)
(viii)

(ix)

(x)
(xi)
(xii)
(xiii)

(xiv)

(xv)

a = 3, 6 = 4,
a = 3, 6 = 4,

= 5, measure A.
= 8, measure A.
a = 5, B = 30, C = 45, measure 6.
a = 4, A =48, B = 33, measure h.
a =7, A=110, B = 40, measure 6.
a = 5, B=125, C = 70, measure 6.
6 = 5, c = 7, C = 72, measure a.
6 = 6, c = 4, C = 40, measure a.
6 = 8, c=6, C=65, measure a,
A = 40, B = 60, C = 80, measure a.
A = 50, B = 40, C = 70, measure a.
A =125, 6=7-3, c = 5'4, measure a.
c

A = 90, a= 11-2, 6=7-3, measure


a = 6 = 6*9, A = 50, measure c.

c.

a =26, 0=^-, measure A.


Jtt

2.

Draw two unequal


AB,

BC,

CD,

parallelogram.

AC, BD bisecting each other; join


ABCD is a
and measure them.

lines

DA

CONSTRUCTION OF TRIANGLES
3.

Draw two
AB,

lines

equal

CD,

BC,

BD

AC,

DA;

each other; join


ABCD is a

bisecting

measure

151

/.ABC.

rectangle.
4.

Draw two unequal

lines

angles; join AB, BC,


is a rhombus.
5.

Draw two
angles

ABCD
f>.

bisecting each other at right

and Treasure them.

Af>CC

BD bisecting each ot^er


DA measure AB, BC,

at right

AB, BC, CD,

/.

ABC.

a square.

is

Draw two unequal perpendicular lines AC, BD such that AC


bisects BD; join AB, BC, CD, DA and measure them.

ABCD
7.

equal lines AC,

join

BD
DA
CD,
AC,

is

kite.

angle of 57 and cut off AB, AC from the arms of


the angle so that AB = 5 cms., AC = 8 cms. ; construct a
=
AB. What sort of a
point D such that BD AC and

Draw an

CD^

quadrilateral
8.

9.

is

ABCD

Construct a parallelogram ABCD, given AB = 7 cms.,


10 cms., BD = 8 cms. ; measure BC, CD.

AC =

Construct an isosceles triangle with a base of 6 cms. and a


vertical angle of

70

measure

its sides.

rhombus ABCD, given


measure /.BAD.

10. Construct a

11. Construct

5 cms.

AB- 5

cms.,

AC~G

cms.

an isosceles triangle of base 4*6 cms. and height


measure its vertical angle.

12. Construct the quadrilateral

AD = DC = 5

cms., /_

ABCD,

ABC =

20

given

AB=BC=3

measure

/.

cms.,

ADC.

rhombus ABCD, given AC = 6 cms., BD = 9 cms. ;


measure AB.
Construct the rhombus ABCD, given L ABC = 40, BD = 7
cms. measure AC.
Construct a rectangle ABCD, given BD = 8 cms. and that AC

13. Construct the

14.

15.

makes an angle
16. Construct a

such that

of 54

with

17. Construct the quadrilateral


(i)

(ii)

BD

measure AB, BC.

sides
trapezium ABCD with AB, CD its parallel
AB - 8, BC - 4, CD - 3, AD - 2 measure L BAD.
;

ABCD,

given that

AB = 4, BC-4-5, CD -3, /.ABC -80, Z.BCD =


110; measure AD;
AB-5, AC = 6, AD = 4, BD-7, CD = 3 measure BC,
;

CONCISE GEOMETRY

152

(iv)

ABC = 70, L BCD = 95, ^CDA=105, AB = 5,


AD = 4 measure BC.
AB = 5, BC = 6, CD = 3, DA=4'S, Z.ADC=100;

(v)

AB = 5,

(iii)

measure /.ABC.
/.

CAB = 35,

/.ADB = 54

,/

ABD = 47,

Z.ACB-65

measure CD.

ABC, given that


= 13 ; measure A.
c=
a-f &= 11, +
16, c4-a
=
=
=
A B 25, C 55, c 7 measure a.
A B C = 1 2 3, a = 3 j measure c.
A + B = ]18, B + C = 96, a==7; measure c.

18. Construct the triangle


1>

(i)
(ii)

(iii)

(iv)

19. Construct

an equilateral triangle

BC given by BD = 3

ABC

such that

if

DAC = 40

is

a point

measure BC.
20. Construct a square having one diagonal 5 cms. ; measure its

on

cms., then

side.

21.

AD
L

22.

23.

24.

is

an altitude of the triangle

ABC = 55,

ABC;

ACB = 65, construct

AD = 4

given

A ABC;

cms.,

measure BC.

is a median of the triangle ABC; given AB = 4 cms.,


AC = 7 cms., AE=4*5 cms., construct A ABC; measure BC.
AD is an altitude of the triangle ABC; given AB = 6 cms.
AD = 4 cms., /.ACB = 68, construct A ABC; measure BC.
AD is an altitude of A ABC; AD = 4 cms., L BAG = 75,

AE

/.ABC = 50,
25.

/.

construct

A ABC;

measure BC.

The

distances between the opposite sides of a parallelogram


are 3 cms., 4 cms., and one angle is 70 ; construct the

parallelogram and measure one of the longer sides.


26. Construct a parallelogram of height 4 cms., having its diagonals
5 cms., 8 cms. in length measure one of the longer sides.
:

27. Construct

an equilateral triangle of height 4 cms.

its side.

28. Construct the triangle ABC, given that


a + J = 2c=14, A
measure a.
(i)

=70;

(ii)
(iii)

(iv)

(v)

a + 6 + c=20, A = 65, B = 70; measure


a = 10, 6-fc=13, A = 80; measure b.
a = 8, & + c=10, B = 35; measured.
a = 9, c-i = 4, B = 25; measure c.

(vi)

a= 9,

(vii)

a = 5,

- c= 2, A = 70
- B = 20
=
6
3, A

measure

measure

i.
c.

a.

measure

MISCELLANEOUS CONSTRUCTIONS
29. Construct an isosceles triangle of height 5 cms.

18 cms.
30.

Each

measure

153

and perimeter

its base.

an

of the base angles of

isosceles triangle exceeds thy

by 24 ; the base is 4 cms.


and measure its other sides.

vertical angle

triangle

II

MISCELLANEOUS CONSTRUCTIONS

construct the

II

EXERCISE XXX
1.

Given two points H,


construct a point

2.

on the same side of a given line AB,


that PH, PK make equal

P on AB such

angles with AB.


Given two points H,
(see

Fig.

174),

K on opposite sides of a given line CD,


construct a point P on CD such that

Z.HPC- L KPC.

*K

Fia. 174.

3.

4.

5.

6.

7.

Given a triangle ABC, construct a line passing through A


from which B and C are equidistant.
Given a triangle ABC, construct a line parallel to BC, cutting
AB, AC at H, K such that BH + CK= HK.
Given a square ABCD, construct points P, Q on BC, CD such
that APQ is an equilateral triangle.
Given a triangle ABC, construct a rhombus with two sides
along AB, AC and one vertex on BC.
Given two parallel lines AB, CD and a point P between them,
construct a line through P, cutting AB, CD at Q, R such
that

8.

QR

is

of given length.

Given a triangle ABC, construct a point which is equidistant


from B and C and also equidistant from the lines AB and

AC.

CONCISE GEOMETRY

154
9.

Given

position the internal bisectors of the angles of a


triangle and the position of one vortex, construct the
in

triangle.

and

measurement, find the height of a


each
regular tetrahedron,
edge of which is 2*.
11. A room is 20 feet long, 15 feet wide, 10 feet high ; a cord is

10.

By

construction

stretched from one corner of the floor to the opi>osite corner


by drawing and measurement the angle

of the ceiling, find

which the cord makes with the


12.

floor.

Construct a square such that the length of its diagonal


exceeds the length of its side by a given length.

EXAMPLES ON THE CONSTRUCTIONS


OF BOOK II
AREAS
EXERCISE XXXI
1.

Find the areas of the following figures, making any necessary


constructions and measurements
:

A ABC, given 6-5, = 4, A = 90.


Rectangle ABCD, given AB = 7, AC= 10.
A ABC, given a = 5, 6=6, c=7.
A ABC, given b = 5, c = 4, B = 90.
A ABC, given 6 = c= 10, a= 12.
A ABC, given a =6, B=130, C = 20.
= 8, AD = 6, Z.ABC -70.
Hgrani ABCD, given AB
<?

(i)
(ii)
(iii)

(iv)

(v)
(vi)
(vii)

A rhombus whose diagonals are 7, 8.


A trapezium ABCD, given AB = 5, BC = 6, CD = 9,
L BCD = 30, and AB parallel to DC.
Quad. ABCD, given AB = 3, BC=5, CD = 6, DA -4,

(viii)

(ix)

(x)

BD = 5.
2.

3.

Draw a

triangle

whose

sides are 5, 6, 8 cms.

area in three different ways.


Draw a triangle with sides 5,
isosceles

measure

triangle with base

6,

7 cms.,

and obtain

its

and construct an

6 cms. equal in area to

it

its sides.

4.

Construct a parallelogram of area 21 sq. cms. such that one


side is 6 cms., one angle is 50 ; measure the other side.

5.

Construct a parallelogram of area 15


6 cms. ; measure its acute angle.

Draw

a triangle with

sq.

cms. with sides 5 cms.,

and construct a
and having one side equal

sides 4, 5, 6 cms.,

parallelogram equal in area to


155

it

CONCISE GEOMETRY

156
to 4 cms.

and one angle equal

to

70;

measure the other

side.
7.

Construct a rhombus each side of which


area 15 sq. cms.

8.

is

5 cms. and of

acute angle.
;
Draw a parallelogram with sides 4 cms., 6 cms., and one angle
70; construct a parallelogram of equal area with sides

5 cms., 7 cms.
9.

measure

measure

its

its

acute angle.

Construct a parallelogram of area 20 sq. cms., with one side


5 cms., and one diagonal 7 cms. measure the other side.
;

10.

Draw

11.

Construct a parallelogram equal in area to a given rectangle

a triangle with sides 5, 6, 8 cms., and construct a triangle


of equal area with sides 5*5, 6*5 cms.; measure the third
side.

and having

its

sides of given length.

in area to a given triangle and


one
side
in
having
equal
length to a given line, and one angle
to
that
side
adjacent
equal to a given angle.

12. Construct a triangle equal

13.

Draw a quadrilateral ABCD such that AB = 6 cms., BC = 5 cms.,

CD = 4

equivalent

Find
14.

L ABC =110, L BCD = 95. Reduce


triangle with AB as base and its vertex

cms.,

to an
on BC.

it

its area.

Draw

a figure like Fig. 149, and reduce it to an equivalent


triangle having AB as base and its vertex on AD.

FIG. 149.

15.

Draw a

figure like Fig.

175 and reduce

triangle.

FIG. 175.

it

to

an equivalent

AREAS
16.

Given four points A, B, C,

P such
and

DP

is

as in Fig. 176, construct a point


and
are of equal area

ABPD

that the figures

i>erpendicular to

157

ABCD

AB.

C
FIG. 176.

17.

Given a parallelogram ABCD and a point O inside it, construct


a line through O which divides ABCD into two parts of
equal area.

18.

Given a triangle

ABC

and a point D on BC such that BD<


P on AC such that (i) ADPC =

jBC, construct a point


J
19.

A ABC,

(ii)

ADPC = f AABC.

Given a parallelogram ABCD, construct point P, Q on BC, CD


such that AP, AQ divide the parallelogram into three portions
of equal area.

Given a quadrilateral ABCD, construct a line through A which


divides the quadrilateral into two parts of equal area.
21. Given a quadrilateral, construct lines through one vertex which
20.

divide
If

and
Fig.

Use

if

into five parts of equal area.

it

ABCD

is

any parallelogram, and


drawn through P

lines are

if

is

parallel

any point on BD,


to AB, BC as in

the parallelograms AP, PC are of equal area.


this fact for the following construction
177,

Construct a parallelogram equal in area to and equiangular


to a given parallelogram and having one side of given length.

Fio. 177.

CONCISE GEOMETRY

158
23.

Given a triangle ABC, construct a point


the triangles

24.

GAB, GBC, GCA

inside

it

such that

are of equal area.

Given a quadrilateral ABCD, perform the following con-

BP

struction for a line

AC

at

(or

AD)

at

through

Bisect
(see Fig. 178).
meet
to
parallel to

bisecting

draw

OP

it

BD

CD

join BP.

SUBL

.IJSION

OF A LINE

EXERCISE XXXII
1.

Draw

2.

Draw

a line ABj;| divide into three equal parts without

measuring
a line

AP =

it.

AB and

construct a point

P on AB such

that

4.

3.

PB *
Draw a line AB and

4.

Divide a given line in the ratio 5

5.

Construct a diagonal scale which can be used for measuring

construct a point

on

AB

produced, such

-g-J:

3 both internally and

externally.

lengths to
6.

inch.

By using a diagonal scale, draw a line of length 2*73 inches


on this line as base construct an isosceles right-angled
:

7.

and measure its equal sides as accurately as possible.


a
Use
diagonal scale to measure the hypotenuse of a rightangled triangle whose sides are 2* and 3*.

8.

On

triangle

a scale of 6" to the mile, what length represents 2000


yards 1 Draw a scale showing hundreds of yards.

SUBDIVISION OF A LINE
9.

What
scale

is

the R.F.

distances
10.

The

[i.e.

2" to the mile

up

It.F. of

to

representative fraction] for a map of


Construct a scale for reading off

5000 yards, and

map

is 1

159

20,000

as small as

500 yards

express this in inches to the

mile and Construct a suitable SCF

ta 'o

read miles ana tenths

of miles.
lines AB, AC and a poii c P between them, construct
a line through P, cutting AB, AC at Q, R so that QP= PR.
12. Given two lines AB, AC and a point P between them, construct

11.

Given two

13.

P with its
P in the ratio 2:3.
Draw a triangle ABC such
a line through

extremitie

on AB,

AC

and divided

at

parallel to

2 cms.
14.

that

BC=^6

AC at
AH HB ?

BC, cutting AB,

What

is

the ratio

cms.
H,

construct a line

such that

HK =

Given a triangle ABC, construct a line parallel


AB, AC at H, K such that HK=BC.

to

BC, cutting

EXAMPLES ON THE CCNS1 RATIONS


OF BOOK III
CONSTRUCTION OF CIRC)

&S, ETC.

EXERCISE XXXIII
Use a

2.

coin to draw a circle, and construct its centre.


Given two points A, B and a line CD, construct a
pass through A and B and have its centre on CD.

3.

Draw a

1.

4.

AB

and construct a circle of radius


5 cms. to pass through A and B.
Draw two lineb AOB, COD intersecting at an angle of 80 ;

make

line

3 cms. long,

AO = 3 cms., OB = 4

cms.,

CO = 5

cms.,

OD = 2-4
Does

construct a circle to pass through A, B, C.

5.

circle to

through D ?
Construct two

common

circles of radii

chord

is

cms.

it

pass

4 cms., 5 cms., such that their

of length 6 cms.

Measure the distance

between their centres.


6.

Draw two lines OAB, OCD


make OA=2 cms., OB = 6

cms.,

OC

3 cms.,

construct a circle to pass through A, B, C.

7.

8.

AC

Does

it

pass

AB

line

line

pass through

to touch

other

and take a point

AB;

at a distance of 3 cms.

construct a circle of radius 4 cms. to

and touch AB.

AC

65

with each

construct a circle of radius 3 cms. to touch

AB and AC.

Draw two

D and

at B.

Draw a

from the
10.

OD = 4

through D ?
Given a circle and two points A, B inside it, construct a circle to
pass through A and B and have its centre on the given circle.
Given a point B 'on a given line ABC and a point D outside
the line, construct a circle to pass through

9.

40
cms.

intersecting at an angle of

lines

AB,

making an angle

of

CONCISE GEOMETRY

162
11.

Draw

touch the

first

Given a straight

line

its

centre

and

to pass through A, and to have


Is there more than one such circle t

circle

a radius of 2 cms.
12.

point A at a
construct a circle to

cms. and take a

a circle of radius 3

distance of 4 cms. from

and a

circle,

construct a circle of given

Is
radius to touch both the straight line and the circle.
If not, state the conditions under
this always possible ?
1

which
1 3.

Draw

it is

a line

impossible.
of length 6 cms.

AB

with A,

as centres

and

radii 3 cms., 2 cms. respectively, describe circles.


Construct
a circle to touch each of these circles and have a radius of

5 cms. Give all possible solutions.


internal or external.]
14. Draw a circle of radius 4*5 cms., and

[The contacts

may

be

draw a diameter AB;

construct a circle of radius 1-5 cm. to touch the circle and AB.

and a point A on the circle and a point B outside


construct a circle to pass through B and to touch
the given circle at A.
16. Draw a circle of radius 5 cms.; construct two circles of
15.

Given a
the

circle

circle,

radii 1'5 cm., 2*5 cms. touching

touching the
17.

Draw a

triangle

construct the
triangle
18.

Draw two
4 cms.

OB

each other externally and

first circle internally.

whose
four

and measure
lines

OA,

4 cms., and
which touch the sides of this

sides are of lengths 2, 3,


circles

their radii.

OB

such that

construct a circle touching

measure

AOB = 40, and OA =


OA at A and touching

Z.

its radius.

Given a triangle ABC, construct a circle to touch AB, AC


and have its centre on BC. Is there more than one solution ?
20. Inscribe a circle in a given sector of a circle,
[i.e. Given two
19.

OA, OB of a circle, construct a circle to touch OA, OB


and the arc AB.]
Given two radii OA, OB of a circle, construct points H, K on
OA, OB such that the circle on HK as diameter touches
the arc AB.
Given two points A, B and a point D on a line CDE, construct
two concentric circles one of which passes through A, B
and the other touches CE at D. When ia this impossible ?
radii

21.

22.

CONSTRUCTION OF CIRCLES, ETC.


23.

Given three points A, B, C, construct three

circles

163
with

points as centres so that each circle touches the other tw>.


Is there more than one solution ?
24.

lines OA, OB intersecting at an angle of 40;


construct a circle touching OA an. OB and auoh tua u tne
chord of contact is of length 4 cms. mea UTJ i+* radius.

Draw two

25. Inscribe a circle in a given

rhombus

2fi.

28.

ABC is a triangle such that BC - 6

ven two points A, B, 4 cms. apait, construct a circle to


pass through A and B and such that the tangents at A and
B include an angle of 100 ; measure 's radius.
27. Find by measurement the radius of the circle inscribed in
;

the triangle whose sides are of lengths

90
to

6, 7,

>

BA = 4

by measurement the radius

find

cms.,

cms.
cms.,

L ABC =

of the circle escribed

BC.

29.

Given two parallel lines and a point between them, construct


a circle to touch the given lines and pass through the

30.

Draw

given point.
a quadrilateral so that
cms.

Does

it

touch the fourth side

circle

CD

to

order are

4, 5, 7,

touch three of the

sides.

two given parallel lines: construct


to touch AB, CD and the given circle.

31. In Fig. 179,

its sides in

inscribe a circle in it

AB,

are

C
FIG. 179.

32.

Given two

parallel lines

AB,

CD and a circle between them,


CD and to touch and enclose

construct a circle to touch AB,

the given
33. Given
first,

two

circle.

circles,

on the
centres A, B, radii a, &, and a point
and
circle to touch the first circle at

construct a

Fig. 180 gives the construction


for the centre P of the required circle, if it touches both
D is found by making CD = b. Perform
circles externally.

also to touch the second.

16

CONCISE GEOMETRY
this construction

and construct

also the circle in the case

where the contacts are external with


circle

B.

How

would

circle touches circle

or externally

with

the constructed

if

A internally and circle B

either internally

Fm.
34.

circle A, internal

be situated

180.

an equilateral triangle ; AB = 4 cms. ; A, B are the


CA is produced
centres of two equal circles of radii 2*5 cms.
a
circle
first
circle
at
D.
Construct
to meet the
touching the
the
second circle
first circle internally at D and touching

ABC

is

State your construction.


externally.
35. Construct a circle to touch a given line

AB

and a given

circle

centre C, at a given point D.


Fig. 181 gives the construction for the centre P of the required circle if the contact is

Perform the construction and construct the case


where the contact is internal.

external.

Fio. ,181.

CONSTRUCTION OF CIRCLES, ETC.


Construct the Figs, in exs. 36-62

165

do not rub out any of your

construction lines.
36. Three arcs each of radius 3 cms.

and each Jth

of a complete

circumference.

B<~
FIG. 182.

37.

AB, BC, CD,

DE

are equal quadrants;

AE

fi

cms.

FIG. 183.

38.

AB, BC, CD, DE, EF, FG, GH, HA are alternately semicircles
and quadrants of equal radius ; XY= 10 cms.

y
FIG. 184.

39. Three arcs each of radius 3 cms. touching at A, B, C.

FIG. 185.

CONCISE GEOMETRY

166
40.

The

aides of the rectangle are 6 cms., 8 cms.

Fio. 186.

41.

The

radii of the arcs

AB, BC,

CA

are 3 '5 cms., 2 '5 cms.,

7 cms.

FIG. 187.

42.

The

radii of the circles are 1 cm., 2 cms., 2 cms., 3 cms., and


the centre of the smallest circle lies on the largest.

FIG. 188.

43.

AP,
is

AQ

are arcs of radii 4 cms.

perpendicular to

CD

PQ

is

of radius

and equals 3 cms.

B
Fio. 189.

8 cms.

AB

CONSTRUCTION OF CIRCLES,
44.

CD

AB, BC,

arc arcs of radii 3 cms.,

AD

ETC.

167

equals 7 cms. and

touches AB, DC.

45.

The

radii of the arcs

5 cms.,

DE = 6'5

BC
AE = 7

AB,

cms.,

are 3*5 cms., 1*2 cm.,

CD

cms.

90

90

C
FIG. 191.

46.

AB,

AD

are arcs of radii 6 cms.

axis of symmetry.

AC

equals 6 cms. and

is

an

CONCISE GEOMETRY

168
47.

BC is a quadrant

the radii of arcs AB, BC,

CA are 4,

2,

3 cms.

Fm.
48.

AF is an
AF = 8

axis of
cms.,

symmetry; AB, BC,

EG = 6

The

radii of the arcs

DE

are equal quadrants;

cms.

49.

193.

ABC,

ADC

are 3, 5'5 cms.

Construct the figure and inscribe in


1*5
cm.
radius

5 cms.

it

chord

AC =

a circle of

50.

CE

is

an axis of symmetry

3 cms.; the
CE = 5 cms.

centre

of

AB,

AB

BC

lies

F
FIG. 196.

are arcs each of radius

on AD.

AD = 10

cms.,

CONSTRUCTION OF CIRCLES,
51.

AB

is

an arc

radius

52.

53.

of radius 3 cms.

cm.

chord

DA

BC, CD,

AE = chord EB = 3

AB, BC are semicircles, each


an angle of 120. The arc

ETC.

are arcs each of

cms.

of radius 2 cms.

AC

touches

DE are arcs each of radius 2 cms.


AE BC = CD = 4 cms. AE = 6 cms.

AB,

169

ar<

intersecting at

AB, CB.

with their centres on

FIG. 199.

54.

CD

is

AE,

an axis of symmetry;

EC

AB = 9'5

cms.,

CD = 3 '5

are arcs of radii 2, 10 '5 cms. respectively.

cms.

CONCISE GEOMETRY

170
55.

AB is a quadrant of radius 2 -5 cms. with


AC = 7 cms. The arc BC touches AB at

its

centre on

AC;

B.

Fiu. 201.

56.

ABCD

a square of side 2 cms.

is

with C,

BE, EF are circular arcs

as centres respectively.

FIG. 202.

57.

AB

is

an axis of symmetry; PAQ is a semicircle of radius


RBS is an arc of radius 1 cin. ; AB = 7 cms. The

2 cms.

arcs PR,

QS

are tangential at each end.

58.

AB = 3'5

cms.,

AC = 6

cms.,

Z.BAC = 90;

1*5 cm.

Bl

A
FIG. 204.

radius of arc

CP

is

171

CONSTRUCTION OF CIRCLES, ETC.


59.

AB=BC=3
angles of

60.

AB

is

cms.; tho arcs AB,

BC

cut the line

ABC

at

30.

a semicircle, radius 3 cms., centre O ; O T3 OQ are arcs


1 cm. ; the arcs AP, AB a-e tangential at A.
,

each of radius

Q/

FIG. 206.

207 is formed by parts of nine equal circles touching


where they meet ; AX, BY, CZ are each axes of symmetry ;

61. Fig.

the radius of each arc

is

1*5 cm.

FIG. 207.

62.

AB, CD, EF,

GH

radius 1 cm. and

CF,

HE

are the

diameters of semicircles each of

when produced form a square

are arcs each of radius 5 cms.

AD, BG,

CONCISE GEOMETRY

172

MISCELLANEOUS CONSTRUCTIONS

III

EXERCISE XXXIV
1.

Draw a

and construct a chord of the


Take a point A inside the circle but

circle of radius 3 cms.,

circle of length

5 cms.

not on the chord, and construct a chord of length 5 cms.


passing through A.

4.

Given a chord PQ of a given circle and a point R on PQ,


construct a chord through R equal to PQ.
Inscribe a regular hexagon in a given circle.
Inscribe an equilateral triangle in a given circle.

5.

A, B,

2.

3.

are three given points on a given circle ; construct a


circle equal to AB and parallel to the tangent

chord of the
at C.
6.

Draw

a circle radius 4 cms. and take a point 6 cms. from the


Construct the tangents from this point to the circle

centre.

and measure
7.

Draw

their lengths.

circle of radius

3 cms., and construct two tangents

which include an angle of 100.


8.

9.

a line AB of length 7 cms. ; construct a line AP such


that the perpendicular from B to A P is 5 cms.
Draw a circle, centre O, radius 4 cms. ; take a point A 6 cms.

Draw

from
P on

draw

AB

perpendicular to

AO

construct a point

AB

such that the tangent from P to the circle is of


5*5
cms. ; measure A P.
length
10. Draw a circle of radius 3 cms. and take a point 5 cms. from
construct a chord of the circle of length 4 cms.
;
which when produced passes through this point.
Draw a line AB of length 5 cms. and describe a circle with AB
the centre

11.

construct a point on AB produced such that


it to the circle is of length 3 cms.
12. Given a circle and a straight line, construct a point on the
as diameter

the tangent from

such that the tangents from


angle equal to a given angle.

line

it

to the circle contain

an

Circumscribe an equilateral triangle about a given circle.


14. On a line of length 5 cms., construct a segment of a circle

13.

containing an angle of 70

measure

its radius.

MISCELLANEOUS CONSTRUCTIONS
15.

On

173

III

a line of length 2 inches, construct a segment of a

circle

measure its radius.


containing an angle of 140
circle
of
a triangle whose angles
a
radius
3
inscribe
In
16.
cms.,
are 40, 65, 75 ; measure its longest side.
a rectangle of length 1 5", and
17. Inscribe in a circle of radius
;

measure
18.

its

breadth.

Circumscribe about a

circle of radius

2 cms. a triangle whose

angles are 50, 55, 75 ; measure its longest side.


19. Given three non-collinear points A, B, C, construct the tangent
at A to the circle which passes through A, B, C without
either
20.

drawing the

Draw two

circle or constructing its centre.

3 cms., with their centres

circles of radii 2 cms.,

6*5 cms. apart; construct their four common tangents.


21. Draw two circles of radii 2*5 cms., 3*5 cms., touching each

other

externally,

and

construct

their

common

exterior

tangents.

a line AB of length 6 cms. and construct a line PQ


such that the perpendiculars to it from A, B are of lengths
2 cms., 4 cms.

22.

Draw

23.

Draw two

circles of radii 2 cms., 3 cms., with their centres


6 cms. apart ; construct a chord of the larger circle of length
4 cms. which when produced touches the smaller circle.

24. Construct the triangle

90,

the altitude

ABC, given

AD = 2

cms.

that

that

ABC, given

26. Construct the triangle

ABC, given
angle BAC.

cms.

cms.,

BAC =

measure AB, AC.

25. Construct the triangle


55, the altitude AD

=4

BC = 6

BC = 5

Z.BAC =

cms.,

measure AB, AC.

BC

and the

27. Construct a triangle given its base and vertical angle


length of the median through the vertex.

and the

altitude

BE and

the

28. Construct a triangle

and the
29.

Draw a

the length of

ABC, given BC = 6

median BE = 5

cms.,

L BAG =52,

cms.

circle of radius 3 cms.,

and construct points A,

B,

on the circumference such that BC = 5 cms., BA + AC = 8*1


cms. ; measure BA and AC.
30. Draw a circle of radius 3 5 tcms. and inscribe in it a triangle
ABC such that BC=5'8 cms., BA- AC = 2 cms. measure
;

BA

and AC.

174

CONCISE GEOMETRY

ABC

31. Construct a triangle


given its perimeter, the angle
and the length of tha altitude AD.
32.

Draw any

33.

Draw two

34*.

circle and take two points A,


outside the circle; construct a point
that PC bisects Z.APB.

B on it and a point C
P on the circle such

lines which meet at a point off your paper


the bisector of the angle between them.

Draw any

triangle

ABC

BAC

(not right-angled).

construct

Construct a

such that

PQ passes through A, QR passes


square PQRS
through B, and PR cuts QS at C.
35*. Construct the quadrilateral A BCD, given that AD = 5 cms.,

BC = 4'6

cms.,

L ABD =

Z.

ACD = 55,

CBD = 43;

measure CD.
36*.

Draw any circle and take two


a point P on the circle such

37*.

Draw

points A, B on it ; construct
that chord PA equals twice

chord PB.
a circle of radius 3 cms., centre O, and take a point P
O ; construct a line through P,

at distance of 5 cms. from

R such that the segment QR contains


an angle of 70 ; measure /. OPQ.
38*. Draw two unequal circles intersecting at A, B ; construct a
line through A, cutting the circles at P, Q such that PA = AQ.
39*. Draw two unequal circles intersecting at A, B ; construct a
cutting the circle at Q,

line

through A, cutting the

circles at P,

such that

of given length.

40*. Circumscribe a square about a given quadrilateral.

PQ

is

EXAMPLES ON THE CONSTRUCTIONS


OF BOOK IV
PROPORTION AND SIMILAIl FIGURES
EXERCISE XXXV
1.

Construct and measure a fourth proportional to lines of length


4, 5, 6 cms.

2.

Construct and measure a third proportional to lines of length


5, 6 cms.

3.

Draw
Draw

4.

(ii)

5.

Draw

a line

a line

in the ratio 3

a line

ratio 3
6.

AB and divide it internally in the


AB and divide it externally (i) in

8.
9.

10.

3.

5:3;

AB and

divide

it

internally

and externally

in the

7.

Use a construction

x 7
- = --.

to solve

Find graphically the value of

5
2
(i)

-*

(ii)

'f

3*8

2'7.

y cms.

Construct a line of length


Draw a line AB and divide

Draw any
ratio

its

AB BC
:

ABC

it

line

PQ

perimeter equals

PQ

and

2:7:3.

in the ratio

and any

triangle

such that

11.

the ratio

5.

7.

ratio 2

construct a triangle
are in the

its sides

CA.

To construct the expressions

(i)

-, (ii)

proceed as follows

Draw two lines OH, OK (see Fig. 209).


From OH, cut off OA = a.
From OK, cut off OB = 6, OC = 0, OF-/, OG=#.
Join AF, draw

BX

parallel to FA, cutting

ab
--.
176

OH

at X, then

CONCISE GEOMETRY

176

Join XG, draw

QY
Use

CY

a^
^=

a ^c

~7*

</~~

fi

parallel to

this construction to find

and extend

it

to find

GX, cutting

(i)

at Y, then

'

*-

3 8
,

4'7

where

OH

(ii)-L*

*"?

4*i

a, 6,

c,

d,

ey

/,

Jl?_
x I'o
</,

h are

given lengths.

FIG. 209.

12. If a,

6, r,

are given numbers, construct,

by the method

of

ex. 11,
13.

14.

Kg. 209,

(i)

*;

(ii)

(iii)

g.

AB, AC and a point D between them, construct


a line through D, cutting AB, AC at P,' Q such that PD = DQ.
Draw a line ABCD; if AB = # cms., BC-y cms., CD = 2 cms.,

Given two

lines

construct a line of length xyz cms.


15.

Given a triangle ABC, construct a point P on BC such that


the lengths of the perpendiculars from P to AB and AC

16.

ABC

are in the ratio 2:3.

17.

is

an equilateral triangle of side 5 cms., construct a

point P inside it such that the perpendiculars from P to BC,


CA, AB are in the ratio 1:2:3. Measure AP.
Draw any triangle ABC, use the method indicated in Sig. 210
to construct a triangle XYZ similar to triangle ABC and

such that

XY=2AB.
A

_-

PROPORTION AND SIMILAR FIGURES

177

Given a quadrilateral ABCD, construct a similar quadrilateral


each side of which is
of the corresponding side of ABCD.
19. Given a triangle ABC and its median AD, construct a similar
triangle XYZ and its median XW, such that XW=$AD.
20. Construct an equilateral triangle s?oh that the length )f ohe
line joining one vertex to a point of trise^tiou of the opposite
side is 2" ; measure its side.
21. Using a protractor, construct a regular jxmtagon such that
the perpendicular from one corner to the opposite side is of
18.

length 7 cms. ; measure its side.


a square ABCD, given that the length of the
lino joining A to the mid-point of BC is 3"; measure

22. Construct

its side.

^BAC=54, /.ABC = 48,

23. Construct a triangle ABC, given


and the sum of the three medians

is

12 cms.

Measure AB.

24. Inscribe in a given triangle a triangle whose sides are parallel


to the sides of another given triangle.
25.

Given two radii OA, OB of a circle, centre O ; construct a square


such that one vertex lies on OA, one vertex on OB, and the
remaining vertices on the arc AB.

26. Inscribe a regular octagon in a square.


an equilateral triangle, one
27. Inscribe in a given triangle
side of which is perpendicular to BC.

ABC

28. Construct a circle to touch

two given

lines

and a given

circle,

centre O, radius a.
[Draw two lines parallel to the given
construct a circle to touch
lines at a distance a from them
:

these lines

and pass through O.

of the required circle.]

Its centre is the centre

from it; P is a
O
a point such that OQ = OP and
Z.POQ = 50. Construct the locus of Q. [The locus of Q
is obtained by revolving AB about O through 50.]
30. ABC is a given triangle ; P is a variable point on BC ; Q is
a point such that the triangles ABC, APQ are similar.
29.

Draw a

line

AB

and take a point

variable point on

31.

AB; Q

is

Construct the locus of Q.


[Use the idea of ex. 29.]
P moves
of
a
is
given shape ; A is a fixed point,
APQ
triangle
on a fixed circle ; construct the locus of Q. [Use the idea
of ex. 29.]

12

CONCISE GEOMETRY
32*. Given a triangle
33*.

P, Q
ABC

ABC

and a point

on BC, construct points


an equilateral triangle.

on AB, AC such that DPQ is


a straight rod whose ends A,

is

AB=6

perpendicular lines OX, OY;


Draw the position of the rod when

move along two

BC = 3

cms.,

cms,

maks

an angle of 30
with OX, and construct the direction in which B is moving
it

at this instant.

34*.

AB and BC are two equal rods hinged together at B the


end A is fixed and C is made to move along a fixed line AX
D is the mid-point of the rod BC construct the direction
in which D is moving when /. BAG = 45.
A piece of cardboard in the shape of a triangle ABC moves so
that AB and AC always touch two given fixed pins E, F
;

35*.

draw the triangle in any position and construct the direction


in which A is moving at that instant.

THE MEAN PROPORTIONAL


EXERCISE XXXVI

mean

proportional between 5 andjS

Construct a

2.

Construct a line of length ^/43 cms.

3.

Find graphically ^37.

4.

5.

6.

7.

measure

it.

[Don't take a mean


inaccurate
to
leads
this
and
1
between
drawing ; take
43,
4
= &'($.]
as
5
and
such
numbers closer together,
8*6, /-

2
Solve graphically the equation (x - 3) = 19.
Draw a rectangle of sides 4 cms., 7 cms., and construct a

square of equal area ; measure its^side.


Construct a square equal in area to an equilateral triangle of
side 5 cms. ; measure its side.

Construct a square equal in area to a quadrilateral ABCD


=
=
=
=
=
given AB BC 4, CD 6, DA 7, AC 6 cms. ; measure
its side.

8.

Draw

a line

AB

construct a point

P on AB such

that

AP = 1
AB J%
9.

Draw
area

centre

circle,

is

one-third of the

construct a concentric circle whose


first circle.

THE MEAN PROPORTIONAL

179

11.

Given a triangle ABC, construct a line parallel to BO, cutting


AB, AC at P, Q so that AAPQ = JAABC.
Given a quadrilateral ABCD, construct a similar quadrilateral

12.

Given a triangle ABC, construct

10.

with

its

area

f-

of the area of

AC CD.
rqv'uateral triangle of

ai<

equal area.
13.

line

14.

15.

whose lengths are a,


x 62
~ ^.
of length x cms. such that
2

Given three

linos

6, c

cms., construct a

a c
two equilateral triangles, construct an equilateral
triangle whose area is the sum of their areas.
Construct a circle to pass through two given points A, B and
touch a given line CD.
Use the method indicated in Fig. 211 and obtain two
Given

solutions.

/
~'

FIG. 211.

16.

circle and two points A,


point P on AB such that PA PB
the tangent from P to the circle.

Given a

outside

is

equal to the square of

it,

construct a

through two given points A, B and


to touch a given circle.
18. Given four points A, B, C, D in order on a straight line,
construct a point P^on BC such that PA PB = PC . PD.

17. Construct a circle to pass

y = 5, xy= 16.
OA = 6 cms., L AOB = 40
OA at A and intercepting on OB

19. Solve graphically the equations x


are two lines such that
20. OA,

OB

construct a circle touching


a length of 5 cms.
21. Construct a circle

to pass through a given point, touch a


given circle and have its centre on a given line.
22. Given three circles, each external to the others, construct a

CONCISE GEOMETRY

180

point such that the tangents from


of equal length.
23. Draw a circle of radius 5 cms.

from the centre


through

and take a point

construct a chord

such that

to the three circles are

it

PQ

3 cms.

of the circle passing

PA = |AQ.

MISCELLANEOUS CONSTRUCTIONS IV
EXERCISE XXXVII
1.

Draw a

line

AB;

if

AB

is

CD

construct a line

OPQ,

of length

of length x 2 inches.
are two given parallel lines,
2. AB,

cutting

inches, construct a line

and

AB,

O is any given point


CD at P, Q so that
;

AP CQ

is equal t;> a given ratio.


a given equilateral triangle of side 5 cms. ; construct
a line outside it such that the perpendiculars from A, B, C
:

3.

ABC

is

to the line are in the ratio


4.

5.

6.

7.

2:3:4 and

measure the

last.

Construct a triangle ABC, given /. BAG = 48,


and the median BE = 5 cms. ; measure AC.

z_BCA = 73,

Construct a triangle ABC, given L ABC =62,


and AB BC = 2 cms. ; measure BC.

^ACB = 75

Inscribe in a given triangle a rectangle having one side double


the other.

Draw

a triangle of sides 5, 6, 7 cms. and construct a square


of equal area; measure its side.
Check your result from
the formula

/s

(s

a) (s

b) (*

- c).

8.

Divide a square into three parts of equal area by lines parallel


to one diagonal.

9.

Given a triangle ABC, construct a line parallel to the bisector


of L BAC and bisecting the area of AABC.

10.

Given two

lines

AB

produced such that

and CD, construct a point P on

PA PB = CD 2
.

AB

REVISION PAPERS
BOOK

1.

It requires four complete turns of the handle to wind up a


bucket from the bottom of a well 24 feet deep.
Through

what angle must the handle be turned


5

to raise the bucket

feet.

2.

The angles

3.

ACB

of a triangle are in the ratio

1:3:5.

Find them.

ACY are equilateral triangles on


=
AB
sides
of
opposite
prove CX BY.
ABCD is a quadrilateral ADCX, BODY are parallelograms
prove that XY bisects AB.
is

a straight line

ABX,

4.

II
5.

If the reflex angle

6.

In

four times the acute angle

ABCD
DA
to

AOB,

is

find the angle be-

which bisect /.ABC and /.ACB.


of an isosceles triangle ABC is produced to

lines

BC
CD = CA,

The base
that

8.

is

A ABC, L BAG = 44, L ABC =112;

tween the
7.

AOB

ZAOB.

find

prove
a parallelogram

is

are produced to meet at

meet at R

so

Z.ABD = 2Z.ADB.

prove

the mid-point of

DP

and

CB

AB

CP

and

are produced

QR = CD.
Ill

9.

2LAOB = #; AO is produced to C; OP bisects Z.BOC; OQ


bisects /. AOB
calculate reflex angle POQ.
In AABC, L ABC = 35, Z.ACB = 75; the perpendiculars
from B, C to AC, AB cut at O. Find 21 BOC.
;

10.

181

182
11.

CONCISE GEOMETRY

BAG cuts BC at D through C a


DA to meet BA produced at P

The

bisector of the angle


line is drawn parallel to

prove
12.

AP = AC.

CAXY

ABC

are squares
is an acute-angled triangle; BANK,
outside the triangle ; prove that the acute angle between BH

CX

and

90 -

equals

Z.

BAC.

IV
13.

Find the sum of the

14.

The sum

15.

ABC

interior

angles of a 15-sided convex

polygon.
of one pair of angles of a triangle is 100, and the
difference of another pair is 60 ; prove that the triangle is
isosceles.
is

a triangle right-angled at

is

a point on

AB

such

Z.PCB=Z.PBC; prove Z.PCA= |Z_BPC.


a point inside an equilateral triangle ABC; CAP is an
equilateral triangle such that O and P are on opposite sides
that

16.

is

of

AB;

prove

BP = OC.

17. If a ship travels


alters

latitude
long. 2

18.

60.

Find her longitude if she starts (i) at lat. 0,


E. and steams 200 miles west ; (ii) at iat. 60 N.,
W. and steams 150 miles east.

long. 2
bisectors of

The

L BOC =135,
19.

due east or west one sea mile, her longitude


2 minutes if in
if on the equator, and

minute

Z.s

ACB of
BAC = 90.

ABC,

prove

/.

A ABC

meet at

O;

if

AABC, Z.ACB = 3/.ABC from AB a pait AD is cut off


=
equal to AC prove CD DB.
In AABC, AB = AC from any point P on AB a line is drawn
perpendicular to BC and meets CA produced in Q
prove
AP = AQ.
In

20.

VI
21.

O is a point outside a line ABCD such that OA = AB, OB = BC,


OC = CD; Z.BOC = a; calculate Z.OAD and Z.ODA in
terms of

x.

KEVISION PAPERS
22. In Fig.

144, page 137,

23.

ABCD

is

24.

ABCD

is

cutting

OQ

if

DA=-DB

a quadrilateral*
a parallelogram;

AC

at P,

bisects

prove

1P3

Z.AOC, pvo\a

DC;

</

BOC

/ BAC-f

prov*

DQ an two parallel
BQ p railel to DP.
BP,

lines

is

VII

AABC, Z.BAC=115, Z.BCA=20; AD is the perA to BC prove AD = DB.


In Fig. 212, AB is parallel to ED; prove that reflex Z.EDC
-reflex Z.ABC= /.BCD.

25. In

pendicular from

26.

27.

28.

ABC = Z. ADC = 90 prove that


the bisectors of /.a DAB, DCB are parallel.
In AABC, Z.ABC = 90, Z.ACB = 60; prove AC=2BC.

ABCD

is

a quadrilateral

Z.

VIII
29.

Two
if

lie

which

outside each other

YZ

cuts

AABC, AB = AC D is a point on AC such


prove /.DBC= Z.BAC.
The altitudes BD, CE of AABC meet at H
=
prove AB AC.

30. In

31.

AYZ

equilateral triangles ABC,


Z.CAY= 15, find the angle at
;

that

BC.

if

BD = BC

HB=HC,

CONCISE GEOMETRY

184
S

32. P, Q, R,

square

if

are points on the sides

PR

is

AB, BC, CD,

perpendicular to QS, prove

DA

of a

PR = QS.

IX
33. In Fig. 213, express

sr

in

terms of

a,

/>,

c.

FIG. 213.

any point on the bisector of /.BAG; DP, DQ are drawn


AB at P, Q ; prove DP =
parallel to AB, AC to meet AC,

34.

35.

ABC

is

DQ.
is

D,

ABCD
PQ

is

are points on

BC

such that

/.

BAD =

AB = AC.

if AD = AE, prove
a square; the bisector of Z.BCA cuts AB at P;
the i>erpendicular from P to AC ; prove AQ= PB.

Z.CAE;
36.

is

X*
37.

38.

39.

40.

ABCDEFGH is a regular octagon calculate the angle at which


AD cuts BF.
In A ABC, AD is perpendicular to BC and AP bisects ZLBAC
if /.ABOZ.ACB, prove ZABC- L ACB = 2 /. PAD.
ABCD is a straight line such that AB=BC=CD; BPQC is
a parallelogram; if BP=2BC, prove PD is perpendicular to
;

AQ.
The sides AB,

AC

of
AABC^are produced to D,
are lines parallel to the bisectors of /.s BCE,
= BC + HK.
BC in H, K: prove AB-f

E; AH,

CBD

AC

AK

meeting

REVISION PAPERS

XI*
41. In Fig. 214, express z in oerm.j of ^,

7>,

x, y.

FIG. 214.

42.

AB, BC, CD,

DE

are successive sides of a regular w-sided

polygon find the angle between AB and DE.


43. In
ABC, AB = AC ; BA is produced to E ; the bisector of
;

44.

Z.ACB meets AB at D; prove ^CDE = f Z.CAE.


O is the centre of the square BPQC
In A ABC, Z.BAC = 90
;

external to the triangle

prove that

AC

bisects /.

BAC.

XII*
45.

is 4 miles duo east of A ; a ship sailing from A to B against


the wind takes the zigzag course shown in Fig. 215, her
directions being alternately N. 30 E. and S. 30 E. ; what is

the total distance she travels ?

Fio. 215.

46.

ABC
75

is
;

a triangular sheet of paper, Z.ABC ==40, Z.ACB =


the sheet is folded so that B coincides with C ; find

the angle which the two" parts of


in the folded position.

AB make

with each other

186
47.

48.

CONCISE GEOMETRY

AB = AC; the bisector of Z ABC meets AC at D;


P is a point on AC produced so that /.ABP= Z.ADB;
=
prove BC CP.
ABC is a A BDEC is a square outside AABC lines
through B, C parallel to AD, AE meet at P prove PA is

A ABC,

In

',

perpendicular to BC.

BOOKS

II

I,

XIII
49.

AD, BE are altitudes


AD = 4*5 cms. find
;

50.

ABC

is

such
51.

is

AABC; BC = 5

of

an equilateral triangle

a variable point on a

ABCD is a
AABD.

cms.,

quadrilateral

if

BC, CA
ARB-120
R; prove
centre O, radius a; C is a

P,

thatBP-CQ; AP cuts BQ

are points on

at

circle,

fixed point at a distance b


least possible lengths of CP.

52.

CA=6

cms.,

BE.

from

,/

find the greatest

and

AACD = ABCD, prove AABC =

XIV
53.

Find in terms of

the area of Fig. 216.

x,

90

90

y
FIG. 216.

AABC, AB = AC a line PQR cuts AC produced, AB, BC


Q if PQ = QR, prove AP-f AR==2AC.

54. In

at R, P,
55.

The diagonals

= ABOC,
56.

In

of the quadrilateral

ABCD

cut at

O;

if

AS AOB, COD are equiangular.


AABC, Z.BAC = 90, AB = 5 cms., AC = 8 cms.;

AAOD

prove

area of the triangle and the length of

its

altitude

find the

AD.

REVISION PAPERS

18?

XV
57.

Find

in

cms.

sq.

the area,

making any

poiihtr'u,ti

and

measurements, of Fig. 217.

FIG. 217.

58.

ABODE

is

a regular pentagon;

BD

CE

cuts

at

P;

prove

+ -~)

inches

BP=BA.
59.

The hypotenuse
long,

and one

of a right-angled triangle

of the other sides

is

x2 -

(
\

is

x*
(
v

#27

inches.

X2/

Find

the third side.


60.

The

BC

side

point

of the parallelogram

K; prove

AABK = quad.

ABCD

is

produced to any

ACKD.

XVI
61.

ABCD

is

a parallelogram of area 24

intersect

at

from CD.
62. In
ABC,

Z.

O; AB = 4*5

cms.;

BAG = 90; BDEC

is

are altitudes of

BE,

64.

AD is an altitude of
BD = #, DC = y, prove

A ABC.

its diagonals
;
the distance of O

prove

A ABC;

DX = AB 4- AC.

A ABC; prove AC = CF
A ABC; AB = 7, AC = 5,

63.

the area of

cms.

a square outside

DX is the perpendicular from D to AC


CF

sq.

find

a;

y =24, and

find

BC = 8;
y

if

find also

CONCISE GEOMETRY

188

XVII

ABCD is a quadrilateral inscribed in a


ABCD in terms of p, q, r, s x, y.

65. In Fig. 218,


find the area of

rectangle;

A ABC, BAG =90; P, Q are points on BC


CA = CP and BA = BQ prove L PAQ = 45.
ABCD is a quadrilateral; /. ABC = L ADC = 90
are drawn parallel to CD, CB, cutting CB, CD
prove QA AB = PA AD.
[Use area formulae.]

66. In

such that

67.

68.

What
,

is

AP,
at P,

AQ
Q

the length of the diagonal of a box whose sides are

4", 12"

XVIII
69.

AD, BE,

CF

BC = BOJ

are

the

altitudes of

A ABC;

AD = 7 '5

CA = 3x

cms.,
cms.,
base BC of the triangle ABC

cms.

AB = 5#
find

cms.,

BE, CF.

70.

The

71.

ACD meet at P; a line through P


AC at Q, R prove QR= BQ~CR.
ABCD is a rhombus; P, Q are points on BC, CD such that
BP = CQ; AP cuts BQ at O; prove AAOB = quad. OPCQ.
In Fig. 219, AB=2", BC = 4", CD=1"; if PD 2 =2PA 2
bisecting /.s ABD,
parallel to BC cuts AB,

72.

is

produced to

the lines

find

PB.

90

Fio. 219.

REVISION PAPERS

189

XIX
73. Soundings are taken at
feet wide, starting

of 4 feet acio. * a river

inte*-* ri v)s

4 feet from on

depths in feet are obtained in


8*4, 10-2, 10*5, 7*8, 4*5; find

1*

*,nk,

oij*-tf

40
and the following
60, 9'3, 9'9, 8'2,

approximately the area of the

river's cross-section.

A ABC, AB-BC

74.

In the

75.

BAC
ABCD is
Z.

cuts

BC

at

A ABC,

Z.

to

is

= BQ

BAC = 90; P

drawn perpendicular

<

prove

a parallelogram;
produced to Q so that AB

76. In

ABC = 90; the


AB + BD = AC.

and

is

BC;

the mid-point of
;

prove

AD; AB

is

ABCD = 2 APQD.

the mid-point of

prove

bisector of

AC; PN

BN 2 =BA 2 + CN 2

is

XX
77.

AB-4

ABCD

is
a parallelogram;
the distance of A from BC is 6 cms.

cms.,
;

BC = 5aj

cms.;

find the distance of

from AB.
Fig. 220, AB
calculate the area

78. In

= BP = 4", BC=PQ = 3", AC=BQ = 5";


common to AS ABC, BPQ.

AB = AC; P is any point on BC; Q, R are the


mid-points of BP, PC QX, RY are drawn perpendicular to
BC and cut AB, AC at X, Y prove BX = AY.
ABC is an equilateral triangle BC is bisected at D and produced to E so that CE = CD, prove AE 2 =7EC2

79. In

A ABC,

80.

CONCISE GEOMETRY

190

XXI
triangle ABC

81. In Fig. 221, the


find its area and the distance of

is

inscribed in a rectangle:

from the mid-point of BC.

FIQ. 221.

82. A,

is

Y,
83.

are fixed points ; X is a variable point such that /.


the perpendicular bisectors of AX, BX cut AB at
;

AXB

obtuse

ABC

prove that the perimeter of AXYZ is constant.


a line XY parallel to BC cuts AB, AC at X,
a
;

is

and

is

produced

to

so that

XZ=BC;

Y
=
ABXY

prove

AAYZ.
84.

The

sides of a triangle are

obtuse-angled

8 cms., 9 cms., 12 cms.

Is

it

*?

XXII*
85.

ABC

a triangle of area 24 sq. cms.;


is a point on BC such that
cms.;
is

distance of
86.

87.

BA>AC.
ABCD is a

is

cms.,

BD=BC;

AC = 9

find the

from AB.

a point inside

drawn
88.

AB = 8

A ABC

such that

OA = AC,

prove that

quadrilateral; AB is parallel to CD; BP,


parallel to AC, AD to meet at P; prove

CP

are

APDC =

ABD.
The length,

breadth, and height of a

room are each 10

feet;

CAE, DBF are two vertical lines bisecting opposite walls,


CA = x feet,
C, D being on the ceiling and E, F on the floor
DB = 4 feet. Find in terms of x the shortest path from A
;

to

these

along these two walls and the ceiling ; (ii) along


two walls and one other wall. What is the condition
(i)

that route

(ii) is

shorter than route

(i)

REVISION PAPERS

89. In Fig. 222, AB---9",


late

XX III
BC-8 A CD -7";
,

91

if

AP

PD,

calcu-

BP.

oo

P
FIG. 222.

90.

ABC

91.

AP is the perpendicular from A


ABC; PQ is drawn parallel to BC to
=
=
prove AQ QB PQ.
is

to the^bisectcr

of

ABP,

PR

ABQ
is

cut

Q;

AB

QR

parallel to PB.
92. In
ABC, Z.BAC-90

at

are equivalent triangles on opposite sides of


parallel to BQ to meet AB at R ; prove

drawn

prove that

AB

H, K are the mid-points of A B, AC


BH + HK2 + KC 2 = BC 2
;

is

XXIV*
93.

The angles

at the corners of Fig.

223 are

all

right angles.

Construct a line parallel to AB to bisect the given figure.


[The fact in Ex. XXXI, No. 22, may be useful.]

FiO. 223.

94.

In

A ABC, L BAG = 90;

P,

are the centres of

squares which can be described on BC;


distances of P, Q from AB are J(AB+AC).

prove

the two
that the

CONCISE GEOMETRY

192
95.

ABCD
AC,

96.

In

a parallelogram

is

AD

at X, Y,

prove

A ABC, ZACB = 90;


3BD 2

BA

cuts

BC,

a median; prove that

AB 2

line parallel to

any

AAXY = ADYZ.

AD

is

BOOKS

I-III

XXV
97.

The

D
98.

side

ABCD
that

is

the

ABC is produced to
2
2=
AD
13AB
prove
a quadrilateral; if Z.ABC+ L ADC -180, prove
perpendicular bisectors of AC, BD, AB are con-

BC

so that

of

an equilateral triangle

CD = SBC

current.
99.

100.

ABCD

is

a quadrilateral inscribed in a

circle

AC

diameter; /.BAG = 43; find ZADB.


Two circles ABPQ, ABR intersect at A, B BP is a
to circle ABR; RAQ is a straight line; prove PQ is
;

is

tangent
parallel

to BR.

XXVI

ABC

a
H, K are the mid-points of AB, AC; P, Q are
;
on
such that BP = ^BC = jBQ; prove PH = QK.
BC
points
102. Find the remaining angles in Fig. 224.
101.

is

FIG. 224.

103.

104.

ABCD is a parallelogram the circle through A, B, C cuts


CD at P; prove AP = AD.
APB, AQB are two circles; AP is a tangent to circle AQB;
PBQ is a straight line; prove that AQ is parallel to the
;

tangent at P.

REVISION PAPERS
XXVII
105.

ABCD is a square P is a point on AB such


Q is a point on PC such tbu PQ = ^PC;
;

that

AP = JAB
APQD =

pro e

$ABCD.
106.

107.

AOB is a diameter of a circle perpeii'iii ular to a chord POQ


AO = &, PQ = et; tind AB in terms of a, h.
The side AB of a cyclic quadrilateral ABCD is produced to
Z.DBE-140
E;
Z.AOC=iOO, Z.ACB = 45; find
;

Z.BAC, /iCAD.

/.BAG = 90

A ABC,

108. In

BC

at

the tangent at

the circle on

cuts

AC

at

AB

as diameter cuts

prove

PD = PC.

XXVIII
ABCD, AB = 7", CD -II", Z.BAD= Z.ADC
=
/.BCD
-90,
60; calculate AC.
Two chords AB, DC of a circle, centre O, are produced to
meet at E; Z.CBE-75
Z.CEB = 22, Z.AOD=144;

109. In quadrilateral

110.

prove
111. In Fig.

112.

AOB = L BAC.
225, O is the centre

/.

and

TQ

bisects

Z.OTP; prove

PCA

are three unequal circles; from any point


on the circle PBC, lines DB, DC are drawn and produced

to

meet the

PAB, PBC,

a straight
13

circles

line.

PBA,

PCA

again at X,

prove

XAY

is

CONCISE GEOMETRY

194

XXIX
113. In

AABC, ZACB = 90, AC = 2CB; CD

prove by
wise that

is an altitude;
or othertheorem
the
of
using
Pythagoras'
figure

AD = 4DB.

114. In Fig. 226, O is the centre of the circle; PQ and


equally inclined to TO ; prove Z.QOT= 3 Z. POT.

PT

are

FIG. 226.

115.

AOB
at

is

A;

a chord of a

circle

Z.BOT=115; find
AABC, AB = AC the
at P; prove. BP= PC.
35,

116. In

ABC T is a point on the tangent


TO produced at P; /.ATO =
;

B meets

the tangent at

Z.BPT.
on

AB

circle

as diameter cuts

BC

XXX
117. X, Y,

are

any points on the

sides

BC, CA,

AB

of the

triangle
;
prove that AX + BY + CZ
J(BC -f CA -f AB).
118. A, B, C, D are the first milestones on four straight roads
running from a town X ; A is due north of D and north-

>

ABC

west of B.

is

E. 20

S. of

find the bearing of

from C.
119.

120.

ABCD is a
AC bisects

quadrilateral inscribed in a circle, centre


is perpendicular to
/. BAD, prove that

AB

diameter

TP

is

OC

of a circle

a tangent

APB

is

O;

if

BD.

produced to any point T;

L BTP + 2 L BPT = 90.

prove

XXXI
121.

ABCD

is

a rectangle; P

quad. ABCP
122.

ABCD

is

is

A APD AD

a circle;

=*

if

arc

any point on
CP.

ABC =

arc

ADC,

CD;
find

prove that

/.ADC.

REVISION PAPERS

C are points on a circle, centre O; BO, CO are pro


duced to meet AC, AB at P, Q; prove Z.BPC + /.BQC =

123. A, B,

L BAC.

124. In

227, AB
AH = BK.

is

Fig.

prove

a diameter;

P
Fro. 227.

XXXII
125. In

A ABC,

Z BAC = 90; AD

JL-J-+..L
AD AB AC
2

126.

ABCD

127.

ABC is

is

an altitude; prove that

'

a square inscribed in a circle; P


the minor arc AB ; prove /. APB = 3 /. BPC.
is

is

any point on

a triangle inscribed in a circle ; the bisector of /. BAC


is a point on PA between P and A
meets the circle at P ;
I

such that PI
128.

Two

circles,

tangents at

= PB

prove

L IBA = L IBC.

centres A, B, cut at X,

prove

L AXB

is

XP,

XQ

are the

equal or supplementary to

XXXIII
129.

ABCD is a parallelogram; P is any point on CD PA, PB,


CB, AD cut any line parallel to AB at X, Y, Z, W prove
DCZW=2AAPY.
In Fig. 228, O is the centre, PQ = AO, Z.AOQ = 90; prove
;

130.

arc

BR = 3

arc AP.

J*

Fro. 228.

CONCISE GEOMETRY

196
131.

A rectangular strip
how many
out of

prove

133.

it 1

CD

132. AB,

Two

of cardboard is 7 inches wide, 4 feet long;

circular discs each of radius 2 inches can be cut

are parallel chords of

equals angle between

Z.AOC

AD

XXXIV
AOB, COD cross

metre rules

ABDC,

circle

centre

and BC.

one another at right

angles the zero graduations are at A, C ; a straight edge


XY, half a metre long, moves with one end X on OB and
:

the other end

40 cms., those

on

for

OD

when the readings

are 50,

60 cms.

for

respectively.

are 50,

Find the

readings at O.
134.

Two

PARB, QASB
R and the

circles

B; a line PQRS
S; prove Z.PAQ =

intersect at A,

cuts one at P,

other at Q,

Z.RBS.

A ABC,

135. In

Z.BAC = 90; D is

the mid-point of BC ; a circle


and cuts AC again at E ;

BC at D, passes through A
arc AD = 2 arc DE.

touches

prove
136.

Two

circular

cylinders

of

6" are bound tightly

radii 2",

together with their axes parallel by an elastic band.


its

Find

stretched length.

XXXV
BC is an arc of radius 8"
OB produced; 08 = 9", L AOB = 90;

137. In Fig. 229,

of a circle touching

AO,

OB

and

arc

whose centre

lies

on

calculate the radius

BC.

B
FIG. 229.

138.

is

139.

a parallelogram ; AB, CB are produced to X, Y ; P


PCD - APAB
any point within the angle XBY ; prove

ABCD

is

= AABC.
A A 2 A3
that A! A 8

is

A^

is

a refeuar polygon of 20 sides, prove

perpendicular to

A 3 A 16

'REVISION PAPERS

C are three points on


BC produced at D; prove
BDA are at right angles.

140. A, B,

197

the tangent at A
;
that the bisect
of ^b

a circle

BM3,

XXXVI
ABC -90, ^BAC = I/*, Jie bisector of
AB at P prov AP 2 = 2PB 2
The diameter AB of a circle is induced to any point P; a
line is drawn from P touching Uio circle at Q and cutting
the tangent at A in R; prove Z.BQP J/.ARP.
In AABC, AB = AC and Z. BAG is obtuse a circle is drawn
touching AC at A, passing through B and cutting BC again
at P prove arc AB = 2 arc AP.
The volume of a circular cylinder is V cub. in. and the

HI. In AABC,

L ACB

142.

/.

meets

143.

144.

area of

its

curved surface

is

in.

sq.

find its radius in

terms of V, S.

BOOKS

I-IV

XXXVII
145. In Fig. 230,
triangles

if

^ADC= Z.BEA-

ABC, XYZ

Z.CFB, prove that the

are equiangular.

DC

FIG. 230.

146.

The tangent

is

at a point

the centre;

is

of a circle meets a chord

the mid-point of

PQ;

prove

PQ

at

T;

/.ROT =

RET.
147.

line

long, is clivided internally and externally


at P,
respectively ; find PQ AB.

AB, 8 cms.

in the ratio 3

CONCISE GEOMETRY

198
148.

ABCD
BD at
EF

a quadrilateral; a line AF parallel to BC meets


F ; a line BE parallel to AD meets AC at E ; prove

is

is parallel

to

CD.

XXXVIII

ABC are produced their own


AB, BC, CA of
=
Z; prove AXYZ 7AABC.
150. ABCD is a quadrilateral ; the circles on AB, BC as diameters
intersect again at P; the circles on AD, DC as diameters
intersect again at Q ; prove BP is parallel to DQ.
151. A town occupies an oval area of length 2400 yards, breadth

149.

The

sides

lengths to X, Y,

a plan is made of it on a rectangular sheet of


What is the best scale to choose?
18"
Jong, 12" wide.
paper
152. ABC is a triangle inscribed in a circle; AD is an altitude;

1000 yards

AP is

AB

a diameter

:
'

prove

AP

AC

and complete the equation

=
AP'

XXXIX
153.

AB

is a diameter of

a circle;

AOC, BOE

are

two chords

2 =
2OC2
such that
;
prove that AC
154. PQ is a chord of a circle T is a point on the tangent at P
such that PT= PQ ; TQ cuts the circle at R ; prove L RPT =

L CAB =

Z. EBA = 22

155. In Fig. 231, AB, CD,


CE - 4" ; find BC. If

156.

AB,

DB

DC
at

at P,

EF are parallel;
EF - 2", AB = 3",

AD=7
find

are parallel sides of the trapezium


the line through
;
parallel to

O
;

prove

PO - OQ.

77
,

DF

CD.

ABCD; AC cuts
AB cuts AD, BC

REVISION PAPERS

XL
157. In

A ABC, AB-AC

bisector of

158.

AOB,

AB

cuts

L BAG =120;

arid

BC

at

X;

prove

the perpendicular

BC = 3BX,

COD are two perj>endicuu< cnorcLo/ a rir le;


AC -f arc BD equals h'tlf the ir ip'itereiice.

prove

that arc
159.

light is placed 4' in front oi a cirtii'ai hoi

of a wall 5'

160.

ABC

is

3" in diameter

the diameter of the illuminated part


behind tho partition pnd parallel to it.

in a partition;

find

a triangle inscribed in a circle;

chord cutting

BC

at

AP AQ

prove

---

AB = AC; AP
AB'2

is

XLI
161. In

AABC, ZBAC=90, Z. ABC- 45; AB is produced to


so that AD DB = AB 2
prove that the perpendicular
.

bisector of

CD

bisects

AB.

ia acyclic quadrilateral; AC cuts BD at O; it CD


touches the circle OAD, prove that CB touches the circle

162.

ABCD

163.

ABCDEF

OAB.
is

2:3:7:4:5;
164.

ABCD

is

at E, F,

straight

line;

find the ratios

AD
DF

AB BC CD DE EF =
:

and ~.

AF

a parallelogram; a line through

A cuts

BD, BC,

CD

G; prove *p =
^.

XLII
165.

AB is a diameter of a circle APB; the tangent at A meets


BP at Q prove that the tangent at P bisects AQ.
PAQ, PBQ, PCQ are three equal angles on the same side
of PQ the bisectors of L s PAQ, PBQ meet at H
prove
that CH bisects L PCQ.
;

166.

167.

Two

3 cms.,
triangles are equiangular: the sides of one are
is
other
the
the
of
;
2J feet; find
perimeter

5 cms., 7 cms.
its sides.

168.

Two

OAB, OCD cut a circle at A, B, C, D; H, K are


on OB, OD such lhat OH = OC, OK=*OA; prove

lines

points
that

HK

is

parallel to

BD.

CONCISE GEOMETRY

200

XLiri
169.

170.

is the mid-point of AB
AP 2 -PB 2 =2AB.CP.

any point on

is

circular cylinder of height 6" is cut

4"

CB

prove that

from a sphere of radius

find its greatest volume.

Show

that the triangle whose vertices are (2, 1), (5, 1), (4, 2)
similar to the triangle whose vertices are (1, 1) (7, 1) (5, 3).
172. Two circles intersect at A, B ; the tangents at A meet the

171.

is

circles at C,

BC = BA
BA BD
-

prove

173.

XLIV
ABCD is a quadrilateral; AP is drawn
BD; prove AAPC = quad. ABCD.

174.

circular cone

and angle 240

made from a

is
;

equal and parallel to

sector of a circle of radius 6"

find its height.

straight rod

AB, 3' 9" long, is fixed under water with


A 2' 6" and B 9" below the surface ; what is the depth of
a point C on the rod where AC = I' 1
176. ABCD is a straight line; O is a point outside it; a line

175.

through B parallel to

OD

cuts

OA,

OC

at P,

if

PB= BQ,

AB _ AD
BC~~CD"

XLV
177. In Fig. 232, OA,

AB

two rods hinged together at A;


AO can turn freely about it ; the

are

the end

end B

constrained to slide in a fixed groove OC.


AB = 4' ; find the greatest length of the groove
can travel over, and calculate the distance of B from

is

OA = 3',
which B

is

fixed,

when AB makes

and

the largest possible angle with

Fir,.

232.

OC.

20 i

REVISION PAPERS
178.

ABC

179.

AOXB, COYD

a triangle inscribed in a circle; P, Q, R are the midthe arcs BC, CA, AB ; prove AP is perpendicular
of
points
is

toQR.
parallel lines

BD

AC, XY,

lines;

are

XB = 3, AC = 2, BD = 4

7,

XY.

find

180.

two straight
=
cutting them j AX
are

any point on the common chord of two circles, centres


B; HPK and XPY are chords of the two circles per=
pendicular to PA, PB respectively; prove HK XY.

is

A,

XLVI
181.

ABC

a triangle inscribed in a

is

external bisectors of

182.

/.

BAG

cut

circle

BC

tangent at A bisects PQ.


circle of radius 4 cms. touches

the internal and

at P,

prove that the

two perpendicular lines;


and the

calculate the radius of the circle touching this circle

two
183.

it

a rectangle; AB = 8", BC=5"; P


whose distances from AD, AB arc 2", I"
is

meet

to

184.

lines.

ABCD

Two

AB

at

CE

CAB, OCD
OA CD AD 2
OB OC BC 2

lines

cuts

AD

at F

meet a

a point inside

is
;

DP

is

produced

calculate EB, AF.

circle at A, B, C,

D; prove

that

XLVil*
185.

ABC

P is any point on BC; AC


=
=
BP
CQ
prove AP PQ.

an equilateral triangle

is

produced to Q so that
186. AB is a diameter of a circle
is

pendiculars from A,

APB; AH, BK

to the tangent at

are the perprove that AH 4-

BK = AB.
187.

AB

chord

tangent;
188. AB,

DC

of a circle

ABT

OA=6", OT = 4",

is

at

DC

at P,

E ; DA,

is

ABCD AC

cuts

EF

AB,

are parallel sides of the trapezium


CB are produced to meet at F

BD

O; OT

produced to
find BT.

AT- 3",

prove SL. == SL.

cuts

202

CONCISE GEOMETRY
XLVIII*

189.

A brick

on the ground and an equal brick

rusts

up against

it

The

as in Fig. 233.

bricks are

is

propped

2".

by

Calculate the height of each corner of the second brick above


the ground, if AB = 1 J".

190. Prove that the area of a square inscribed in a given semicircle is | of the area, of the square inscribed in the whole
circle.

191.

The

bisector of

Z.BAC

perpendicular to
192.

chord

DA

BC

at

AC

cuts AB,

AD is parallel
C meets AD

tangent at

cuts

the line through

D;

at Y,

prove

BC of a circle;
BC AE = BD 2

to a diameter

at

prove

BY = BD
CZ DC

the

XLIX*
a fixed point on a given circle ; a variable chord AP is
produced to Q so that PQ is of constant length ; QR is drawn
perpendicular to AQ ; prove that QR touches a fixed circle.

193.

194.

Four equal

is

circular cylinders, diameter 4*, length 5", are

packed in a rectangular box


unoccupied space in the box 1
195.

what

the least amount of

is

A rectangular sheet of paper ABCD is folded so that B falls


on CD and the crease passes through A AB = 10", BC = 6"
;

find the distance of the

new

position of

BC at Q, find CQ.
a parallelogram ; a line through

B from

C.

If the

crease meets

196.

ABCD is
in P, Q,

R; prove

PO = PD 2
-|

*
.

A cuts BD, CD, BC

203

REVISION PAPERS

197. In Fig. 234,


The area of

ABCD is a rectangle BP= 2CQ; AD = 2AB = 6".


APQD is 10 sq. in. find BP.
;

FIG.

198.

ABC

is

'234.

a triangle inscribed in a circle ; the tangents at B,


T ; a line through T parallel to the tangent at

meet at

meets AB,
199.

AC

produced at D,
parallel to BC cuts

HK
between HK and BC is
line

are 9 sq. cms.,

200. In

A ABC,

sponding to

40

is

BC

sq.

5 cms.

cms.

AB,
;

prove Al

the areas of

find

Al x =

distance

AHK and HKCB

HK.

the in-centre and


;

=
prove DT TE.
AC at H, K the

AB

is
.

the ex-centre corre-

AC.

CONCISE GEOMETRY

204

WHEN

learning propositions, do not use the figure printed in the

book, but
It is

draw your own

figure instead.

more trouble but gives

bettor results.

For

this reason,

no attempt has been made to arrange the whole proof of every


theorem on the same page as the corresponding figure.

A freehand

figure is

good enough.

PROOFS OF THEOREMS
BOOK
DEFINITION.

CD

line
is

is

If

is any point on the straight line AB, and if a


so that the angles ACD, BCD are equal, each

drawn

called a right angle.

AC

Fro. 235.

Therefore

ACB

is

if

is

any point on the

THEOREM
(1) If

straight line

AB, the angle

equal to two right angles, or 180.

one straight line stands on another straight line, the


of the two adjacent angles is two right angles.

(2) If at a point in a straight line,

opposite sides of
to

two right

it,

make

angles, these

two other straight

sum

lines,

on

the adjacent angles together equal


straight lines are in the same

two

straight line.

E>

FIG. 1.
205

CONCISE GEOMETRY

206
(1)

To Prove L

AB at C.
ACE + ^ BCE =

ACE+ ^BCE^

Given

CE

meets

80.

L.

ACB

= 180,

since

ACB

is

st. line.

Q.E.D.

Ey

ACB

FIG. 236.

ACE + L BCE = 180.


To Prove ACB is a straight line.
Produce AC to F.
ACE + L FCE - 180, since ACF is
BCE- 180, given.
But ACE 4ACE -h FCE = L. ACE + - BCE.
/.

(2) Given

.'.

L.

L.

st. line.

L.

L.

^FCE=

/.

CB
But

L.

L.

ACF

is

st.

Z.BCE.

falls

line

.*.

along CF.
ACB is a

st. line.

Q.E.D.

THEOREM
If

two straight

lines intersect, the vertically opposite angles

equal.

FIG.

To Prove

2.

that x = y and a = /3.

x 4- a = 1 80
a + y = 1 80

Similarly

adjacent angles.
adjacent angles.

a = j8.
Q.K.D.

For

riders

on Theorems 1-2, see page

2.

PROOFS OF THEOREMS
THEOREM
If

207

triangles have two sides of one equal respectively to two


sides of the other, and if the included angles are equal, then

two

the triangles are congruent.

FIG. 2#.

Given

AB = PQ, AC - PR, BAG = L QPR.


To Prove A ABC = A PQR.
L.

Apply the triangle ABC to the triangle PQR, so that A


on P and the line AB along the line PQ ;
Since AB = PQ, .'. B falls on Q.
Also since

AB

falls

along

PQ and L BAG = L QPR,

.".

falls

AC falls

along PR.

But

AC = PR,

C falls on R.
ABC coincides with

/.

.".

the triangle

/.

AABC-APQR.

the triangle

PQR.
Q.K.D.

For

riders

on Theorems

3, 9, 10,

see page 16.

THEOREM 4
If

one side of a triangle is produced, the exterior angle


than either of the interior opposite angles.

is

greater

CONCISE GEOMETRY

208

BC

is

produced to D.

>

L ABC and L ACD > L BAG.


To Prove L ACD
Lot F be the middle point of AC. Join BF and produce
to G, so that BF= FG.
Join CG.
In the triangles AFB,

it

CFG

AF - FC and BF = FG, constr.


L AFB
u CFG, vert. opp.
=--

.'.

AAFB-ACFG.

/.

.LBAF- Z.GCF.

But L

DCA>
DCA >

its

part

L,

GCF.

L BAI; or L BAC.
if BC is bisected and if AC is produced
Similarly,
can be proved that L BCE > L. ABC,
But L ACD - L BCE, vert. opp.
.'.

/.

ACD>

to E, it

ABC.
Q.E.D.

Straight lines which lie in t/ie same pl<me and


which never meet, however far tlifj are produced either way, are

DEFINITION.

called parallel straight lines.

PLAYF AIR'S AXIOM. Through a given point, one and only one
straight line can be drawn parallel to a given straight line.

THEOREM

If one straight line cuts two other straight lines such that
either (1) the alternate angles are equal,

or

or

(2) the corresponding angles are equal,


(3) the interior angles on the same side of the cutting
line are supplementary,

then the two straight lines are

/A
P

parallel.

PROOFS OF THEOREMS

ABCD

cuts PQ,

RS

209

at B, C.

(1) Given ^PBC= L BCS.


To Prove PQ is parallel to RS.
If PQ, RS are not parallel, they

will

meet when produced,

at H, say.
Since BCH is a triangle,
ext.

which

is

.".

L.

PBC

>

int.

<L

BCH,

contrary to hypothesis.

PQ

cannot meet

RS and

is

.*.

parallel to

it.

Q.B.P.

\?) Given

ABQ =

To Prom PQ

L.

is

BCS.
parallel to RS.

ABQ = L PBC, vert. opp.


But L ABQ =
BCS, given.
-L PBC = L. BCS.
/.
/. by (1), PQ is parallel to RS.
SCB - 180.
Given L. QBC 4To Prove PQ is parallel to RS.
L. QBC+ <- PBC = 180, adj.
angles.
But _ QBC + L. SCB - 180, given.
_ QBC +
PBC = L QBC f L SCB.
L. PBC SCB.
/.
/. by (1), PQ is parallel to RS.
L.

L.

(3)

.'.

L.

L.

Q.E.D.

THEOREM
If

a straight line cuts two parallel straight lines,


Then (1) the alternate angles are equal;
(2) the corresponding angles are equal ;
(3) the interior angles on the same side of the cutting
line are supplementary.
V

P
B

FIG. 239.

14

CONCISE GEOMETRY

210
AB,

CD

are two parallel st. lines ; the


To Prove (1) Z.AQR- ^QRD.
(2)
(3)

(1) If

AQR
to

But

L.

is

PS

line

PQB = QRD.
^.BQR+ ^.QRD-180

L.

cuts

them

at Q, R.

L.

not equal to L

QRD,

let

the angle

XQR

be equal

QRD.

these are alternate angles.


is parallel to RD,

QX

.*.
.'.

(2)

But

two intersecting lines QX, QA are both parallel to RD,


which is impossible by PI ayfair's Axiom.
L.
AQR cannot be unequal to L QRD.

LPQB=- L AQR,

vert. opp.

^AQR=

alt.

But
.'.

angles.

AQR - 180, adj. angles.


AQR = L QRD, alt. angles.
L
^BQR + QRD = 180.

L.

(3)

1.QRD,

BQR +

L.

Q.E.D.

For

riders

on Theorems

5, 6, see page 6.

THEOREM

'

of a triangle is produced, the exterior angle


(1) If a side
to the sum of the two interior opposite angles.
(2)

The sum

ABC

of the three angles of

any triangle

is

two right

a triangle ; BC is produced to D.
L CAB + L ABC.
To Prove (1) L. ACD

is

(2)

L.

CAB +

L.

ABC 4-

is

ACB = 180.

equal

angles.

PROOFS OF THEOREMS
(1)

Let

CF

be drawn parallel to AB.

adding, L

Add

FCD = L ABC, corresp. angles*


ACF = L CAB, alt. angles,
ACF = L ABC + L CAB.

L
L
L

FCD +

1ACD=

.'.

(2)

to each the angle

.LABC + /.CAB.

ACB.

ACD + ACB - L ABC + _ CAB +


_ ACD + L ACB = 180, adj. angles.
L ABC +
CAB + - ACB - 180.
.*.

But

211

,1

L.

ACB.

21

.'.

Q.K.D.

THEOREM 8
a convex polygon, together with four
are
equal to twice as many right angles as the
right angles,

(1) All the interior angles of

polygon has

sides.

(2) If all the sides of a

sum

convex polygon are produced in order, the

of the exterior angles

is

four right angles.

Let n be the number of sides of the polygon.


(I) To Prove that
the

sum

rt.

the

of

angles of the polygon

+4

rt. L. s

= 2n

LS.

Take any point

inside the polygon

and

join

to each

it

vertex.

The polygon
But the sum
.".

the

sum

is

now

divided into

triangles.

of the angles of each triangle is 2


of the angles of the n triangles is

But these angles make up 'all the angles


together with

all

the angles at O.

of

rt.

2n

s.

rt.

s.

the polygon

212

CONCISE GEOMETRY

Now
.".

sum of all the angles at


the angles of the polygon

the
all

-h

is

rt.

rt. <- s

s.

= 2n

rt.

s.

FIG. 242.

(2)

At each
.".

vertex, the interior

the

sum

_ + the exterior - - 2

of all the interior angles

exterior angles

=-

2n

rt.

-j-

the

rt.

sum

s.

of all the

s.

]}ut the HUIII of all the interior angles -f I rt. L. s


the sum of all the exterior *_ s
4 rt. .L s.
.*.

= 2n

rt.

s.

Q.E.D.

may
The sum of
8(1)

also be vt--lod as follows

the interior angles of any convex polygon of

sides is 2w

-4

right angles.

For riders on Theorems

7, 8,

see page 10.

TJIEOUKM 9

Two

if two
angles and a side of one are
two angles and the corresponding side

triangles are congruent

respectively equal to
of the other.

Given either that

BC^QR.

PROOFS OF THEOREMS

213

= QR.
z_A3C= A. PQR.

or that FiC

To Prove AABC== APQR.


The sum of the three angles

of any triangle is 180.


in each case, the remaining pair of angles is equal.
to the triangle PQR so that B falls
Apply the triangle
.*.

ABC
Q and BC falls along QR.
Since BC = QR, C falls on R. -A
And since BC falls on QR aiul'-L ABC=on

PQR,

.".

BA

falls

L PRQ,

/.

CA

falls

L.

along QP.

And

since

CB

falls

on

RQ

falls

on

P.

and

L.

ACB =

along RP.
/.
.*.

the triangle
.'.

ABC

coincides with the triangle

PQR.

A ABC = A PQR.
Q.E.D.

THEOREM 10
(1) If

two sides of a triangle are equal, then the angles opposite

to

those sides are equal.


(2) If

two angles of a triangle are equal, then the

sides opposite to

those angles are equal.

FIG. 244.

ABC

a triangle
at D.

is

let

the line bisecting the angle

AB - AC.
To Prwe L ACB = L ABC.

(1) Given

As ABD, ACD.
AB AC, given.
AD is common.

In the

BAC

meet

BC

CONCISE GEOMETRY

214

BAD = L CAD, constr.


the AS are congruent.
L ABD - L ACD.
Given L ABC = L ACB.
To Prove AC = AB.
L

.*.
.'.

(2)

AS ABD, ACD.
ABD = L. ACD, given.
L BAD = L CAD, constr.
AD is common,
AS are congruent.
AB = AC.

In the
i.

the

.'.

Q.E.D.

For

riders

on Theorems

3, 9,

10 see page

THEOREM

Two

triangles are congruent

if

15.

11

the

three sides of

one are

re-

spectively equal to the three sides of the other.

FIG. 245(2).

Given that AB = XY, BC = YZ, CA = ZX.


To Prove
ABC s? XYZ.

ABC so 'that B falls


since BC = YZ, C falls on

Place the triangle

YZ;

/.

on
Z.

Y and BC

along

PROOFS OF THEOREMS
Let the point

YZ to X.
Now YF = BA,
But BA^YX,

constr.

.'.

Join XF.

given.

YFX.

sides of the triangle

.YXF = /.YFX.
Z.ZXF- _ ZFX.

.'.

Similarly,

adding in Fig. 245(1) or subtracting in Fig. 245(2)


^_YXZ= <_YFZ.

But L
L.

.'.
.'.

a point F on the opposite side of

YF-YX

But these are

.*.

fall at

215

in the

BAC=

L YFZ,

constr.

BAG - L YXZ.

AS ABC, XYZ
AB = XY, given.
AC = XZ, given.
Z.BAC- ^.YXZ,

A ABC A

proved.

XYZ.

==

/.

J\

Q.B.D.

THEOREM 12

Two

right-angled triangles are congruent if the hypotenuse and


side of one are respectively equal to the hypotenuse and a
side of the other.

Fid. 246.

Given /.ABC

= 90= L. XYZ.
AC = XZ.
AB = XY.

To Proe

AABC== AXYZ.
ABC so

Place the triangle

along XY, and so thai


opposite side of

XY

th^t

to Z.

on X and AB falls
some point F on the

falls

falls at

CONCISE GEOMETRY

216

AB = XY, B falls on Y.
XYF = L ABC = 90 and u XYZ = 90.
Z.XYF + ^ XYZ =180.
ZYF is a straight line.
But XF = AC, and AC is given equal to XZ.
XZF is a triangle, in which XF XZ,

Since
L.

.'.

But L
.'.

/.

XFY-

L ACB,

constr.

XZY - ACB.
in the AS XYZ, ABC.
L XYZ = L ABC, given.
L. XZY =
ACB, proved.
XY - AB, given.
A XYZ = A ABC.
L.

L.

L.

/.

Q.E.D.

THEOREM 13
(1)
(2)

The opposite sides and angles of a parallelogram are


Each diagonal bisects the parallelogram.

equal.

Fm.
Given

ABCD

is

To Prove

as

a parallelogram.
(1)

AB - CD and AD - BC.
DAB = L DCB and L ABC = ADC.
AC and BD each bisect the parallelogram.
L.

(2)

L.

Join BD.

In the AS ADB, CBD


L ADB = L CBD, alt. L
L ABD = L. CDB, alt. _

BD
/.
.'.

is

s.
s.

common.

A ADB s A CBD.
AB = CD, AD = BC, c L DAB == L BCD
and BD bisects the parallelogram.

PROOFS OF THEOREMS

217

by joining AC it may bo proved that L


L ADC, and that AC bisects the parallelogram.

Similarly,

ABC =

Q.E.D.

THEOREM 14
The diagonals

of a parallelogram bisect one another.

D
FIG. 29.

The diagonals AC, BD of the parallelogram


To Prove AC = OC and BO - CD.
In the

intersect at O.

As AOD, COB,
L DAO = L BCO,
L

/.

ABCD

ADO

alt. L s.
L CBO, alt. L s.
BC, opp. sides of

AD =
||gram.
AOD = A COB.
AO = CO and BO = DO.
Q.E.D.

THEOREM 15
The

straight lines
straight lines

and

which join the ends of two equal and parallel


towards the same parts are themselves equal

parallel.

C
FIG. 30.

Given

AB

is

equal and parallel to CD.

To Prove
Join BC.

AC

is

equal and parallel to


1

CD

CONCISE GEOMETRY

218
In the

As ABC, DOB

ABC =

AB = DC, given.
BC is common.
DCB alt. angles, AB

being

||

to

CD.

AABC-ADCB.
AC-DBand ACB = ^ DBC.
.'.

.-.

<L

J3ut these are alt. angles,

.'.

AC

is

parallel to

DB.
Q.E.D.

This theorem can also be stated as follows

quadrilateral which has one pair of equal

a parallelogram.
Other tests for a parallelogram are
(1) If the diagonals of

and

parallel sides is

a quadrilateral bisect each other,

is

it

is

it

is

it

parallelogram.

the opposite sides of a

(2) If

quadrilateral are equal,

parallelogram.
(3) If the opposite angles of a quadrilateral are

equal,

parallelogram.

For

riders

on Theorems

11, 12, 13, 14, 15, see

BOOK

page 23.

II

THEOREM 16
(1) Parallelograms

on

the

same base and between the same

parallels are equal in area.

(2)

The area

of a parallelogram

base and

(1) Given

measured by the product of

its

are two 'parallelograms on the same base


and between the same parallels AB, DP.

ABCD, ABPQ

AB

is

its height.

219

PROOFS OF THEOREMS
To Prove that ABCD, ABPQ are equal in area.
In the As AQD, BPC,
L ADQ = L BCP, corresp. L. s AD, BC being
L AQD= L BPC, corresp. L. s; AQ, BP being
;

AD = BC,
From

||

lines.

||

lines.

opp. sides Hgram.

the figure

ABPD,

subtract in succession each of the

equal triangles BPC, AQD.


the remaining figures ABCD,
.".

ABPQ

are equal in area.

Fia. 32.

(2) If

BH

is the perpendicular from B to


CD, the area of ABCD
measured by AB BH.
Complete the rectangle ABHK.
The Hgram ABCD and the rectangle ABHK are on the same

is

base and between the same parallels and are therefore


equal in area.
But the area of ABHK = AB BH ;
.

.'.

the area of

ABCD = AB BH.
.

Q.B.D.

THEOREM 17
The area of a triangle
and the height.

is

measured by half the product of the base

C
FIG. 247.

CONCISE GEOMETRY

220
Given that

AD

the perpendicular from

is

triangle

To Prove

to the base

BC

of the

ABC.
that the area of

A ABC = JAD

BC.

Complete the parallelogram ABCK.


Since the diagonal AC bisects the parallelogram

A ABC

=*

i parallelogram

ABCK = AD BC
AABC-IAD.BC.

But parallelogram
/.

ABCK,

ABCK.
.

Q.E.D.

THEOREM 18
(1) Triangles

on the same base and between the same

parallels are

equal in area.

and on the same

(2) Triangles of equal area on the same base


side of it are between the same parallels.

(1) Given

two

triangles

ABC,

ABD

on the same base

between the same parallels AB, CD.


To Prove the triangles ABC, ABD are equal in

Draw CH, DK

But

CH

is

AB, and

perpendicular to

parallel* to

DK,

AB

or

AB

since each

area.

produced.

is

CD is given parallel to HK.


CDKH is parallelogram.
CH = DK? opp. sides.
A CAB equals A DAB in

AB and

perpendicular to

9>

area.

221

PROOFS OF THEOREMS
(2)

Given two triangles ABC, ABD of equal area.


To Prove CD is parallel to AB.

Draw CH, DK perpendicular to AB or AB produced.


Now ACAB = JCH AB and A DAB = JDK AB.
CH AB = DK AB.
CH - DK.
/.
But CH is parallel to DK, for each is perpendicular to AB.
Since CH and DK are equal and parallel, CHKD is
.

.'.

.".

parallelogram.
.*.

CD

is

parallel to

HK

or

AB.
Q.E.D.

THEOREM 19
If

a triangle and a parallelogram are on the same base and between


the same parallels, the area of the triangle is equal to half
that of the parallelogram.

ABC and the parallelogram ABXY on tho same


and between the same parallels AB, CX.

Given the triangle


base

AB

To Prove

AABC = J

||gram

ABXY.

Join BY.

The AS ABC, ABY


same parallels.
.'.

on the same base and between the

AABC = AABY in

Since the diagonal

.'.

are

BY

area.

bisects the ||gram

ABXY,

AABY = \ Hgram ABXY


AABC - 1 Jlgrani ABXY.

The following formula for the area

Q.E.D.

of a triangle is important
If a, 6, c are the lengths of the sides of a
triangle and
s
6
the
of
area
the
+
+
J(a
c),
triangle

if

CONCISE OEOMETBY

222
By

using the results

Area of parallelogram = height x base,


= height x base.
Area of triangle
Proofs similar to the proof of Theorem 18 can be easily obtained
theorems
on
equal bases and between the same
(1) Triangles

for the following

parallels are

equal in area.
(2) Parallelograms

on equal bases and between the same

parallels

are equal in area.

which are on equal bases in the same


and on the same side of it, are between the

(3) Triangles of equal area,

straight line

same

parallels.

which are on equal bases in the


same straight line and on the same side of it, are between
the same parallels.
The area of a trapezium = the product of half the 'sum of the
parallel sides and the distance between them.

(4) Parallelograms of equal area,

(5)

For

riders

on Theorems 16, 17, 18, 19, see page 28.

THEOREM

20.

[PYTHAGORAS' THEOREM.]

In any right-angled triangle, the square on the hypotenuse is equal to


the sum of the squares on the sides containing the right anglp!

P
FIG. 248.

223

EKOOFS OF THEOREMS
Given L

BAG

is

a right angle.

BC = the

To Prove the square on


on AC.
Let

ABHK, ACMN, BCPQ

square on

BA 4- the

square

be the squares on AR, AC, BC.

Through A, draw AXY


BC,
QP at X, Y.
cutting
Since L. BAC and L BAK arc right angles,

Join CH, AQ.

parallel

KA

and

to

BQ

AC

are

same straight line.


= - QBC.
L. HBA - 90
Again
Add to each L. ABC, .'. u HBC= L ABQ.
in the

In the

As HBC, ABQ
HB = AB,
CB = QB,

HBC^ ABQ,
AHBC-AABQ.
_

/.

sides of square.
sides of square.

t-

proved.

Now

Also

A ABQ and rectangle BQYX

HBC and square HA are on the same base


between the same parallels H B, K AC ;
.'.

A HBC=| square

and between the same


.".

AABQ - \

.'.

square

Similarly,

and

HA.

parallels

rect.

HB

are on the same base


BQ, AXY.

BQ

BQYX.

HA -- rect. BQYX.

by joining AP, BM,

it

can be shown that square

M Afreet. CPYX;
.'.

square HA

+ square MA = rect. BQYX + rect. CPYX

= square

BP.
Q.E.D.

THEOREM

21

square on one side of a triangle is equal to the sum of


the squares on the other sides, then the angle contained by
these sides is a right angle.

If the

r.

FIG. 249.

CONCISE GEOMETRY

224
Given

AB 2 + BC 2 -= AC 2
To Prove u ABC = 90.
Construct a triangle XYZ such that XY-AB, YZ-BC,
L. XYZ - 90.
Since L XYZ = 90, XZ 2 = XY a + YZ a
But X Y - AB and YZ - BC.
XZ 2 -AB 2 + BC 2 = AC 2 given.
/.
XZ-AC.
/.
in the A ABC, XYZ
AB = XY, constr.
BC = YZ, constr.
AC = XZ, proved.
.

.'.

AABC-AXYZ.

.'.

L.

.'.

But L
/.

L.

ABC ~L XYZ.
XYZ = 90 constr.
ABC 90.
Q.K.D.

For riders on Theorems 20, 21, see page 38.


DEFINITION.
if

AH, BK

called

If

AB

and

CD

are

any two

are the perpendiculars from A,


of AB on CD.

FIG. 250(1).

Thus, in Fig. 248,

QY is the projection of BA on
XC is the projection of AC on
BX is the projection of QA on
Or, in Fig. 250(2),
AN is the projection of

BN

is

the projection of

QP,
BC,
BC.

AC dh AB,
BC on AB.

vstraight lines,

to

CD, then

and

HK

is

PROOFS OF THEOREMS

225

THEOREM 22
In an obtuse-angled triangle, the square on the side opposite the

sum of the squares on the sides


containing it plus twice the rectangle contained by one
of those sides and the projection on it of the other.
obtuse angle is equal to the

AN

FIG. 250(2).

Given L

BAG

is

CN

obtuse and

is

the perpendicular from

to

BA

produced.

To Prove

BC 2 = BA 2 + AC 2 + 2BA AN.
.

[Put in a small letter for each length that


and also for the altitude.]

BC = a units, B A = c
CN = h units.

Let

units,

It is required to prove that a?

Since L

BNC = 90,

Since L

ANC = 90,

b*

= (c

units,

AN

units,

= c2 + b2 + 2cx.

a* =

:.

or

AC = b

comes in the answer

BC 2 = BA 2 + AC2 + 2BA AN.


.

Q.E.D.
*

THEOREM 23
In any triangle, the square on -the side opposite an acute angle
is

equal to the
15

sum

of the squares

on the

sides containing

CONCISE GEOMETRY

226

minus twice the rectangle contained by one


sides and the projection on it of the other.
it

of

those

X
PIG.

FIG. 59(2).

5~9(1).

BAC is acute and CN is the perpendicular


AB produced.
To Prove BC2 = BA2 + AC 2 - 2AB AN.

Given L

from

to

AB

or

[Put in a small letter for each length that comes in the answer
and also for the height.]

BC = a units, BA = c
CN==A units.

Let

units,

It is required to prove that

a2 = c2 + 6 2 -

BNc-#;

In Fig. 59(1),
Since L. CNB - 90, a 2 or

/
Since

in each case,

units,

AN = x

units,

2ra.

BN=#-c.

in Fig. 59(2),
2
A2 in Fig. 59(1),
(c
a?) +
2
2
(x c) + A in Fig. 59(2) ;

a2 a 2 c2

ANC = 90, 62 =
or

AC = b

a?

+ A2

BC 2 = BA2 + AC 2 - 2 AB AN.
.

Q.K.D.

THEOREM
In any

24.

sum

[APOLLONIUS* THEOREM.]

of the squares on two aides is


equal
on half the base phw twice the square
on the median which bisects the base.
triangle, the

to twice the square

^ROOFS OF THEOREMS
Given

the mid-point of BC.

is

To Prove

AB 2 + AC 2 = 2AD 2 + 2BD 2

Draw AN
From the
From the

perpendicular to BC.
triangle
triangle

BD = DC,

But

227

.'.

AB 2 - AD 2 + DB
AC 2 = AD 2 + DC 2 - 2DC DM.
BD.DN-DC.DN and BD 2
AB 2 + AC 2 = 2AD 2 + 2DB 2
S

ADB,
ADC,

given;

adding,

.'.

Q.R.D.

For

riders

on Theorems 22, 23, 24, see page 44.

THEOREM 25
(1) If A, B, C,

are four points in order

AC BD - AB CD + AD BC.
Tf a straight line AB is bisected
.

(2)

on a straight

point on AB, then

at O,

and

if

AP 2 + PB 2 = 2AO 2 + 2OP 2
x

is

ss
*

FIG. 251(1).

AB x units, BC = y units, CD ~z
AC = # + y, BD = y + z.
AC.BD = (o?-fy) (y-f z)

(1) Let

units.

Then
.'.

AD = x y -f
AB CD + AD BC = xz + (x + y + z)

Also
/.

-i-

AC BD = AB CD + AD
.

"A

>

FIG. 251(2).

(2)

Let

AO = #

units,

OJP^y

units.

PB = OB - OP rr x - y
andAP = AO-fOP

Also

.*.

line,

then

BC.

B~

any other

CONCISE GEOMETRY

228

= 2AO 2 + 2OP 2

Q.B.D.

For

riders

on Theorem 25, see page 46.

GEOMETRICAL ILLUSTRATIONS OF ALGEBRAIC


IDENTITIES

Fio. 252(1).

PQ of length a + b inches and take a point R on


RQ is of length b inches.
On PQ and RQ describe squares PQXY, RQHK on the
same side of PQ and produce RK, HK to meet XY, PY

Draw
it

a line

such that

at M, L.

Then the area of PQXY is (a 4- b)'2 sq. inches.


The areas of LKMY and RQHK are a2 sq. inches and
sq. inches.
The area of each of the rectangles

II.

(a

+ J)(a-6) = tt 2
R

FIG. 252(2).

PK,

KX

is

ab

sq. inches.

62

229

PROOFS OF THEOREMS
Draw
part

a line

PR

PQ

of length

a inches (a>b) and cut

off

of length b inches.

On PQ and PR describe squares PQXY, PRHK; produce


KH to meet QX at L.
Produce KL, YX to E, F so that LE = XF = 6 inches.
Now LX = QX-QL = QX-RH=a-6 inches.
the rectangle LXFE equals the rectangle HLQR.
the rectangle KYFE equals the sum of the rectangles
KYXL and HLQR equals PQXY - PRHK = a2 -6* sq. in.
But KY = a - b inches, YF = a 4- b inches.
.*.

.".

THEOREM 26
two sides of a triangle are unequal, the greater side has the

(1) If

greater angle opposite to

two angles

(2) If

it.

of a triangle are unequal, the greater angle has

the greater side opposite to

it.

FIG. 253.

(1)

GumAOAB
To Prove L

ABC>

L ACB.

cut off a part AX equal to AB.


AB = AX, L. ABX = L AXB*
But ext. L AXB > int. opp. L. XCB,
L ABX > L XCB.
/.

From AC

Join BX.

Since

But
/.

2L.AfeOz.ABX,
L ABC > L XCB or

L ACB.

CONCISE GEOMETRY

230
(2) Given

ABC >

To Prom
If

AC

L ACB.

AOAB.

not greater than AB,


or less than AB.
is

it

must either be equal

to

AB,

Fia

61.

ABC = L- ACB, which is contrary to hypothesis.


L ACB, which is contrary to hypothesis.
^.ABC<
IfAC<AB,
If

AC

AB, _

AC

.".

must be greater than AB.


Q.E.D.

THEOREM 27
Of

all

straight lines that can be drawn to a given straight line


from an external point, the perpendicular is the shortest.

FIG. 62.

Given a fixed point

ON

is

and a

fixed line

the perpendicular from

AB.
to

AB, and

OP

is

any other

from O to AB.
To Prove ON<OP.
line

Since the
since

sum

of the angles of a triangle is 2

^ONP=1
.'.

/.

L
L
L

rt.

NPO +

rt.

angles,

angle.

NOP = 1

rt.

angle.

NPO< 1 rt. angle.


NPO< - ONP.
ON<OP.
Q.B.D

and

231

PROOFS OF THEOREMS
THEOREM 28

Any two

sides of a triangle are together greater than the third

side.

x
*V
'I
*

*''

I
i

A-''

C
B
Fio. 254.

Given the triangle ABC.

To Prom BA + AOBC,
BA to P and cut

Produce
Since

AX = AC,
But
/.

.'.

off

ACX -=
L BCX >
L BCX >
L.

AX

equal to AC.

Join CX.

u AXC.
L ACX.
L.

AXC.

BCX > - BXC.


BX>BC.
BX = BA + AX = BA + AC.

in the triangle

BXC,

.'.

But
/.

BA + AOBC.
Q.E.D.

The following theorem


The shortest and longest

an easy rider on the above


distances from a point to a

is

circle lie

along the diameter through the point.

FIG. 255.

If

AB

is

a diameter, tod If P

PA>PQ>PB.

lies

on

AB

produced,

CONCISE GEOMETRY

232
Join

to the centre O.

PA = PO + OA-PO l-OQ>PQ.
PB + BO=PO<PQ + QO.
For

riders

on Theorems 26, 27, 28 see page 49.

THEOREM 29
The

straight line joining the


triangle

middle points of two sides of a

parallel to the base

is

and equal

to half the base.

C
FIG. 256.

Given H,

are the middle points of AB, AC.


is parallel to BC and

To Prove

HK =

HK

CP

Through C, draw

BA

parallel to

to

BC.
meet

HK

produced

at P.

In the

As AHK, CPK.
/.AHK- -CPK, alt. _ s.
i_HAK^ 1.PCK, alt. L s.
AK = KC, given.

AAHK-ACPK.

.'.

/.

But
.'.

Also

CP

is

CP = AH.
AH = BH,
CP = BH.

drawn

/. the lines

given.

parallel to

BH.

CP, BH are equal and parallel.


BCPH is a parallelogram.

.".
.'.

Also

HK = KP
But
.'.

HK Is

parallel to

from congruent

HP = BC oppf sideft
HK-JBC.

BC.

triangles.

of parallelogram.
Q.E.D.

PROOFS OF THEOREMS

233

THEOREM 30
If there are three or

cepts

more

parallel straight lines,

made by them on any

and

if

straight line cutting

the inter-

them

are

equal, then the intercepts made by them on any other

straight line that cuts

them

are equal.

Given three parallel lines cutting a line

PT

other line

at Q, R,

AE at B, C, D
BC = CD.

S and that

and any

To Prove

QR^RS.
Draw BH, CK parallel to PT to meet
Then BH is parallel to CK.
in the AS BCH, CDK.
L CBH ^ L DCK corresp.
L BCH = L CDK corresp.

OR,

DS

at H, K.

.'.

BC=^CD,

a.

s.

given.

ABCH = ACDK.

.'.

/.

But

BQRH

is

BH-CK,
a ||gram since
.'.

And CRSK

is

/.

opposite sides are parallel.

BH=QR.

a ||gram since
:.

its

its

opposite sides are parallel.

CK-RS.
QR = RS.

Q.E.D.

For

riders

on Theorems 29, 30 see page 52.

CONCISE GEOMETRY

234

BOOK

III

THKORKM
(1)

The

straight line

31

which joins the centre of a

middle point of a chord (which

(2)

The

is

circle to the

not a diameter)

is

per-

pendicular to the chord.


line drawn from the centre of a circle perpendicular to a
chord bisects the chord.

FIG. 69.

(1) Given a circle, centre O,


is N.

To Prove u

ONA

Join OA, OB.


In the As ONA,

/.

is

and a chord AB, whose mid-point

a right angle.

ONB,

OA = OB, radii.
AN = BN, given.
ON is common,
ONA = A ONB.

i.ONA= -LONB.
But these are adjacent angles,
/.

.".

each

ON

is the
(2) Given that
perpendicular
circle to a chord AB.

To Prove

that

is

a right angle.

of a

the mid-point of AB.

In the right-angled triangles ONA,

/.

is

from the centre

ONB.

OA = OB, radii.
ON is common.
A ONA = A ONB.
/. AN = NB.
*

Q.E.D.

THEOREMS

PJ800FS OF

235

THKOBEM 32
In equal

same

circles or in the

circle

(1) Equal chords are equidistant from the centres.


(2) Chords which are equidistant from the centres are equal.

FIG. 258(1).

(1)

Given two equal

circles

ABX, CDY,

centres P, Q,

equal chords AB, CD.


To Prove that the perpendiculars PH,

CD

QK

from

P,

and two

to

AB,

are equal.

Join PA, QC.


Since PH, QK are the perpendiculars from the centres to the
chords AB, CD, H and K are the mid- points of AB and CD.

AH = JAB

/.

and

CK = 'CD.

But

AB = CD,
AH = CK.

/.

given.

in the right-angled triangles

.*.

PA = the

hypotenuse QC,

.'.

A
.'.

PAH, QCK,

the hypotenuse

radii of equal circles.

AH = CK, proved.
PAH 53 A QCK.
PH = QK.
Q.E.D.

(2) Given that the perpendiculars

To

PH,

QK from

P,

to the chords

CD

are equal
Prove that AB = CD.

AB,

In the right-angled triangles PAH* QCK, the hypotenuse


PA = the hypotenuse QC, radii of equal circles.

PH =* QK,
.'.

given.

A PAH B A QCK.
/.

AH-CK.

CONCISE GEOMETRY

236

But the perpendiculars PH, QK bisect AB, CD.


AB = 2AH and CD = 2CK.
.'.
AB = CD.
.'.
Q.E.D.

The proof

is

unaltered

if

the chords are in the same

circle.

FIG. 258(2).

For

riders

on Theorems 31, 32, see page 57.

THEOREM 33
The angle which an arc of a circle subtends at the centre is double
that which it subtends at any point on the remaining part
of the circumference.

/N
FIG. 259(2).

FIG. 259(1).

Given

AB

is

an arc of a

circle,

centre

FIG. 259(3).

is

any point on the

remaining part of the circumference.


To Prove L AOB = 2 L APB.
Join PO, and produce it to any point N.
Since
OA = P, L CAP = L OPA.

But

ext.

Z-NOA = int. L CAP 4- int. Z.OPA.

PROOFS OF THEOREMS
Similarly
/.

237

NOB - 2 L OPB.

adding in Fig. 259(1) and subtracting in Fig. 259(2), we


L AOB = 2 L APB.
have
Q.E.D.

Fig. 259(3) shows the case where the angle AOB


the proof for Fig. 259(3)
greater than 180
:

is

reflex, i.e.

is

the same

as for Fig. 259(1).

THEOREM 34
Angles in the same segment of a circle are equal.
in a semicircle is a right angle.
2)
1)

The angle

B-

'B
FIG. 76(1).
1)

Fio. 76(2).

Given two angles APB, AQB in the same segment of a


To Prove L APB = L AQB.
Let

O be the centre. Join OA, OB.


^ at centre = twice L
Then ^ AOB - 2 L APB.
<_ AOB = 2 L
and
AQB.
_ APB = L AQB.
.'.

circle.

at

Qce.

Q.E.D.
2)

Given

AB

a diameter of a

circle, centre O,

the circumference.

To Prove L APB = 90.

Fm.

77.

and P a point on

CONCISE GEOMETRY

238

L at centre twice L at
L. AOB = 2 L APB.
*=
_
is a straight line
since
AOB
Bat
AOB 180,
=
L
APB 90.
/.

O ce

Q.K.D.

THEOREM 35
(1)

The opposite angles of a

cyclic quadrilateral are supplementary.


is produced, the exterior

(2) If a side of a cyclic quadrilateral

angle

(1) Oiven

is

equal to the interior opposite angle.

ABCD

is

To Prove L
Let

ADC = 180.

be the centre of the

Let the arc

and

a cyclic quadrilateral.

ABC +

let

Join OA, OC.

circle.
Q

ADC

subtend angle X at the centre,


the arc ABC subtend angle y at the centre.

L ABC.
L. at centre = twice L at
^
ADC.
y
/. 2 L ABC + 2 L ADC = 360.
L ABC + /L ADC = 180.
.'.

Now
and

=2

FIG. 78.

Q.B.D.

239

PROOFS OF THEOREMS
(2) Given the side
duced to P.

AD

ABCD

of the cyclic quadrilateral

L PDC =* L ABC,
To Prove
Now L ADC 4- L PDC = 180, adj. angles,
and L ADC -f L ABC - 180, opp. L s cyclic quad.
L ADC + L PDC = L. ADC + L ABC.
.'.
L ABC.
L PDC
.'.
==>

For

riders

on Theorems 33, 34, 35

i*

pro-

Q.E.D.

see page 62.

THEOREM 36
two points subtends equal angles at two
other points on the same side of it, then the four points lie
on a circle.

(1) If the line joining

(2) If the opposite angles of a quadrilateral are supplementary,

then the quadrilateral

is cyclic.

FIG. 261.

(1) Given that


side of

APB =

AQB

where

P,

Q are points on the

same

AB.

To Prove that

A, P, Q,

If possible, let

the circle through A, B, P not pass through


cut AQ or AQ produced at X.
Join BX.

and

let it

lie

on a

circle.

Then L AXB = L. APB, same segment,


and t- AQB = L APB, given.
u AXB = L AQB.
that

is,

the exterior angle of the triangle

interior opposite angle,


/.

BQX

equals the

whicbjs impossible*

the circle through A,-B,

P must

pass through Q,

Q.RD.

CONCISE GEOMETRY

240
(2)

Given that in the quadrilateral ABCD, L ABC + L


To Prove that A, B, C, D lie on a circle.
If possible let the circle

D, and let

it

cut

AD

or

through A, B,

AD

ADC = 180.

not pass through


Join CX.

produced at X.

.D

Flu. 2G2.

Then L ABC + L AXC = 180, opp. L


But L ABC + L ADC - 180, given.
/. L AXC - L ADC.
That

this,

s cyclic

the exterior angle of the triangle

interior opposite angle,

quad.

CXD

equals the

which is impossible,
C must pass through D.

the circle through A, B,

.".

Q.E.D.

For

riders

on Theorem 36, see page 83.

THEOREM 37
In equal

circles (or in the

same

circle), if

two arcs subtend equal

angles at the centres or at the circumferences, they are equal.

Given two equal


(1) Given that

To Prove

circles,

that

ABP, CDQ,

centres H, K.

AHB = L CKD.
arc AB = arc CD
L

PKOOFS OF THEOREMS

241

Apply the circle AB to the circle CD so that the centre H


falls on the centre K and HA along KG.
Since the circles are equal, A falls on C and the circumferences coincide.

Since -

AHB =

_ CKD,

the arcs AB,


arc

CD

HB

falls

on KD, and B

falls

on D.

coincide,

AB = arc CD.

_ APB = L CQD.
(2) Given that
To Prove that arc AB = arc CD.
Now L. AHB ^= 2 L. APB, - at centre = twice - at Oce.
L CKD = 2 L CQD.
L APB = L. CQD, given.

and

But

*.

AHB =

arc

L.

CKD.

AB = arc CD.
Q.E.I).

THEOREM 38
In equal circles (or in the same circle), if two arcs are equal, they
subtend equal angles at the centres and at the circumferences.

Given two equal


arcs

ABP, CDQ,

centres H, K,

and two equal

AHB = L CKD.
(2) z.APB-iCQD.
Apply the circle AB to the circle CD so that the centre H falls
on the centre K and HA along K9
Since the circles are equal, A falls on C and the circumTo Prove

(1)

circles

AB, CD.
(1)

L.

ferences coincide.

AB = arc CD, /. B falls' on D


AHB coincides with" L CKD.
AHB
CKD.

But

arc

/.

_
-

.'.

16

L.

and

HB

on KD.
Q.E.D.

CONCISE GEOMETRY

242
(2)

=<_ CKD.
AHB = L CKD, just
L APB =
CQD.

But

L.

proved.

L.

.'.

Q.E.D.

THEOREM 30
In equal
(1)

same circle
two chords are equal, the arcs which they cut

circles or in the
if

off

are equal
(2)

if

two arcs are equal, the chords of those arcs are equal.

Given two equal


Given chord

(1)

To Prove

circles

ABP, CDQ,

centres H, K.

AB = chord CD.
arc AB = arc CD.

Join HA, HB, KC, KD.


In the As HAB, KCD,

HA ~ KC, radii of equal circles.


HB = KD, radii of equal circles.
AB = CD, given.
*

.".

.'.

AHAB-AKCD.

/.

^.AHB= Z-CKD.

the arcs AB,


the centres.
.*.

CD

arc

of equal circles subtend equal angles at

AB = arc CD.
Q.E.D.

(2)

AB = arc CD.
To Prove chord AB =fchordr CD.

Given arc

Since AB,

CD

are equal arcs of equal circles,

= LCKD.

PKOOFS OF THEOREMS
/. in

the

AS

243

HAB, KCD,

H A = KC,

radii of equal circles.

HB = KD, radii of equal


z.AHB= Z.CKD, proved.

circles.

AB-CD.
Q.E.D.

For

riders

on Theorems 37, 38, 39 see page 72.

THE TANGENT TO A CIRCLE

FIG. 265.

AB of a circle.
A and moves along
positions Q Q 2 Q 8

Let P be any point on an arc


Suppose a point Q starts at
towards P, taking successive
the lines

PQ V PQ 2 PQ 3

lf

the arc

AP

and draw

Also suppose a point R starts at B and moves along the arc BP


towards P, taking successive positions R 1? R 2 R 3
and draw
,

the lines

PR 1}

PRg,
All lines in the

PR 8

PQ system cut off arcsalong PA, the lengths of


which decrease without limit as Q tends to P.
All lines in the PR system cut off gjrcs along PB, the lengths of
which decrease without limit as R tends to P.

Produce AP, BP to X, Y.
All lines drawn from P in the angle

APY

or

BPX belong either

CONCISE GEOMETRY

244
to the

PQ

PR

system or to the

system, except

t/ie

single line

which

cuts off an arc of zero length.


This line is called the tawjent at P.

The tangent at P is therefore the lino CPD, which is the intermediate position between lines of the PQ system and lines of the

PR

system, arid cuts off an arc of zero length at P.

THEOREM 40
The tangent

to a circle is at right angles to the radius through the

point of contact.

FIG. 267(1).

Given

FIG. 267(2).

P is any point on a circle,


To Prove the tangent at P
Through
circle

centre O.

perpendicular to OP.
draw any line XPQY, cutting the
Join OP, OQ.
is

P, see Pig. 267(1),

again at Q.

t
.'.
.*.

,\

OP OQ,

radii,

L OPQf= L OQP.
their supplements are
equal,
<_

OPX

L OQY.

245

PROOFS OF THEOREMS

Now

the tangent at P is the limiting position of the line


XPQY, when the arc PQ is decreased without limit, so

Q coincides with P, see Fig. 267(2).


in Fig. 267(2), z_OPX
2LOPY.
But these are adjacent angles, .*. each is a right angle.
/. in Fig. 267(2), L.
90, where PX is the tangent at P.
that

.'.

OPX

Q.E.D.

THEOREM 41
from the point of contact,
drawn, the angles which the chord makes with

If a straight line touches a circle and,

a chord

is

the tangent are equal to the angles in the alternate segments


of the circle.

"V

Y/

FIG. 268(2).

Fui. 268(1).

Given

YPX

is a tangent at P to the
chord through P.
To Prove L APX = L PKA and

circle

PLAK, and PA

L APY

In Fig. 268(1), draw through P any line


circle again at Q.
Join QA.

Then

L AQX = L PKA

ext.

is

L PLA.
YPQX cutting

of cyclic quad.

= int. opp.

any

the

/.

Now the tangent at P is the limiting position of the line


YPQX when the arc PQ is decreased without limit, so
that Q coincides with P, see Fig. 268(2).
But the limiting position of /. AQX is /. APX.
.". when YPQX becomes the
tangent at P,
Similarly

it

may

be} proved that Z.

APY= L PLA.
Q.E.D.

CONCISE GEOMETRY

246
The converse
For

of this theorem

is

frequently of use in rider- work.

on Theorems 40, 41, see page 68.

riders

THEOKEM 42
If

two tangents are drawn to a circle from an external point


(1) The tangents are equal.
(2)

(3)

The tangents subtend equal angles at the centre.


The line joining the centre to the external point

bisects

the angle between the tangents.

FIG. 86.

Given TP, TQ are the tangents from


To Prove (I) TP-TQ.

to a circle, centre O.

(2)
(3)

Since TP, TQ are tangents at P, Q, the angles


are right angles.
.".

in the right-angled triangles

OP = OQ, radii.
OT is the common

TOP,

TPO,

TQO

TOQ

hypotenuse.

and ZTOP=ZTOQ,
and Z.OTP = ZOTQ.
Q.E.D.

THEOREM 43
t

If

two

touch one another* the line joining their centres


(produced if necessary) passes through the point of contact.
circles

PROOFS OF THEOREMS
Given two

circles, centres

To Prove

AB

247

A, B, touching each other at P.

(produced

if

necessary) passes through P.

Fio. 87(2).

Since the circles touch each other at P, they have a

XPY

common

at P.

tangent
Since XP touches each circle at P, the angles

XA, XPB

are

right angles.
.".

A and B
to

each

on the

lie

line

through P perpendicular

PX.
.".

A, B,

lie

on a straight

line.

Q.E.D.

Note.

If

two circles touch each other externally

the distance

(Fig. 87(1)),

between their centres equals the sum of

the radii.
If

For

two circles touch each other internally (Fig. 87(2)),


the distance between their centres equals the difference
of the radii.
riders

on Theorems

42, 43, see page 77.

THEOREM 44
In a right-angled triangle, the line joining the mid-point of the
hypotenuse to the opposite vertex is equal to half the
hypotenuse.

Given

ABC

is

triangle, right-angled at A,

point of BC.

and

is

the mid-

CONCISE GEOMETRY

248

To Prow AD
Draw a circle through
Since

/.

A, B, C.
is a diameter.

BAG = 90, BC

FIG. 269.

But D

is

the mid-point of BC,

.".

is

the centre of the

circle.

.'.

DA = DB = DC,
DA-JBC.

radii.

Q.E.T).

DEFINITION.

If

a point moves in such a

way

given geometrical condition, the path traced oat

that

it

obeys a

by the point

is

called the locus of the point.

THEOREM 45
The

which is equidistant from two given points,


the perpendicular bisector of the straight line joining the

locus of a point,
is

given points.

A
,

B and any
PA = PB.

Given two fixed points A,

moves so that
To Prove that P
Bisect

AB

at N.

lies

FIG. 105.

position of a point

P which

on the 'perpendicular bisector of AB.

Join PN.

249

PROOFS OF THEOREMS
In the

AS ANP, BNP,
AN = BN, constr.
AP = BP, given.
PN is common.

AANPsABNP.

.'.

L ANP =

.'.

But these are adjacent

PN
.*.

lies

is

angles,

Z.

BNP.

.*.

each

perpendicular to

AB

is

and

a right angle.

bisects

it.

on the perpendicular bisector of AB.


Q.E.D.

THEOREM 46
The locus of a point which

is

equidistant from two given interwhich bisect the

secting straight lines is the pair of lines

angles between the given lines.

Fin. 106.

COD

and any position of a point P


which moves so that the perpendiculars PH, PK from P to

Given two fixed lines

AOB,

COD

To Prove P

AOB,

are equal.
lies

on one of the two

BOC, BOD.
Suppose P is situated

in the angle

lines bisecting the angles

BOD.

In the right-angled triangles PHO, PKO,

PH = PK,

PO

is

the

given.

common

hypotenuse.

/.

A PHO s A PKO.

/.

L POH = L ROK.

on the line bisecting the angle BOD.


In the same way if P is situated in either of the angles
BOC, COA, AOD, it Jj#s on the bisectors of these angles.
.".

For

lies

riders

on Theorems 45, 46, see page 94.

QBD
'

'

'

CONCISE GEOMETRY

250

THEOREM 47
The perpendicular
concurrent

bisectors of the three sides of a triangle are

(i.e.

meet

in a point).

Given that the perpendicular bisectors OY,

OZ

of

AC,

AB

meet

at O.

To Prove the perpendicular

BC at X, join OX
the As OZA, OZB,

Bisect

In

BZ

bisector of

also join

BC

passes through O.

OA, OB, OC.

ZA, given.

OZ is common.
/. BZO = /. AZO, given rt.
/.
A OZA -A OZB.
OA-OB.
Similarly from the AH OYA, OYC, it can
OA = OC,
OB - OC.
In the As OXB, OXC,
OB = OC, proved.
XB = XC, constr.

</ s.

.'.

be proved that

.'.

OX

is

common.

/.

AOXB AOXC.

/.

L OXB -/. OXC.

==

But these are adjacent


.*.

For

riders

OX

is

angles,

.".

each

is

rt.

the perpendicular bisector of BC.

on Theorem 47, see page 99.

Q.E.D.

PROOFS OF THEOREMS

251

THEOREM 48
The

internal

the

of

bisectors

three

angles

of

a triangle are

concurrent.

PC

FIG. 109.

Given that the internal bisectors

meet at I.
To Prove that
Join A.

In the

Draw

As

IB,

1C of the angles ABC,

bisects the angle

IP,

ACB

BAC.

IQ, IR perpendicular to BC, CA, AB.

IBP, IBR,

ZJBP=
ZIPB=
IB

/.

is

Z.IBR, given.

Z.IRB, constr.

Z.s.

= IR.
AS ICP, ICQ
IP = IQ,
IQ = IR.
IP

Similarly from the

.'.

rt.

common.

it

may be

proved that

In the right-angled triangles IAQ, IAR,


IQ =
IA

.'.

is

IR, proved.

the

common

hypotenuse.

ZJAQ=Z.IAR.
IA bisects the angle

BAC.
Q.E.D.

For

riders

on Theorem 48, see page M)0.

THEOREM 49
The

three altitudes of a triangle (i.e. the lines drawn from the


vertices perpendicular to the opposite sides) are concurrent.

CONCISE OEOMETBY

252

Given AD, BE, OF are the altitudes|of the triangle ABC.


To Prove AD, BE, CF are concurrent.

Through A,

B,

draw

lines

parallel to

BC, CA,

AB

to

form the triangle PQR.

P
FIG. 270.

AR and AC is to BR,
BCAR is a parallelogram.
BC^AR.
/.
= AQ,
Similarly, since BCQA is a parallelogram, BC
AR = AQ.
Since AD is perpendicular to BC, and since QR, BC
Since

BC

is

||

to

||

.'.

are

parallel,

AD is perpendicular to QR.
AD is the perpendicular bisector of QR.
AR = AQ,
and
BE
CF are the perpendicular bisectors
Similarly,
.'.

But

.*.

PR, PQ.
But the perpendicular

PQR

of

bisectors of the sides of the triangle

are concurrent
.*.

AD, BE,

CF

are concurrent.
Q.E.D.

For riders on Theorem 49, see page 101.

THEOREM 50
(1)

The three medians

of a triangle (i.e. the lines joining each


vertex to the middle point of the opposite side) are concurrent.

253

PROOFS OF THEOREMS
(2)

(1)

The point at which the medians intersect is


way up each median (measured towards the

Given the medians BE,

CF

of

the triangle

one-third of the
vertex).

ABC,

intersect

at G.

To Prove that AG, when produced, bisects BC.


Join AG and produce it to H, so that AG - GH.
Let AH cut BC at D ; join HB, HC.
Since AF = FB and AG = GH,
FG is parallel to BH.
Since AE = EC and AG = GH,
EG is parallel to CH.
Since FGC and EGB are parallel to BH and CH,
.*.

BGCH is a parallelogram ;
the diagonals BC, GH bisect each other ;
/.

BD-DC.
Q.E.D.

(2)

For the same reason,

GD = DH.
But AG*=GH.
C.
/.

or

AG-2GD.
AD-3GD.
GD

For riders on Theorem 50* see page 103.

254

CONCISE GEOMETRY

BOOK

IV

THEOREM 51
If

two

triangles have equal heights, the ratio of their areas

is

equal to the ratio of their bases.

Y
FIG. 113.

XYZ

Given two triangles ABC,

having equal heights AP, XQ.

A ABC = BC
The area

of a triangle
.'.

and

"
But

AP = XQ,

height x base.

AABC=JAP.BC.
AXYZ-=JXQ.YZ,
AABC = |AP.BC
A"XYZ""|XQ7YZ'

given,

"
.

AABC = BC
A XYZ Yr
Q.E.D.

THEOREM 52
(1) If a straight line is drawn parallel to one side of a triangle, it
divides the other sides (produced if necessary) proportionally.
(2) If a straight line divides two sides of a triangle proportionally,
it is

parallel to the third side.

H
FIG. 114(2).

FIG. 114(3).

PROOFS OF THEOREMS

BC

(1) Given a line parallel to


at H, K.
ni
Prove
To

cuts

AC

AB,

255

(produced

if

necessary)

AH = AK
----HB KC

Join BK, CH.

The
to

KHA, KHB have a common

triangles

to

HKA,

triangles

HKC

have a

common

AC.

AHKA ~= AK

A H KC

A KHB, AKHC are

base

HK

Given a line

KG*

equal in area, being on the same

and between the same

parallels

HK, BC.

AH_AK

"
(2)'
V

altitude from

AKHB~HB'

But

AKHA^AH

"
The

altitude from

AB.

HB'KC'

HK cuttingb

AB,

AC

at H,

such that

To Prove HK is parallel to BC.


The triangles KHA, KHB have a common
to AB.

to

triangles

HKA,

altitude from

from

AKHA^AH
AKHB HB'

The

AH = AK
HB KC

HKC

have a

common

altitude

AC.

AHKA^AK

AHKC~~KC'
AH- = AK
given.
HB KG' b

u
J3ut

AKHA _ AHKA

But these triangles are on the same base


*
same side of it.
.*.

HK

is

parallel to

HK

and on the

BC.
Q.B.D.

CONCISE GEOMETRY

256
COROLLARY

a line

If

1.

HK

cuts

AB,

AHAK

AC

at H,

so that

Now

HB

A1
A1S

AB

AH == KC AK
X
HB AC iTc

AH_AK
AB~AC*
COROLLARY

If

2.

a lino

AH = AK
Then
AB AC
COROLLARY

and

If a line

3.

A -K

AB
For

riders

HK
,

rn,

AC

BC cuts AB, AC at H,
HB = KC
AB AC

parallel to

HK
then

K,

cuts AB,

HK

is

AC

at H,

parallel to

so that

BC.

on Theorems 51, 52 see page 106.

THEOREM 53
If

two

triangles are

equiangular, their

corresponding sides are

proportional.

A
X

C
FIG. 272.

.PROOFS OF THEOREMS
Given the triangles ABC,

XYZ

XY XZ

YZ

are equiangular, having

From AB, AC cut off AH, AK


In the As AHK, XYZ,

AH = XY,
AK - XZ,

equal to XY, XZ.

But /.XYZ =

,/

Z.A

Join HK.

constr.
constr.

ZHAK- Z.YXZ,
AAHK-AXYZ.

/.

257

ABC,

given.

given.

Z.AHK=Z.ABC.

.'.

But these are corresponding

angles,

HK

.".

is

parallel to

BC.

" AB_AC
.

But

AH - XY

and
.

Similarly

DEFINITION.

it

AH~AK*
AK = XZ.
A]3 ~
= AC
XY XZ'

can be proved that

If

AC = BC
-

two polygons are equiangular, and

if

their

corresponding sides are proportional, they are said to be similar.


Theorem 53 proves that equiangular triangles are necessarily
similar.

THEOREM 54
one triangle are proportional to the three sides
of the other, then the triangles are equiangular.

If the three sides of

Y,

c
Fio. 273.

CONCISE GEOMETRY

258
Given the

AS ABC, XYZ

are such that


/v T

^o Prove Z.A= Z.X,

ZB=

Z.Y,

- --- = ~y'
i

Z.C =

z.

2./V

Z.Z.

YZ opposite to X, draw YP and ZP so that


/.ZYP- Z ABC and Z.YZP= ZACB.
Since the AS ABC, PYZ are equiangular, by construction,
On

the side of

AB_BC
YP~YZ'

AB _ AB

VD
Tr
AT.
ZP = XZ.

Similarly
/. in the

AS XYZ, PYZ.
XY-PY, proved.
XZ = PZ, proved.
YZ is common.

AXYZ^APYZ.
But

ZLXYZ- ZPYZ and Z.XZY=ZPZY.


Z.PYZ- Z.ABC and /.PZY= Z.ACB,
L XYZ = z. ABC and L XZY = L ACB.

/. also Z.

YXZ = Z BAG.

constr.

Q.E D.

THEOREM 55
If

two triangles have an angle of one equal to an angle of the


other, and the sides about these equal angles proportional,
the triangles are equiangular.

HA.

XK

259

PROOFS OF THEOREMS
Given in the triangles ABC, XYZ,

Z.BAC- Z.YXZ

and

AC
.

j\ i
XZ'
To Prove /.ABC = Z.XYZ and Z.ACB = Z.XZY.
From AB, AC, cut off AH, AK equal to XY, X7. Join HK.
In the As AHK, XYZ,
AH = XY, constr.
AK = XZ, constr.

Z.HAK- ZYXZ,

given.

AAHK-AXYZ.
/.AHK- Z. XYZ and /AKHr-^XZY.
A B AC
^
d XY ^ AH, XZ - AK.
XY XZ

Now

AB_AC
~
AH

.".

HK

is

AK'
BC.
and
- Z.XYZ and

parallel to

L AHK =

ZAHK

Miit

/.

ABC

L ABC - L XYZ
For

riders

and

Z AKH = /_ ACB, corrosp.


_ AKH - Z.XZY, proved.
L ACB = L XZY.

on Theorems 53, 54, 55 see page 1J2.

/. s.

Q- K - !)

THEOREM 56
two chords of a circle (produced if necessary) cut one another,
the rectangle contained by the segments of the one is equal to
the rectangle contained by the segments of the other.
If
from any point outside a circle, a secant and a tangent are
(2)
drawn, the rectangle contained by the whole secant and the part

(1) If

of it outside the circle

129(1).

is

equal to the square on the tangent.

Fio. 129(2).

CONCISE GEOMETRY

260
(1)

Given two chords AB, CD intersecting at O.


To Prove OA OB = OC OD.
.

Join BC, AD.


In the As ADD,

BOG,

Z.OAD= Z.OCB,

same segment,

in the

Fig. 129(1)

and Fig 129(2).

ZAOD=

,/

vert. opp. in Fig. 129(1),

COB,

same

L ODA = the

L in Fig.
L OBC.

129(2).

third

.'.

the third

.".

the triangles are equiangular.

OA_OD
/.

6C~OB*
OA.OB = OC.OD.

(2) Given a chord

AB

Q.E.D.

meeting the tangent at

in O.

Join AT, BT.

FIG. 180.

In the

As AOT, JOB,

^TAO=
.'.

/.

Z.BTO,

alt.

segment.

Z.AOT= Z.TOB, same angle.


the third ,/ ATO = the third Z.TBO.
the triangles are equiangular.

OA^OT,
~
OT OB'
/.

OA.OB = OT2

Note.

This

may

limiting case

Q.E.D.

also be

when D

deduced from (1) by taking

coincides with

in Fig. 129(2).

tlje

PKOOFS OF THEOREMS
The converse properties are as follows
If two lines AOB, COD are such that
(i)

261

then A, B, C,
(ii)

If

two

lines

OBA,

then A, B, C,
(iii)

If

two

lines

lie

lie

OT

on a

OD,

OD,

circle.

ODC are such that OA OS = OC

OBA,

on a

AO OB -= CO

circle.

OA OB = OT2
T touches OT at T.

are such that

the circle through A, B,


These are proved easily by a reductio

then

ad absurdum method.

THEOREM 57
If

AD

is

an altitude of the triangle ABC, which


(i)

AD 2 =BD

(ii)

BA 2 =BD

A, then

is

right-angled at

DC.
BC.

C-

(1) Since /.

BDA = 90,

add up

to

/.

But
/.
.'.

/.

.'.

in the

the

the remaining angles of the triangle

ABD

90.

L DAB 4- Z DBA = 90.


L DAB + L DAC = 90, given.
L DAB + L DBA = L DAB + L DAC.
Z.DBA=/.DAC.

AS ADB, CDA,

Z.ADB= /.CDA, right angles.


L DBA = Z. DAC, proved.
third L BAD = the third /. ACD.
.".

the triangles are equiangular.

AD_BD
*
DC"" DA'
BD'.DC.

Q.E.D.

262
(2)

CONCISE GEOMETRY

In the

ADB, CAB,

L ADB=

Z.CAB,

right angles.

Z.ABD= Z.CBA, same angle.


the third /. DAB - the third L ACB.

.'.

the triangles are equiangular.

AB_BD
BC~AB
AB 2 =BD BC.

.'.

An

Note.

Also

For

AD

is

BA

is

riders

Q.E.D.

method

alternative

of proof

is

given on page 121.

mean proportional between BD and DC.


the mean proportional between BD and BC.

called the

on Theorems 56, 57 see page 122.

THEOREM 58
The

ratio of the areas of

ratio of the squares

two similar

triangles

on corresponding

is

equal to the

sides.

FIG. 133.

Given the triangles ABC,

A XYZ
Draw
In the

XYZ
YZ 2

the altitudes AH,

XK.

As AHB, XKY,
Z.ABH=ZXYK,

ZAHB=
.'.

are similar.

Z.XKY,

given.
rt.

Z.s constr.

L BAH - the third L YXK.


As ArtB, XKY are similar.

the third
/.

the

"

AH_AB
XK~KY

But

AB = BC

*
,

since

AS ABC, XYZ

are similar.

PROOFS OF THEOREMS

"
.

But

AH_BC
XK~YZ"

AABC-^AH BC and AXYZ-^XK


.

'

A ABC ~ AH

'

AXYZ XK
AH

But

BC

YZ'

YZ.

BC

AABC_BC 2
Q.E.D.

two polygons are similar, it can be proved that they can be


up into the same number of similar triangles.
Hence it follows that the ratio of the areas of two similar
is equal to the ratio of the squares on corresponding
If

divided

polygons
sides.

THEOREM 59
If three straight lines are proportionals, the ratio of the area of
any polygon described on the first to the area of a similar

polygon described on the second


first line

is

equal to the ratio of the

to the third line.

FIG. 134.

Given three

lines

figures

_
m Prove
To

AB, CD, EF such that

ABP, CDQ.

figure

figure

ABP
~~ = AB

==
CDQ EF

Since the figures are similar,


figure
figure

ABP ^AB8
2

CDQ ""CD

'

and two

similar

CONCISE GEOMETRY

264

CD 2 = AB

But

AB 2 *_

EF, given.

AB 2

AB

CD ~AB7EF~EF'
2

ABP_ AB
figure CDQ~EF'
figure

Q.E.D.

For

riders

on Theorems 58, 59 see page

1127.

THEOREM 60
(1) If the vertical angle of a triangle

externally by a straight

produced,

it

line

is

bisected internally or

which cuts the base, or the base

divides the base internally or externally in the

ratio of the other sides of the triangle.


If
a straight line through the vertex of a triangle divides the
(2)

base internally or externally in the ratio of the other sides,


it bisects the vertical angle internally or externally.

D
FIG. 275(1).

FIG. 275(2).

(1) Given the line

AD

bisecting the angle

BAC,

Fig. 275(1), externally in Fig. 275(2),

internally in
BC or BC

meets

produced at D.

BD

BA
c
draw
CP parallel to DA to meet AB or AB
C
Through
BA is produced to E in Fig. 275(2).
duced at P.
In Fig. 275(1). ZBAD= ZAPC, corresp. /.s.
Z.DAC= L AGP, alt. /.s.
But
BAD = /_ DAC, given.
Z.APC-Z.ACP.
In Fig. 275(2). Z.EAD = Z.APC, corresp. Z.s.
?>
To Prove

^~

Z.DAC*= Z.ACP,

alt.

L*.

pro-

265

PROOFS OF THEOREMS
Z. EAD = Z DAC,
Z.APC-ZACP.
in each case, AP = AC.
But CP is parallel to DA.
BA ""_ BD
AP DC'

But

given.

Q.B.D.

AD

2)' Given that

To Prove
Through
duced
XT

Now
T>

that

BC

or

BC

produced so that
L

BD
DC

AD bisects Z. BAC internally or externally.


CP parallel to DA to meet AB or AB pro-

draw
n

parallels.
L

i.

But

.*.

BA = -BD
AP DC
BA = BD

BA ~~ BA
AP AC*
AP = AC.
/.
Z.APC-ZACP.

'

In Fig. 275(1) Z.APC-- Z.BAD, corresp.

Z_s.

APC = Z. ACP,
Z BAD - Z DAC.
275(2) Z. APC = ZEAD,

Z.s.

But

BA

AC

at P.

by
J

cuts

Z.

proved.

.'.

In Fig.

corresp.

ZACP=
1

Z.DAC, alt. Z..


Z. APC - Z ACP, proved.

But
.'.

/.

Z.

EAD = /_ DAC.
AD bisects Z.BAC
AD bisects Z.BAC

in Fig. 275(1),
in Fig. 275(2),

internally,

and

externally
Q.E.D.

For

riders

on Theorem 60 see page 152.

CONSTRUCTIONS FOR BOOK


CONSTRUCTION

From a given

point in a given straight line, draw a straight line


with
the given line an angle equal to a given angle.
making

FIG. 276.

on a given line AB and an angle XYZ.


To Construct a line AC such that Z.CAB= Z XYZ.
With centre Y and any radius, draw an arc of a

Given a point

cutting YX, YZ at P, Q.
centre A and the same radius,
EF, cutting AB at E.

With

With
a

centre

circle,

Join

Then

Proof.
In the

radius equal to
EF at F.

arc of a circle

QP, describe an arc of

cutting the arc

AF and

AC

E and

draw an

circle

produce

it

to C.

the required
Join PQ, EF.
is

line.

As PYQ, FAE,
YP = AF, constr.
YQ = AE, constr.
PQ = EF, constr.
</XYZ=/.CAEf.
Q.E.F.
267

CONCISE GEOMETRY

268

CONSTRUCTION 2
Bisect a given angle.

Fi.
Given an angle BAG.
To Construct a

With A

277.

line bisecting the angle.

as centre

and any

radius,

draw an arc

of a circle,

cutting AB, AC at P, Q.
With centres P, Q and with any sufficient radius, the same

draw

for each,

Then AR
Proof.

In the

is

arcs of circles, cutting at R.


the required bisector.

Join AR.

Join PR, QR.

As

APR, AQR,
AP = AQ, radii of the same circle.
PR = QR, radii of equal circles.
AR is common.

,/PAR=/.QAR.
Q.E.F.

CONSTRUCTION 3

Draw

the perpendicular bisector of a given finite straight

FIG. 278.

line.

CONSTRUCTIONS FOR BOOK


Given a

finite line

269

AB.

To Construct the line bisecting AB at right angles.


With centres A, B and any sufficient radius, the same
each, draw arcs of circles to cut at P, Q.
Join

PQ

Then

and

cut

let it

AB

at C.

the mid-point of AB, and

is

for

PCQ

AB

bisects

at

right angles.

Proof.
In the

Join PA, PB, QA, QB.

As PAQ, PBQ,
PA = PB,

radii of equal circles.

QA = QB,

radii of equal circles.

PQ

common.

APAQ-APBQ.

.'.

Z.APQ=Z.BPQ.

/.

In the

is

As

APC, BPC,

PA = PB, radii of equal


PC is common.
L APC = Z. BPC, proved.
AAPC = ABPC.

.'.

circles.

AC^CB.
and

ACP=

Z.BCP.
But these are adjacent angles,
/.

.*.

each

is

a right angle.
Q.E.P.

CONSTRUCTION 4

Draw

a straight line at right angles to a given straight line from


a given point in it.

'P

/Q

Fio. 279.

Given a point

on a

line

AB.

from C perpendicular to AB.


To Construct a
With centre C and any* radius, draw an aroof a
line

cutting

AB

at P, Q.

circle

CONCISE GEOMETRY

270
With centres
each, draw

Then

CR

Proof.
In the

P,

and any

sufficient radius, the

arcs of circles to cut at R.

same

for

Join CR.

the required perpendicular.

is

Join PR, QR.

As

RCP, RCQ,
RP = RQ, radii of equal circles.
CP = CQ, radii of the same circle.
CR is common.

.'.

A RCP = A RCQ.

/.

L RCP -Z. RCQ.

But these are adjacent

angles,

.*.

each

is

a right angle.
Q.K.F.

CONSTRUCTION 5

Draw a

perpendicular to a given straight line of unlimited length

from a given point outside

it.

FIG. 280.

Given a line

AB and

a point

outside

it.

To Construct a line from C perpendicular to AB.


With C as centre and any sufficient radius, draw an

arc of a

cutting AB at P, Q.
as centres and any sufficient radius, the same for
With P,
draw
arcs of circles, cutting at R.
Join CR and let
each,
circle,

it

cut

AB

Then-*X
Proof.

is

at X.

perpendicular

Join CP,

t<?

AB.

CQ, RP, RQ.

CONSTRICTIONS FOK BOOK


In the

As CPR, CQR,
CP = CQ, radii of the same circle.
RP ~ RQ, radii of equal circles.
CR is common.

In the

AS CPX, CQX,
CP = CQ,

CX

is

271

radii.

common.

ZPCX- ^QCX,
ACPX ACQX.

proved.

==

.'.

But these are adjacent

angles,

each

.".

is

a right angle.
Q.E.P.

CONSTRUCTION 6
'hrough a given

i>oint,

draw a

straight line parallel to a given

straight line.

9
J

Fro. 281.

liven

AB

line

and a point

outside

it.

To Construct a line through


parallel to AB.
With C as centre and any sufficient radius, draw an
a

PQ, cutting AB at P.
as centre and the same radius,

arc of

circle

With P
circle,

cutting

AB

cutting the arc

PQ

at

to CR, draw an arc of a


on the same side of AB as

Join CQ.

C.

Then

CQ

Proof.
In the

is

parallel to

AB.

Join CR, CP, PQ.

As CRP, PQC,
CR = PQ, constr.
RP =* QC radii'of
PC

is

arc of a

at R.

With centre P and radius equal


circle,

draw an

common.

equal

circles.

CONCISE GEOMETRY

272

Z.CPR-Z.PCQ.

.'.

But these are alternate

angles,

.".

CQ

is parallel

to RP.
Q.E.F.

CONSTRUCTION 7

Draw

a triangle having

any two

of

its

sides equal to three given straight lines,

which are together greater than the third

side.

-\(1

id

Bj

FIG. 282.

Given three

lines a, b y

To Construct a
a, by

c.

triangle

whose sides are respectively equal to

c.

Take any line AX, and with A as centre and radius equal to
c, draw an arc of a circle, cutting AX at B.
With A as centre and radius equal to 6, draw an arc of a
circle; and with B as centre and radius equal to a, draw
an arc of a circle, cutting the former arc at C.
Join AC, BC.
Then ABC is the required triangle.
Proof.

By

construction,

AB = c.

Q.E.F.

CONSTRUCTION 8

Draw

a triangle, given two angles and the perimeter.

Y and

a* line HK.
To C&istruct a triangle having two of its angles equal
and Y and its perimeter equal to HK.

Given two angles X,

to

CONSTRUCTIONS FOR BOOK

QK

Construct lines PH,

on the same side of

Z.PHK= Z.X and ZQKH= Z.Y.


Construct lines HA, KA intersecting

at

273

HK

A and

such that

bisecting the

PHK, QKH.

angles

Construct through A, linos AB,


cutting HK at B, C.

AC

parallel

to

PH, QK,

ABC is the
L BAH
Proof.
Then

required triangle.
/.

AHP,

Z.BHA= Z.AHP,

since

AB

is

parallel to

PH.

constr.

BH = BA.
may be proved that CK = CA.
AB -f BC + CA = HB + BC + CK - HK.
Also Z.ABC= /.PHK^ Z.X, corresp.
and Z.ACB= Z.QKH - /.Y, corresp.

Similarly
/.

it

.".

ABC

is

/.s.
/.s.

the required triangle.


Q.E.F.

CONSTKUCTION 9

Draw a

triangle given one angle, the side opposite that angle


sum of the other two sides.

the

>

B/*'

.'
P

A^
Fio. 284.

18

and

CONCISE GEOMETRY

274
Given two

lines a,

p and an

To Construct a

Draw

triangle

PQ

a line

circle,

as centre

cutting

bisector of

Then

ABC

Proof.

ABC

such that

PB

it

and radius equal

PB

PB and

BC = a, BA + AC =j

cut off a part PC equal to p.


such that L BPC equals -J L X.

and from

Construct a line

With

angle X.

at

to a,

draw an arc

of a

Construct the perpendicular


meet PC at A. Join AB, BC.

B.

let it

the required triangle


Since A lies on the perpendicular bisector of PB,
is

/.
.'.

Also

and
.".

AP-AB.
ZAPB=Z.ABP.
L BAC - L APB + Z. ABP.
= 2ZAPB.
= Z X since Z APB
AB + AC = AP + AC-PC =
BC = a, by construction.
ABC is the required triangle.

-=

L X.

/>.

Q.K.F.

Note.

Since there are two possible positions of B, namely,


B there are two triangles which satisfy the

B and

given

conditions.

CONSTRUCTION 10
Given the angle BAC, construct points P, Q on AB, AC such that
PQ is of given length and the angle APQ of given size.

H B

P\

Fio. 285.

Given the angle BAC, a link

and an angle X.

To Construct points P, Q*on AB,


I and _ APQ
equals /.X.

AC

such that

PQ

equals

CONSTRUCTIONS FOR BOOK


Take any point H on

Z.AHK=
From HK

AB

and construct a

cut off

By

line

HK

such that

^.X.

HF

Through F draw FQ
Through Q draw QP

equal to I.
parallel to AB to cut AC in Q.
parallel to FH to cut AB in P.
Then PQ is the required line.

Proof.

275

construction,

is

a parallelogram,

ZQPA = ^FHA=

and
.*.

PQFH

PQ

is

the required

Z.X,

line.

Q.E.F.

CONSTRUCTION 11
Describe a square on a given straight

line.

**

-t-

A
FIG. 286.

Given a line AB.

To Construct a square on AB.


From A draw a line AC perpendicular
off

AP

to

AB

Through P draw PQ parallel to AB.


Through B draw BQ parallel to AP, cutting
Then ABQP is the required square.
Proof.

from

AC

equal to AB.

By

But
But

construction,

L BAP = 90,
AB = AP,

PQ

at Q.

ABQP is a parallelogram.
ABQP is a rectangle.
ABQP is a square.

/.
.*.

Q.K.F.

cut

CONSTRUCTIONS FOR BOOK

II

CONSTRUCTION 12
(1)

Reduce a quadrilateral to a triangle of equal

(*J)

Ueducu any given

rectilineal figure to

area.

a triangle of equal area.

D
JC

)\Given a quadrilateral ABCD.


To Construct a triangle equal in area to

it.

Join BD.

Through C, draw CK parallel to DB to meet AB produced


at K.
Join DK.
Thou ADK is the required triangle.
The triangles BCD, BKD are on the same base BD
Proof.
and between the same parallels BD, KC.
.".

Add
.".

/.

area of
to each

ABCD = area of ABKD.


AABD.

area of quad. ABCD area of


AKD is the required triangle.

FIG. 288.
276

A AKD.
Q.E.F.

FOR BOOK
ABODE.

(2) Given a pentagon

To Construct a
Proceed as in
lateral

triangle equal in area to


(1).

AKDE

Proceed as in

277

IT

it.

This reduces the pentagon to a quadri-

of equal area.

(1).

EK; through D draw DL

Join

parallel

EK to meet AK produced in L.
Then AEL is the required triangle.
to

This process can be repeated any number of times.

C< >NSTIU ICTTON

Bisect a triangle by a line through a given point in one sid<\

A
'

0,

PC

FIG. 289.

Given a point P on the side BC of the triangle ABC.


To Construct a line PQ bisecting the triangle.
Supi>ose P is nearer to
Join
Bisect AB at H.

than B.

PH.

Through C, draw CQ parallel


Join PQ.
Then PQ is the required line.
Join CH.
Proof.

to

PH

to

AH = HB, area of AAHC - area


ABHC = JAABC.
Since HP is parallel to QC.
Area of AHPQ = area of AHPC.
Add to each, ABHP.
area of A BPQ = area of ABHC.
But ABHC = |AABC.
Since

meet

of

AB

at Q.

ABHC.

.'.

.*.

/.

ABPQ=JAABC.

.'.

PQ

bisects

A ABC.
Q.B.F.

CONCISE GEOMETRY

278
If

it

required to draw a line PQ, cutting off from the triangle


ABC a triangle BPQ equal to a given fraction, say |, of the

is

triangle

ABC, take a

and proceed as

in the

point

H on BA

such that

BH = |BA

above Construction.

CONSTRUCTION 14
Divide a given straight line into any given number of equal parts.

c
h''
,KX \

F-

A
A

FIG. 290.

Given a line AB.

To Construct

points dividing

AB

into

any number (say 5)

equal parts.

Through A, draw any line AC.


Along AC, step out with compasses equal lengths, the number
of such lengths being the required number of equal parts
(in this case 5).

Let the equal lengths be AF, FG, GH, HK, KL.


Join LB, and through

meeting

AB

F, G, H,
at P, Q, R, S.

K draw

lines parallel to

BL,

Then AP, PQ, QR, RS, SB

are the required equal parts.


Since the parallel lines FP, GQ, HR, KS, LB cut off
equal intercepts on AC, they cut off equal intercepts on AB.

Proof.

Q.E.F.

CONSTRUCTION 15
Divide a quadrilateral into any number of equal parts by lines
through one vertex.

Given a quadrilateral

ABCD,

To Construct lines through A which divide


number (say 5) equal parts.
Join AC; through D draw DP parallel to
produced at P.

ABCD
AC

to

into

any

meet

BC

CONSTRUCTIONS FOR BOOK


Divide

BP

into the required

number

279

II

(in this case 5) of equal

Q H Q 4 Q 4 P.
parts, BQ V QjQ 2 Q 2 Q
?
Through those points which lie on BC produced,
Q 3 Q 4 draw lines Q 3 R 3 Q 4 R 4 parallel to PD
,

in this case

to

meet

CD

R3 R

in

Then

AQ^ AQ 2 AR 3 AR 4

are the required lines.


and the quad.
construction, the
,

Q 2 AQ 3 Q 3 AQ 4 Q 4 AP

AS

But the areas

ABCD

AABP

Proof.
By
are equal in area.

of
BAQ^ QjAQ,,,
are equal, fc* their bases are equal

and they have the

same height.
each

/.

Further,

= I quad. ABCD.
1 A ABP
AACQ 3 =AACR AACQ = AACR4 AACP =

AACD,

3,

being on the same base and between the same

parallels.

/.

AAR 3 R 4
AAQ Q4
3

And

AACR

AACQ 4

AAR4 D = AAQ 4 P.
AQ2 CR 3 = AAQ 2 C + AACR
AACQ

AQ AQ 2 AR 3 AR 4
1?

parts.

Note.

AACR 4

similarly

Also quad.

.*.

=r

divide quad.

ABCD

into five equal


Q.E.F.

The same method may be used


figure with any number of sides

a.

for dividing a rectilineal

any number of equal


bylines through a vertex or by Jines through
a given point on one of the sides.
parts, either

into

CONSTRUCTIONS FOR BOOK

III

CONSTRUCTION 16
Construct the centre of a

circle,

an arc of which

Ls

given.

A
Km.
Given an arc

AB

292.

of a circle.

To Construct the centre of the circle.


Take three points P, Q, R on the arc.
Construct the perpendicular bisectors
intersecting at O.

Then

Proof.

is

OX,

OY

of

PQ, QR,

the required centre.

The perpendicular

bisector of a chord of a circle

passes through the centre of the circle.


the centre of the circle lies on
.*.
and on

OX

OY,

the centre

is

at O.
Q.E.F,

CONSTRUCTION 17
Construct a circle to pass
through three given points, which do not
He on a straight line.

*--.

j*.-

Fio. 293.
880

CONSTRUCTIONS FOR BOOK


Given three poin

281

III

_, _.

,,,

To Construct a circle to pass through A, B, C.


Construct the perpendicular bisectors OX, OY of AB, BC,
intersecting at O.

With
This

as centre

and

OA

as radius, describe a circle.

the required circle.


Since O lies on the perp. bisector of AB,
Proof.
is

OA = OB.
Since

lies

on the perp. bisector of BC,

OB = 00.
.".

the

circle,

centre O, radius

OA,

passes through B, C.
Q.E.F.

CONSTRUCTION 18
(1) Construct the inscribed circle of a given triangle.
(2) Construct an escribed circle of a given triangle.

FIG. 294.

Given a triangle ABO.

To Construct

(1) the circle inscribed in


(2) the circle

A ABC.

which touches

AB

produced,

AC

produced and-CC.

ABC, ACB and

(1) Construct the lines Bl, Cl, bisecting the angles

intersecting at

Draw IX
With

I.

perpendicular

as centre

toBC.

and IX as

radius, describe

CONCISE GEOMETRY

282

This circle touches BC, CA, AB.


Since lies on the bisector of
Proof.
I

/.

ABC,

equidistant from the lines BA, BC.


Since lies on the bisector of Z.ACB.
I

is

is

equidistant from the lines

CB, CA.

equidistant from AB, BC, CA.


the circle, centre I, radius IX, touches AB, BC,
is

CA.

AC to H, K. Construct the lines Bl r Cl^


the
bisecting
angles HBC, KCB and intersecting at A

(2) Produce AB,

Draw
With

X1
1

perpendicular to

and

as centre

This circle touches


Proof.

Since

BC.

as radius, describe a circle.

produced, AC produced and BC.


on the bisector of /_ HBC,

AB

lies

equidistant from BH and BC.


Since l lies on the bisector of /.KCB,
is

Ij

is

equidistant from

CK

and

CB

equidistant from HB, BC, CK.


the circle, centre 1? radius l X i touches HB,
\

is

BC, CK.
Q.E.F.

CONST RUCTION

(1) Construct a tangent to a circle at a given point

on the circum-

ference.
(2) Construct the tangents to a circle

Fi<4.

(1)

Given a point

from a given point outside

295.

on the circumference of a

circle.

tangent ac A to the circle.


Construct the centre
of the circle.
Join AO.

To

Orr*,stritt the

it

CONSTRUCTIONS FOR BOOK

283

III

Through A, construct a line AT perpendicular to AO.


Then AT is the required tangent.
The tangent is perp. to the radius through the point
Proof.
of contact. But AO is a radius and /_ OAT = 90,

AT

.".

(2)

Given a point

is

the tangent at A.

outside a circle.

To Construct the tangents from


Construct the centre

With

at F.
let it

to the circle.

of the circle.

Join

OT

and

bisect it

centre F and radius FT, describe a circle and

cut the given circle at P, Q.


Join TP, TQ.
are
the
TQ
required tangents.
Since TF=FO, the circle, centre F, radius

Then TP,
Proof.

passes through O, and


.*.

TO

is

FT,

a diameter.

L TPO = 90 = Z. TQO. L in semicircle.


OQ are radii of the given circle.
TQ are tangents to the given circle.

But OP,
TP,

Q.E.F.

CONSTRUCTION 20
(1)

(2)

Draw the direct (or exterior) common tangents to two


Draw the transverse (or interior) common tangents
non-intersecting circles.

circles.

to

two

CONCISE GEOMETRY

284
(1)

Given two

Let

a, b

a>6.

B.

circles, centres A,

To Construct

their direct

common

be the radii of the

With A

tangents.

circles, centres A, B,

as centre

and a

and suppose

b as radius, describe a

and construct the tangents BP, BP' from B to this


Join AP, AP' and produce them to meet the circle,
circle.
radius a, in Q, Q'.
Through Q, Q' draw lines QR, Q'R'
parallel to PB, P'B.
circle

Then QR, Q'R'

are the required

Draw BR, BR'

Proof.

Q'R' at R,

parallel

common
to

tangents.
to meet

AQ, AQ'

QR,

R'.

]}y Construction,

PQRB

is

a parallelogram.

BR=PQ = AQ- AP-a -(-&)=&.

.'.

on the

lies

circle,

centre B, radius

b.

BP is a tangent, /. BPA = 90.


L RQA = 90 and L. BRQ - 90, by parallels.

Also, since
/.

a tangent at Q and R to the two circles.


Similarly it may be proved that Q'R' is also a

QR

.'.

is

common

tangent.

(2)

Two

Given.

non-intersecting circles, centres A, B.

To Construct the transverse common tangents.


Let

be the radii of the circles, centres A, B.


as centre and. a -f b as radius, describe a circle
construct the tangents BP, BP' to it from B.
a, b

With

Join AP, AP', cutting the

Through

Q,

Then QR, Q'R'

Q'

draw

circle radius

lines

are the required

QR,

at Q, Q'.

Q'R' parallel to

common

and

tangents.

PB,

CONSTRUCTIONS FOR BOOK


Through B draw BR, BR'
meet QR, Q'R' at R, R'.

Proof.

By

construction,

.*.

lies

PBRQ

on the

is

circle,

286

III

parallel to

to

AQ, AQ'

a parallelogram.

centre B, radius

6.

BP is a tangent, /_BPA=90.
*AQR = 90 and /. BRQ = 90, by parallels.

Also, since
/.

QR

.".

is

Similarly

a tangent at Q and R to the two circles.


may be proved that Q' R' is also a

it

common

tangent.
Q.B.F.

CONSTRUCTION 21

On

a given straight

line,

construct a segment of a circle containing

an angle equal to a given angle.

AT

B
\
\

c\
FIG. 299.

fiven a straight line

AB

To Construct on

and an angle X.

AB

a segment of a

circle

containing an

angle equal to Z.X.


A, make an angle BAG equal to /. X.
Draw AD perpendicular to AC.
Draw the perpendicular bisector of AB and

At

let it

meet

AD

at O.

With O as centre and


Then the segment of

OA

as radius, describe a circle.


on the side of AB opposite

this circle

the required segment.


O lies on the perpendicular bisector of AB,
Since
Proof.
OA = OB; /. the circle passes through B.
to

Since
/.

is

AC is perpendicular to the fadius


/.X== /.CAB = angle

OA,

AC

is

a tangent ;

in alternate segment.'
Q.E.F.

CONCISE GEOMETRY

286

CONSTRUCTION 22
Inscribe in a given circle a triangle equiangular to a given triangle.

FIG. 300.

circle and a triangle ABC.


To Construct a triangle inscribed

Given a

in

the circle

and equi-

ABC.
Take any point R on the circle and construct the tangent
XRY at R to the circle.
Draw chords RP, RQ so that ^PRY= Z.CBA and Z.QRX
angular to

= Z.CAB.

Join PQ.

Then

PQR

Proof.

and

.'.

the

is

the required triangle.

L PQR = L PRY,

alt.

segment.

Z.QPR= Z.QRX, alt. segment.


= Z.CAB.
remaining Z.QRP = the remaining
Q.K.F.

CONSTRUCTION 23
Describe about a given circle a triangle equiangular to a
given
triangle.

/Y

P
FIG. 301.

CONSTRUCTIONS FOR BOOK


and a triangle ABC.
To Construct a triangle with
and equiangular to ABC.

287

III

(liven a circle

Construct the centre

its

sides touching the circle

of the circle

OP

draw any radius

Draw radii OR, OS so


=
and
Z.ACB
Z.QOS = Z.ABC. Draw
^QOR
at
P, R, S, forming the triangle XYZ.
tangents
and produce

Then

XYZ

is

PO

But

the

the required triangle.

Z.ORZ = 90= Z.OPZ

Proof.

that

to Q.

since PZ,

RZ

are tangents.

ORZP is a cyclic quadrilateral.


L QOR = /. PZR, ext. L cyclic quad. = int. opp.
Z.QOR= /.ACB, constr,

Z.

Z.PZR-Z.ACB.
Similarly Z.PYS-ZABC.
the remaining Z.YXZ of
.'.

the

AXYZ^the

roinaiiiing

CONSTRUCTION 24
Construct a circle to pass through a given point
given circle at a given point B.

and

to touch a

FIG. 302.

Construct the centre O of the given circle.


Construct the perpendicular bisector of AB and produce
to cut OB, or OB produced at P.

With P

as centre

the required
Since P
Proof.
is

and PB

as radius, describe a circle.

it

This

circle.

lies

on the perpendicular bisector of AB,

PA=PB.
Since

lies

on OB, or

OB

produced, the two circles touch

at B.
Q.B.F.

CONCISE GEOMETRY

288

CONSTBUCTION 25
Construct a circle to touch a given circle and to touch a given line
ABC at a given point B on it.

'D
FIG. 303.

Construct the centre

of the given circle.

Construct the perpendicular

BD

AB

to

and cut

oft*

a part

BE

equal to the radius of the given circle.


Construct the perpendicular bisector of

OE

to cut

EB, or

EB

With P as centre and PB as


This

and produce

radius, describe a circle.

the required circle.


[There are two solutions according to which side
Let PO cut the given circle at Q.
Proof.

is

lies

on the perpendicular bisector

In Fig. 303,
/.

/.

the

lies

of

is

of

AC.]

and

OE.

PO=PE.

/.

Also

it

produced at P.

BE=OQ and PE=PO.


PB = PQ.

on

OQ

produced.

centre P, radius PB, passes through


touches the given circle at Q.
circle,

Q.E.F.

CONSTRUCTIONS FOR BOOK IV


CONSTRUCTION 26
Divide a given
(ii)

a given ratio

finite straight line in

(i)

internally,

externally.

p
'

FIG. 304.

Given two lines p,

and a

<?

To Construct

(i)

finite line

a point

in

AB.

AB

such that

AY v\
_=-.
XB q

Y in AB produced such that


=-?.
eBY ?
Draw any line AC and cut off successively AH=jt>, HK =
(i)
Join KB. Through H draw a line parallel to KB to
<.
cut AB at X.
(ii)
x '

a point

c,

Y -

B
FIG. 305.
(ii)

Draw any

HK =
KB to
19

line

AC

Join KB.

cut

AB

cut off

AH =^>, and

Through

produced at Y?

H draw

from

HA

cut off

a line parallel to

CONCISE GEOMETRY

290

CONSTBUCTION 27
Construct a fourth proportional to three given

lines.

,'Y

>

FIG. 306.

Given three lines of lengths

To Construct a

a, b, c units.

line of length

units,

such that - =

Draw any two lines OX, OY.


From OX cut off parts OP, OQ such that OP = a,
From OY cut off a part OR such that OR = c.

OQ = b.

Join PR.

Through Q, draw a line QS parallel to PR to meet


Then OS is the required fourth proportional.
Proof*

Since

PR

is parallel

to

OY

at

QS

OP OR
OQ~OS'

Q.E.F.

To

construct a third proportional to two given lines,


lengths a, b units, is the same as constructing a fourth
proportional to three lines of length a, 6, b units.

Note.

CONSTRUCTION 28

To

construct a polygon similar to a given polygon


corresponding sides are in a given ratio.

Given a polygon

To

QABCD \md

Cwutruot
A'3't=

AB

.
*
,

a ratio

XZ.

OA'B'O'D'

polygon

= XY
XZ

XY

and such that

such

that

9*' =

OA

291

CONSTRUCTIONS FOR BOOK IV


Join OB, OC.

Draw any
XY, XZ.

line

OQ

and cut

off parts

OP

OP',

equal to

Join PA.

Through P' draw P'A' parallel to PA to meet OA at A'.


Through A' draw A'B' parallel to AB to meet OB at B'.
Throiigh B' draw B'C' parallel to BC to meet OO at C'.
Through C' draw C'D' parallel to CD to meet OD at D'.
Then OA'B'C'D' is the required polygon.
Since A'B' is parallel to AB, AS OA'B', OAB are
Proof.
similar,
*"
.

.,

OA' ~
= A'B'

OA

__

AB

OB'

OB'

OB' = B'C' = OC'


-, and
,

Similarly

"
A1
Also

OA' = A'B' _
~~

OA

AB

OA'

OP'

&& ^ Qjy = D'O

BC """CD""
XY
OA--6P--55-

/. the sides of

OABCD

BO on.

DO"

OA'B'C'D' are proportional to the sides of

in the ratio

XY

XZ.

Further, by parallels, the polygons are equiangular.


/. the polygons are similar and tjjieir corresponding sides are
in the given ratio.
Q.E.F.

CONCISE GEOMETRY

292

CONSTRUCTION 29
Inscribe a square in a given triangle.

,'X'

Y''

_JY

V...

FIG. 308.

Given a triangle ABC.

To Construct a square with one


corners on AB and AC.

side on

BC

On BC describe the square BXYC.


Join AX, AY, cutting BC at X', Y'.
Through X', Y' draw X' B', Y'C' parallel to
cut AB, AC at B', C'.
Join B'C'.
Then B'X'Y'C' is the required square.
,
AB'
B'X'
,
x,
v
Proof.
By parallels

AB

Y'C'

YC

BX

and

XB

its

(or

other

YC)

AX'

X'Y'

AY'

AX

XY

AY

to

= AC'
"

AC

A =
AB AC

.'.

Since

.'.

B'X'Y'C'

is

B'C'

similar to

is parallel

BXYC and

to

is

BC and

.".

^^=
fA'.
BC AB

a square.
Q.E.F.

The following

is

a more general but

less

neat method.

>

293

CONSTRUCTIONS FOR BOOK IV

PQRS with PQ parallel to BC, and circumscribe a triangle FGH about this square equiangular to
ABC. [Draw SF, RF parallel to AB, AC produce FS,
FR to meet PQ produced at G, H.]
Divide BC at X' in the ratio GP PH.
Take any square

Then X' is one corner


and perpendiculars.

of the square

complete by parallels

CONSTRUCTION 30

mean

Construct a

Given two

proportional to two given lines.

lines of lengths a, b units.


i

To

Construct a line of length x units such that

Of

- = - or

x* = ab

FIG. 310.

Take a point O on a line and cut off from the


on opposite sides of O, parts OA, OB of lengths &, b units.
On AB as diameter, describe a circle.

METHOD L
line

Draw OP
Then

OP

Proof.

PQ

is

perpendicular to AB to cut the circle at P.


the required mean proportional.

is

Produce PO to meet the circle at Q.


a dhord perpendicular to the diameter AB,

PO = OQ.
PO OQ = AO OB,
/.

But

or

OP~

intersecting chords of a circle.

'

b
Q.E.F.

294

CONCISE GEOMETRY

MKTHOD
line

II.

Take a point

on a

on the name side of O, parts OA,

lino

OB

and cut

off

from the

of lengths a, b units.

On OB

as diameter, describe a circle.

Draw AQ perpendicular to OB to meet the circle at Q.


Join OQ. Then OQ is the required mean proportional.

OQB = 90 angle in semicircle.


a
OQ
tangent to the circle on QB as diameter.
But Z.QAB = 90, /. circle on QB as diameter
/.

Proof.

is

.".

passes

through A.

OQ 2 = OA

OQ.-...

OB, tangent property

of circle.

or
Q.B.P.

In practical constructions,
ferable to Method I.

Note.

Method

II. is often pre

CONSTRUCTION 31
(i)
(ii)

Construct a square equal in area to a given rectangle,


Construct a square equal in area to a given polygon.

FIG. 312.

(i)

Given a rectangle ABOD.


To Coitstruct a square of equal area.
Produce AB to E, making BE = BC.

295

CONSTRUCTIONS FOR BOOK IV


On AE

as diameter, describe a semicircle.

Produce

On BP
This

is

Proof.

CB

to

meet the semicircle at

P.

describe a square.
the required square.

By
.'.

the proof of Constr.

BE=BC.
BP 2 = AB BC = area
.

30,

BP 2 = AB.

BE, but

ABCD.

of

QE.F.
(ii)

Given any polygon.


To Construct a square of equal area.

FIG. 313.

By

the method of Constr. 12, reduce the polygon

to

an

XYZ.

equivalent triangle
the altitude XK and bisect

Draw
Use

(1) to construct

YZ at Q.
a square of area equal to a rectangle

sides are equal to YQ and


the square required.
Area of polygon = area of
Proof.

whose

This

XK.

is

A XYZ.

= JYZ XK.
= YQ.XK = square.
.

Q.E.F.

CONSTRUCTION 32
(i)

(ii)

(i)

Construct a triangle equal in area to one given triangle and


similar to another given triangle.

Construct a polygon equal in area to one given polygon and


similar to another given polygon!

A* ABO, PQR.
To Construct a AXBZ equal

Given two

Suppose

to

AABC

APQR placed with QR parallel to

and similar

BC.

to

CONCISE GEOMETRY

296
Through

A draw

Through B draw
Through H draw

AD

a line

parallel to

BC.

BH parallel to QP to meet AD at
HK parallel to PR to meet BC at

C Z

Fm.

H.

K.

314.

mean proportional BZ to BC, BK.


Z
draw
ZX parallel to KH to meet BH at
Through
Construct the

Then

XBZ

is

By

Proof.
/. to

X.

the required triangle.

AXBZ

parallels,

similar

is

AHBK

to

and

APQR.

BC
BK

BC BK
2
BK2
BK
AHBK
=
XBZ
ABC.
A
A
.

AABC

.'.

Q.E.F.
(ii)

Given two polygons F and

OSTUV.

To Construct a polygon OS'T'U'V


equal to

similar to

OSTUV

and

F.

JJ'

FIG. 315.

Reduce the two polygons F and


triangles

ABC, PQR

On

take a point S' such that

O& construct the

Then OS'T'U'V'

is

to

equivalent

respectively and proceed as

in

(i).

[See Fig. 314.]

On OS

OSTUV

OS =
OS QR

polygon* OS'T'UV similar to


the polygon required.

OSTUV.

^9

CONSTRUCTIONS FOR BOOK IV

OSTUV

os

ez 2

os'

S'T'U'V'
Proof.

QR 2

AXBZ AABC
APQR APQR

OSTUV'
OS'T'U'V'=F.

.*.

Q.E.F.

Note the use made of Theorems 58, 59.

CONSTRUCTION 33
Construct a

circle to pass

through two given points and touch a

line.

given

'O

FIG. 316.

Given two points A, B and a line CD.


To Construct a circle to pass through A, B and touch CD.
Join AB and produce it to meet CD at O.

mean

Construct the
off

from

CD

Construct the

circles

These are the required


Proof.

circles.

OA OB = OG 2 = OP 2 = OQ 2
OQ are tangents to the circles

Since

OP,

OG

to OA, OB, and cut


OP,
OQ equal to OG.
parts
P
and
A, B, Q.
through A, B,

proportional

on each side of

ABP, ABQ.
Q.E.F.

Note that the method


special case forms

fails if

an easy

AB

is

parallel to

CD.

This

exercise.

CONSTRUCTION 34
Construct a circle to pass throbgh two given p6int ahd touch a
given

circle.

CONCISE GEOMETRY

298

Given two points A, B and a

To Construct a
Construct any

circle S.

circle to pass

circle to pass

through A,

through A,

B and touch S.
S at C, D say

to cut

FIG. 317.

CD

Produce AB,

From O, draw

to

meet at O.

the tangerfts OP,

OQ

to S.

C6nstruct the circles through A, B, P and A, B, Q.

These are the required

circles.

OA OB = OC OD, property of intersecting chords.


= OP 2 = OQ 2 tangent property.
OP, OQ are tangents to the circles A, B, P and A, B, Q,

Proof.

.".

.".

these circles also touch S.


Q.E.F.

CONSTRUCTION 35
Construct a

given

circle to pass

through a given point and touch two

lines.

FIG. 318.

299

CONSTRUCTIONS FOR BOOK IV


Given two

AC

and a point D.
To Construct a circle to touch AB,
lines

AB,

[The centres of
of

all circles

AC

and pass through D.


AC lie on a bisector

touching AB,

L BAG.]

Draw any circle touching AB, AC and let P bo


P being in the same angle BAC as D*
Join

AD

and

centre

its

cut the circle at Q, Q'.

let it

parallel to QP to meet AP at E.
centre E and radius ED, describe a circle.

Draw DE
With

This circle will touch AB, AC.


Proof.

If

EH,

it

meet

AB

to

AB.

at H.

be proved to touch AC.


is obtained by drawing DE' parallel to Q'P

may

A second circle
to

are the perpendiculars from E,

centre E, radius ED, touches

circle,

Similarly

PX

AP

at

E'.

Q.K.P.

Take the image of D in the bisector of


L BAC, call it D'. By the method of Constr. 33, draw a circle to
pass through D, D' and to touch AB ; this circle will then touch
AC.

ANOTHER MKTHOD.

CONSTRUCTION 36
Construct a

circle to

touch two given lines and a given

circle.

B'

Given two

lines

AB,

To Construct a

AC

and a

circle to

circle S, centre* D, tacfius r.

touch AB, AC, and S.

CONCISE GEOMETRY

300
Draw two

lines A'B', A'C' parallel to

AB,

AC

and

at a dis-

tance r from them.


Constr. 35, draw a circle to touch A'B', A'C'
be its centre.
through D. Let

By

and to pass

With

draw a circle
touch AC and S.

will also

Proof.

to touch

as centre,

Let

P',

AB.

This

circle

Q' be the points of contact with A'B', A'C'.

Let OP', OQ', OD cut AB, AC, S at P, Q, E.


Then PP' = QQ' = r = ED ; but OP' = OQ' - OD.

OP=OQ=OE and OP, OQ

.'.

the circle, centre O, radius


There are in all four solutions

.".

Note.

solutions, since

two

are perp. to

AB, AC.

OP, touches AB, AC, S.


this construction gives two

can be drawn to touch A'B', A'C'


And by drawing A'B', A'C' at dis-

circles

and pass through D.


tance r from AB, AC on the other

side,

two other solutions

are obtained.

CONSTRUCTION 37
Bisect a triangle

by a

line parallel to

one

side.

Given a triangle ABC.


To Construct a line parallel to BC, cutting AB,

PQ

so that

Bisect

AB

at P,

at F.

Construct the

mean

From AB
Draw PQ

cut off

Then

is

PQ

AC

AABC.

bisects

proportional

AP

ecjual to

parallel to

AG

BC, cutting
the required Kne.

AC

Proof.

AABC AB 2

AB 2

between AF, AB.

AG.

AB

at Q.

CONSTRUCTIONS FOR BOOK IV

301

CONSTRUCTION 38
Divide a given line into two parts so that the rectangle contained by
the whole and one part is equal to the square oti the other part.

FIG. 321,

Given a line AB.

To Construct a point

Draw BC

on

perpendicular to

AB
AB

so that

AB BX = AX 2

and equal

to

|AB.

Join CA.

From CA
From AB
Then

is

cut off

Let

Proof.

CP
AX

equal to CB.

equal to AP.
the required point.

cut off

AB = 21.
BC = I

BX = 21-

AX 2 = Z 2 ^5 - 1)2 = J2 (6 - 2
AB.BX = AX 2

and
.'.

CONSTRUCTION 39
Construct an isosceles triangle, given one side and such that each
base angle is double of the vertical angle.

302

CONCISE GEOMETRY

Given a side AB.

To Construct a
With centre

ABC

triangle

A and

radius

AB

such that

AB = AC

and /.ABC

describe a circle.

construct a point P such that AB BP = AP 2


Place a chord BC in the circle such that BC = AP.

On AB

Join AC.

ABC

Then

is

the required triangle

AB BP = AP 2

Proof.

AB.BP-BC

.'.

BC

AS

AP - BC.

touches the circle

BCP,

BAC

AB = AC,
ButCB = AP,
But

APC.

are equiangular

Z ABC

is

common

CB = CP.
CP = PA.

/.
/.
.'.

.'.

but

ZBCP=ZCAP.

.'.

But

ZPAC-ZPCA.

Z PAC = Z PCB, /. Z BCA - 2 /. PAC


Z ABC - Z BCA - 2 Z BAC.
A

or 2

Z BAC.

--

FIG. 323.

Note.

Since the angles of a triangle add

Z.ABC
/.

BC

Z BCA =72
is

and

up

to

180.

Z BAC = 36.

the side of a regular decagon inscribed in the

circle.

C, draw CH perpendicular to AB and produce it


meet the circle at D then CH = HD and Z. CAD -- 72.

From

to

CD

is

the side of* a regular pentagon inscribed in the

circle.

The following result is useful :-yIf p and d are the lengths of the

sides of a regular

pentagon

303

CONSTRUCTIONS FOR BOOK IV


and a regular decagon inscribed

in a circle of radiua a,

AB = a, CD=*p, CB = d; it is required to
=
a 2 -f d2
prove that p
Since AB. BP=*BC 2 and BP=BA - AP=BA- BC = a-d.
2
2
2
/.
a(a-d) = d ora -ad-d = o.
2
2
2
=
From ACHB, CH + HB
CB
but CH - JCD = \p and
In Fig. 323,

let
2

/.

/>

.".

jt?

= a2 + d 2 - 2(a 2 - ad - d*).
= a 2 4- ^2 since a2 - ad ~ d 2 = o.
,

CONSTRUCTION 40
Inscribe

(i)

given

pentagon

regular

(ii)

a regular decagon

in

circle.

FIG. 324.

Let

EAB a diameter of the given circle.


AF be a radius perpendicular to AB.
Bisect AE at G.
With G as centre and GF as radius, describe a circle, cutting
AB at P join PF.
Then AP and PF are equal in length to the sides of a regular
be the centre and

Let

decagon and a regular pentagoji inscribed in the

circle.

The

regular figures are therefore constructed by placing


chords in the circle end to end equal to these lines.

Proof.

FS

From GF

cut off

equal to FR.

GR

equal to

GA

froip

FA

cut off

CONCISE GEOMETRY

304

2
Constr. 38, FA AS= FS
=
=
NowGR GAandGP QF, .'. AP=RF- = or.
BPAS.
But AF = AB, .*.
BA. BP = AP2
.'.
/.
by Constr. 39, AP is equal to a side of the

Then by

regular

decagon.

But
/.

AP2 + AF2 =PF2


PF

is

equal

to

a side

(See pp. 302, 303.)

of

the

regular

pentagon.

306

CONCISE GEOMETKY

NOTES

307

308

CONCISE GEOMETRY

NOTES

809

310

CONCISE GEOMETRY

NOTES

311

312

CONCISE GEOMETRY

GLOSSARY AND INDEX


ACUTE angle : any angle less than 90.
Alternate angle, 5.
Altitude the altitude of a triangle is
the perpendicular from any vertex
to the opposite side.
Angle in a semicircle ; an angle
whose vertex lies on the circumference and whose arms pass through
the extremities of a diameter.

Diameter a chord of a
through tho centre.
:

circle passing

Apollonius* theorem, 226.


Arc of a circle : any part of the circumference.
Area of circle, 86.
Area of triangle and trapezium, 27.

Elevation, angle of, 145.


Fquilateral having all its sides equal.
Equivalent equal in area.
Exeentre, 97.
:

External bisector:

BAC

if

is

an

angle and if BA is produced to X,


is called
the line bisecting Z.
the external bisector of
BAC.

CAX

a figure with six sides.


Horizontal line a line ]>erpen<1wular

Hexagon

to a vertical line.

Bisect

divide into two equal parts.

the side of a rightangled triangle opposite the right

Hypotenuse

Centroid, 98.
Chord : the line joining any two
points on the circumference of a
circle.

Circle
the locus of a point which is
at a constant distance (called the
radius) from a fixed point (called
the centre) is called the circumference of a circle.
:

angle.
Identities, geometrical, 228.
Image, 93.

Incentre, 97.
Isosceles triangle
two sides equal.

a triangle with

Locus, 248.

Circumcentre, 97.

Common

Mean

tangents, 283

Complementary angles
sum is 90.

angles whose

Concentric having the same centre.


in all respects.
Congruent
equal^
The symbol is zz.
Corresponding angles, 5.
:

proportional, 121.

Median

the line joining a vertex of


a triangle to tho mid-point of the
opposite side.
Mensuration formulae, 86.
:

Nine point

Cyclic quadrilateral a quadrilateral


whose four corners lie on a circle.

circle, 102.

Decagon a figure with ten sides.


Degree $Vth part of a right angle.
Depression, angle of, 145.
the line joining two opDiagonal
:

posite corners of a quadrilateral.

Obtuse angle an angle greater than


90* and less than 180.
Octagon a figure with eight sides.
:

Orthocentre, 98.
Parallel lines, 208

Parallelogram, 22.
318

314

GLOSSARY AND INDEX

Pedal triangle, 98.


Pentagon a figure with five sides.
Perimeter: the sum of the lengths
of the sides bounding a figure.
Perpendicular at right angles to.
Playfair's axiom, 208.
:

Projection, 224.
Proportional (third or fourth), 290.
Pythagoras' theorem, 222.

a semicircle
ment,

if less

is called a major sega minor segment.

Similar, 257.
Square, 22.

Supplementary angles

sum

Symbols

= equal in
==

X
Reflex angle

an angle greater than

180.
Regular polygon
all its sides

Rhombus,

and

a polygon having
all its angles equal.

22.

angles whose

area.

congruent.
the difference between

Rectangle, 22.
Reflection, 93.

180.

is

and

is

sented by X
greater than.

>
<

~repreY.

less than.

Z angle.
A triangle.
umtn

Right angle, 205.

||

Sector of a circle : the area bounded


by two radii of a circle and the arc
they cut off.
Segment of a circle the area bounded
by a chord of a circle and the arc
it cuts off ; a segment greater than
:

'
(

parallelogram.
circumference.

Tangent, 243.
Trapezium, 22.
a line which when
produced passes through the centre

Vertical line

of the earth.

ANSWERS.
1.

Where only one form

the unit

is

of unit occurs in the question, the nature of

omitted in the Answer.

Answers are not given when intermediate work

is unnecessary.
Results are usually given correct to three figures, and for angles to
the nearest quarter of a degree.

2.

3.

EXERCISE
7. 135.
6; 11; 22.
300 ; (v) 990.
13. 120.
15. 120.

8.

5.
9.

18.

(iv)

120.

25. 110

29.

19.
;

149J

80.

111J.

5.

122.
2a;-180

72.

20. 5*.

24. 40.

27.46.

28.111.

81. 180 -IK.

32.

27.

y=

32.

~, y=6,

8-a?

XU

&

III (p. 10)

80.
10. 36.
16. 37.
8.
21. |(g - y) + 90.

36.

15.

120.

20.

33.

EXERCISE IV
3. (i)

90, 45

a=60Jy.

5.

68.

(ii)

72, 36.

62.

23.

60, 60,

36.*

315

17.

x=c-a-b.

(p. 16)

9.

67i.

80.
86.
25. 162.
12.

6.

EXEROJSE.V
7.

31.

28. 6.

10, 18, 42.

C*

7.

65.

(ii)

7.

14. Least is

19.

167.

17.

93.

6.

72.

EXERCISE
13. 80.

112i

12.

2474.

251.

83;

10. 20.
16.

26.15.

I (p. 2)

(p.

70.

6.

33.

23)

3=360-2?.
26f.

316

CONCISE GEOMETRY
EXERCISE VI

EXERCISE X
13.

(p. 28)

(p. 49)

7.

EXERCISE XI
21. 12*

EXERCISE XII
1.

9-16.

2.

13.'

3.

6.

5-38.

7.

3-46.

8. 5.

11. 4-8.

12. 3*12.

13.

a;

40.

7.

54

99.

2.

55",

3.

8.

105.

9.

110.
72.

57)

(p.

11-5.

EXERCISE
1.

(p. 52)

17*.

4.

7*V

9.

4.

5.

10.

+ xy = a? -Jr.

XIII
4.

10.

(p.

37.
124.

8'58, 0*58.
8.

14. 5'22.

62)
5.

11.

107.
54.

6.

100; 110

12. 105.

SWEIIS
XIV
W

1.

62.

2.

117

6.

94, 8.

7.

120.

3.

2t>,

8.

:*>

(p.

5.

103",

90, 77, 90.

(p. 72)
u

1.

30, 45, 105 or 15, 30, 135.

4.

3:1.

5.

1.

3.

2.

2'5, 1-5, 4'6.

3.

8, 4, 3.

4.

5 '3, 3 '6, 4 '5.

5.

10-5, 1-6.

6.

7.

1J.

8.

32, 8.

9.

3.

10.

1-5, 2-5.

11.

'5,

19-1, 12.

14.

7, 1.

15.

4-45, 11-125.

74, 22J, 150 or 22 J


46, 37.

EXERCISE XVI

13.

18.1-44,36.

(p.

4.

2-1.

5.

5-89.

6.

11. 84-8.
IS,

314

20. 65'4; 78'5.

21.

100,000, 000 sq. m.

24. 20'1.

25,

119; 44-0.

35. 29'3.

12.

16. 5 -3\/2~-0'757'.

(p. 87)

4'57.

158-5.

28.

12,

628 yd., 31,420 sq. yd.

25.

9.

15.

77)

2-5.

EXERCISE XVIII
25-1 in., 50-3 sq. in.

30 \ 1274".

20.1+^2 = 2-41.

19.24.

1.

68)

58, 64.

KXEKCISE XV

204.

2f.
29. 77-4.

36. 102'5.

7.

12. 21-5.

18.

17.

18.

jj.

4.

26. 78-5.

2.

57

628; 408.
288,

22, 8'2.
27.

514; 500;

828'5 sq. ft,


37. 8 ; 14 ; 1,

30.

EXERCISE XIX

(p.

94)

32. 20 in.

EXERCISK XXI

0'8.

18'.

(p.

100)

3.

1-1.

8.

3'2.

14.

3.

19.

48

23.

9'21.

96

9'0.

34. 11'8.

38. 6'86;

137; 186.

CONCISE GEOMETRY

318

EXERCISE XXIII
6.

2.

3.

6. (i) 6

11. 7-07

12

(ii)

13*04.

I j.

2-31

(iii)

4.

21^.

(iv)

0707.

12.

(p. 122)

10.

13.

9.

40.

2.

16

10.

0.

6.

18. 2s. 3d.

3-35.

2.

2.

7140.

3.

6.

2-77.

7.

S. 37

W.

9.

8-42 mi. N. 12

W.

12. 10-6.

13. 321.

18. 326.

19.

59

20. E.

1.

(i)

2-5.

7.

(xii)

11-3

(iii)

(xiii)

2'59

S.

8'49

12.

10.

10tfi.

11.

16.

49J.

17. (i) 4-96

18.

(i)

25i

(ii)

49i8'25

21. 4*67.

22. 7.

27. 4-62.

28.

(i)

676

(iii)

7'67;

(ii)

29. 6-22.

17. 84'0.

(i)

6.

2'64.

150)

(p.

(vii)

(iii)

5'18

10*1

(iv)

63J

(viii)

5'66, 3'53

574.

8.

14. 2*55.
;

24. 6-16.
(iii)

6'68

9.

15. 7*13, 3'63.


(v) 3'82.

19. 8 -64.

20. 3 '53.

25. 4*26.

26. 4*96.

(iv)

478;

5 '23.

(v) 7'82; (vi)

871.

30. 5 '34.

(p. 153)

21|.
(p.

155)

50-0; (iii) 147; (iv) 6; (v) 48; (vi) 9'43 ; (vii) 45-1;
2. 15*0.8. 575.
4. 4*57.
5. 30.
(ix) ?l t (X) 18.
9. 4'07.
10. 5*80 or W6.
13. 29'1
36j. 8. 40.

(ii)

7.

148)
2'83.

(xv) 1044.

EXERCISE XXXI
10;
(viii) 28

1.

(p.

479

EXERCISE XXX
11.

10. 1*63.

32.

W.
N. 31J W.
23. 34 '4.

(iv) 6 -21.

7'10;

5.

22. 137.

8.

7*0 mi. N. 34

21. 177.

13. 5'41.

62J.

(ii)

23. 6'09.

6 -64.

(vii)

(xiv) 8*87

10' 8".

8.

11. 34 -8 mi.

(v)

4.

16. 2'59.

6'13.

2'93

(iv)

145)

(p.

15. 85'3.

EXERCISE XXIX
36 50'

162.

135)

(p.

86| N.

91'9,

14.

3 sq. in.

6.

5'17 mi.

EXERCISE XXVIII
6.

21. 40 -6

13' 9".

10. E.

31.

19. 9if.

40'.

EXERCISE XXVII
94-3.

15. 512.

5. 9J.

EXERCISE XXVI

1.

8. 4'2.

101J.

13. 4if.

(p. 132)

12.

4.

45' or 14

69. 81

30. 4*8.

127)

(p.

5i.

EXERCISE XXV
3, 15.

3-36.

H.

8.

6J.

^?!L

9.

12.

9.

17. 6.

16. 1*024.

3.

4,

gi-^j*-^

12 sq. ft.
3*75 sq. in.

1.

2 or 10.

7.

J?5l

EXERCISE XXIV
1.

5.

ANSWERS
EXERCISE XXXII
6.

1-93.

5.

6-65.

3-61.

7.

8.

6*82".

9.

319
(p.

168)

j.

ffl

EXERCISE XXXIII
27. 1-94.

17. 0-64, 1'16, 1-93, 5 '80.

4'47.

18. 8-13.

320.

9.

14. 2'66.

7*5.

35. 4'16.

7.

7'2.

2.

37.

(i)2-89;

(ii)

7.

19.

5-00.

10.

4. 7 '36 or

5.

4-81.

300.

6.

37.

67J.

41.

49.

375.

112.

68. 13".

81. 12;

89. 2.

107.

29.

65. 4[p(y

69. 15, 9.

72.

102.

55, 40.

57.

9.

(p.

180)

(p.

181)

75.

CD

33.

rt.

a = 540-a-&-c.
46.

angles.

80.

'

61. 2J.

18'4(5).

73. 300.

77. 7*5.

T^8iT416

88. (ii)>/*

60, 80.

109. 13.

AB

6.3*29.

5. 5 '29.

+ r) + q(r + s) + x(s~y)].

85. 2.

47.

178)

66. 4*24.

5'66.

16. 3'63.

or -7'22.

42. *?

z=180-a-&-a;-y.

53. ^(xy-{-yz).

99.

110.

10.

64, 5-5, 2-5, 17'3.

1-21.

3'83.

7.

REVISION PAPERS
1.

(p.

1 '36.

a =7 '22 or -2*22, y=2'22

578.

4.

11. 4'12

23. 5'36.

EXERCISE XXXVII
3.

1'32.

175)

(p.

10'3.

EXERCISE XXXVI
3.6-08.

17.

29. 5'87, 2*23.

11J.

22. 2*68.

21. 4-55.

1.6-325.

16. 5 -80.

25. 6'Ob*, 4 '02.

EXERCISE XXXV
1.

26. 3'11.

172)

(p.

1-66.

15.

24, 5'60, 2'14.

30. 6-89, 4'89.

20. 2*27.

24. 2'13.

1-4(5.

28. 4'61.

EXERCISE XXXIV
6.

161)

(p.

18.

10. 3'1 7,

115.

106.

78. 2-16.

(iii)a;>6J.

a* + * h

lh

16.

118. E. 25

N.

9V
~.

131. 17.

133. on

147. f.
132.

155. 9i, J.

159. 6J in.

167. 6, 10, 14 in.

170.

174. 4'47.

175. 24 -4 in.

177. 6, 5.

179. 2*2.

182. 0-69 or 23'3.

189. 3-2, 1*2, 4-4.


197. 4.
199. 4-8.

10,

on

20.

194. 687.

186. 43'2.

137. 6.

144.

183. 5| f 2J T .
187. 2.
\95. 2, 2?.

I'KINTKD BY

AND GIHH LIMIT*


KDINKUROH

M OK IU SON

You might also like